EAQ TESTING Q & A

A client is diagnosed with condyloma acuminatum. Which finding in the client supports the diagnosis?

1
Moist, fleshy projections on the penis
2
Pus-filled ulcers on the penis
3
Swollen penis with tight foreskin
4
Macules on the penis

1 Moist, fleshy projections on the penis with single or multiple projections is a clinical manifestation of condyloma acuminatum. Macules on the penis or scrotum are clinical manifestations of penile erythema. Chancroid is manifested by pus-filled ulcers on the penis. A swollen penis with tight foreskin is a clinical manifestation of paraphimosis

A nurse is caring for a client with a diagnosis of cancer of the prostate. The nurse should teach the client that which serum level will be monitored throughout the course of the disease?

1
Albumin
2
Prostate-specific antigen (PSA)
3
Blood urea nitrogen (BUN)
4
Creatinine

.2..The PSA is an indication of cancer of the prostate; the higher the level, the greater the tumor burden. Albumin is a protein that is an indicator of nutritional and fluid status. Increased creatinine or BUN levels may be caused by impaired renal function as a result of blockage by an enlarged prostate but do not indicate that metastasis has occurred.

The nurse is caring for a client with ureteral colic. To prevent the development of renal calculi in the future, which strategy should be included in the client's plan of care?

1
Excluding milk products from the diet
2
Interventions to decrease the serum creatinine level
3
Instructing the client to drink 8 to 10 glasses of water daily
4
A urinary output goal of 2000 mL per 24 hours

.3..Increasing fluid intake dilutes the urine, and crystals are less likely to coalesce and form calculi. An elevated serum creatinine has no relationship to the formation of renal calculi. Calcium restriction is necessary only if calculi have a calcium phosphate basis. Producing only 2000 mL of urine per 24 hours is inadequate. STUDY TIP: Answer every question. A question without an answer is the same as a wrong answer. Go ahead and guess. You have studied for the test and you know the material well. You are not making a random guess based on no information. You are guessing based on what you have learned and your best assessment of the question.

When assessing a client during peritoneal dialysis, a nurse observes that drainage of the dialysate from the peritoneal cavity has ceased before the required volume has returned. What should the nurse instruct the client to do?

1
Turn from side to side
2
Deep breathe and cough
3
Drink a glass of water
4
Rotate the catheter periodically

1...Turning from side to side will change the position of the catheter, thereby freeing the drainage holes of the tubing, which may be obstructed. Drinking a glass of water and deep breathing and coughing do not influence drainage of dialysate from the peritoneal cavity. The position of the catheter should be changed only by the primary healthcare provider.

A nurse is teaching a birthing/prenatal class about breast-feeding. Which hormone stimulates the production of milk during lactation?

1
Progesterone

2
Prolactin

3
Inhibin

4
Estrogen

2...Prolactin is the hormone that initiates and produces milk during lactation. Inhibin prevents the secretions of follicle stimulating hormone and gonadotropin releasing hormone. Estrogen and progesterone are the sex hormones produced by the ovaries.

A client is diagnosed with condyloma acuminatum. Which finding in the client supports the diagnosis?

1
Moist, fleshy projections on the penis

2
Pus-filled ulcers on the penis

3
Swollen penis with tight foreskin

4
Macules on the penis

1...Moist, fleshy projections on the penis with single or multiple projections is a clinical manifestation of condyloma acuminatum. Macules on the penis or scrotum are clinical manifestations of penile erythema. Chancroid is manifested by pus-filled ulcers on the penis. A swollen penis with tight foreskin is a clinical manifestation of paraphimosis.

Which statement regarding erythropoietin is true?

1
An erythropoietin deficiency is associated with renal failure.

2
Erythropoietin is released only when there is adequate blood flow.

3
An erythropoietin deficiency causes diabetes.

4
Erythropoietin is released by the pancreas.

...1 correct Erythropoietin is produced by the kidneys; its deficiency occurs in renal failure. Erythropoietin is released by the kidneys, not the pancreas. Erythropoietin deficiency causes anemia. Erythropoietin is secreted in response to hypoxia, which results in decreased oxygenated blood flow to the tissues.

Which drug prescribed to a client with a urinary tract infection (UTI) turns urine reddish-orange in color?

1
Nitrofurantoin

2
Ciprofloxacin

3
Phenazopyridine

4
Amoxicillin

...3...Phenazopyridine is a topical anesthetic that is used to treat pain or burning sensation associated with urination. It also imparts a characteristic orange or red color to urine. Amoxicillin is a penicillin form that could cause pseudomembranous colitis as a complication; it is not associated with reddish-orange colored urine. Ciprofloxacin is a quinolone antibiotic used for treating UTIs and can cause serious cardiac dysrhythmias and sunburns. It is not, however, responsible for reddish-orange colored urine. Nitrofurantoin is an antimicrobial medication prescribed for UTIs. This drug may affect the kidneys but is not associated with reddish-orange colored urine.

A client arrives at a health clinic reporting hematuria, frequency, urgency, and pain on urination. Which diagnosis will the nurse most likely observe written in the client's medical record?

1
Pyelonephritis

2
Nephrotic syndrome

3
Cystitis

4
Chronic glomerulonephritis

...3...Cystitis is an inflammation of the bladder that causes frequency and urgency of urination, pain on micturition, and hematuria. Chronic glomerulonephritis is a disease of the kidney that is associated with manifestations of systemic circulatory overload. Nephrotic syndrome is a condition of increased glomerular permeability characterized by severe proteinuria. Pyelonephritis is a diffuse, pyogenic infection of the pelvis and parenchyma of the kidney that causes flank pain, chills, fever, and weakness.

The nurse providing postoperative care for a client who had kidney surgery reviews the client's urinalysis results. Which urinary finding should the nurse conclude needs to be reported to the primary healthcare provider?

1
Acidic pH

2
Glucose negative

3
Presence of large proteins

4
Bacteria negative

...3..The glomeruli are not permeable to large proteins such as albumin or red blood cells (RBCs), and it is abnormal if albumin or RBCs are identified in the urine; their presence should be reported. The urine can be acidic; normal pH is 4.0 to 8.0. Glucose and bacteria should be negative; these are normal findings.

What is the function of the structure labeled in the given figure?
3204932528

1
Holds the fetus

2
Secretes ovum

3
Serves as entry to the sperm

4
Massages the ovaries

...4...The structure labeled in the figure represents the fallopian tubes, fingerlike projections that massage the ovaries to facilitate ovum extraction. The ovaries produce ovum. The uterus accommodates the fetus. The cervix serves as an entry to the sperm and is also involved in expulsion of menses.

Tests reveal that a client has phosphatic renal calculi. The nurse teaches the client that the diet may include which food item?

1
Chocolate

2
Apples

3
Cheddar cheese

4
Rye bread

Test-Taking Tip: If you are unable to answer a multiple-choice question immediately, eliminate the alternatives that you know are incorrect and proceed from that point. The same goes for a multiple-response question that requires you to choose two or more of the given alternatives. If a fill-in-the-blank question poses a problem, read the situation and essential information carefully and then formulate your response.

...Apples are low in phosphate; fresh fruit is low in phosphorus. Chocolate contains more phosphate than apples. Rye bread contains more phosphate than apples. Cheese is made with milk, which contains phosphate and should be avoided. Dairy products are high in phosphorus.

A nurse educates the client about the relationship between the kidneys and blood pressure. Which term should the nurse use to describe the part of the kidney that senses changes in blood pressure?

1
Macula densa

2
Calices

3
Glomerulus

4
Juxtaglomerular cells

...1..The macula densa, a part of the distal convoluted tubule, consists of cells that sense changes in the volume and pressure of blood. Calices are cup-like structures, present at the end of each papilla that collect urine. The glomerulus is the initial part of the nephron, which filters blood to make urine. Juxtaglomerular cells secrete renin. Renin is produced when sensing cells in the macula densa sense changes in blood volume and pressure.

The urinalysis report of a client reveals cloudy urine. What does a nurse infer from the client's report?

1
The client has a biliary obstruction.

2
The client has diabetic ketoacidosis.

3
The client has been on a starvation diet.

4
The client has a urinary infection.

4 The urine becomes cloudy when an infection is present due to the presence of leukocytes. Therefore the nurse concludes that the client has a urinary infection. In cases of biliary obstruction, the urine contains bilirubin. The presence of ketones in the urine indicates diabetic ketoacidosis or prolonged starvation

A client is scheduled to have an indwelling urinary catheter inserted before abdominal surgery. The nurse should insert the catheter in what location in the illustration?
3142352560

1 d
2 c
3 a
4 b

Correct 4 b Option B is the urethral orifice, which anatomically is between the clitoris and the vagina; it is the opening into the urethra, the tubular structure that drains urine from the bladder. Option A is the clitoris, which is situated beneath the anterior commissure, partially hidden between the anterior extremities of the labia minora. Option C is the opening of the vagina; it is the part of the female genitalia that forms a canal from the vaginal orifice through the vestibule to the uterine cervix. Option D is the anus; it is the terminal end of the anal canal that is connected to the rectum; the rectum is a portion of the large intestine that is between the anal canal and the descending sigmoid colon. Test-Taking Tip: Practicing a few relaxation techniques may prove helpful on the day of an examination. Relaxation techniques such as deep breathing, imagery, head rolling, shoulder shrugging, rotating and stretching of the neck, leg lifts, and heel lifts with feet flat on the floor can effectively reduce tension while causing little or no distraction to those around you. It is recommended that you practice one or two of these techniques intermittently to avoid becoming tense. The more anxious and tense you become, the longer it will take you to relax.

A client is admitted to the hospital with severe flank pain, nausea, and hematuria caused by a ureteral calculus. What should be the nurse's initial intervention?

1
Obtain a urine specimen for culture.

2
Administer the prescribed analgesic.

3
increase oral fluid intake.

4
Strain all urine output.

2...Pain of renal colic may be excruciating; unless relief is obtained, the client will be unable to cooperate with other therapy. Urine can be saved and strained after the client's priority needs are met. Increasing fluid intake may or may not be helpful. If the stone is large the fluid can build up, leading to hydronephrosis; however, if the stone is small, fluids may help flush the stone. Although a culture generally is prescribed, this is not the priority when a client has severe pain.

Which retrograde procedure involves the examination of the ureters and the renal pelvises?

1
Urethrogram
2
Cystogram
3
Pyelogram
4
Voiding cystourethrogram

3... A pyelogram is a retrograde examination of the ureters and the pelvis of both kidneys. A cystogram is a retrograde examination of the bladder. An urethrogram is a retrograde examination of the urethra. A voiding cystourethrogram is used to determine whether urine is flowing backward into the urethra.

Which parts of the nephron are the sites for the regulation of water balance? Select all that apply.

1
Loop of Henle

2
Descending limb (DL)

3
Proximal convoluted tubule (PCT)

4
Bowman capsule (BC)

5
Glomerulus

1, 2 A loop of Henle is a part of a nephron that continues from the proximal convoluted tubule (PCT). It is permeable to water, sodium chloride, and urea and is a site for the regulation of water balance. The descending limb (DL) continues from the loop of Henle. It is permeable to water, sodium chloride, and urea and is a site for the regulation of water balance. The glomerulus is a site of glomerular filtration. The Bowman capsule (BC) is a site of the collection of glomerular filtrate. The proximal convoluted tubule (PCT) is a site for the reabsorption of sodium, chloride, glucose, water, amino acids, potassium, and calcium.

A nurse evaluates that a client with chronic kidney disease understands an adequate source of high biologic-value (HBV) protein when the client selects which food from the menu?

1
Apple juice

2
Raw carrots

3
Cottage cheese

4
whole wheat bread

3... Cottage cheese contains more protein than the other choices. Apple juice is a source of vitamins A and C, not protein. Raw carrots are a carbohydrate source and contain beta-carotene. Whole wheat bread is a source of carbohydrates and fiber. Test-Taking Tip: Come to your test prep with a positive attitude about yourself, your nursing knowledge, and your test-taking abilities. A positive attitude is achieved through self-confidence gained by effective study. This means (a) answering questions (assessment), (b) organizing study time (planning), (c) reading and further study (implementation), and (d) answering questions (evaluation).

The nurse observes a client with kidney failure has increased rate and depth of breathing. Which laboratory parameter does the nurse suspect is associated with this client's condition?

1
Potassium 8 mEq/L

2
Phosphorous 7 mg/dL

3
Hemoglobin 10 g/dL

4
Bicarbonate 15 mEq/L

4 An increased rate and depth of breathing is called Kussmaul respiration and occurs due to metabolic acidosis in clients with kidney disease. Serum bicarbonate level decreases in metabolic acidosis. The normal range of serum bicarbonate is 23-30 mEq/L. Therefore the bicarbonate value of 15 mEq/L is associated with Kussmaul respirations in the client. The normal serum potassium is 3.5-5 mEq/L. Therefore a potassium level of 8 mEq/L indicates hyperkalemia and is associated with changes in cardiac rate and rhythm. The normal range of hemoglobin is 12-16 g/dL in females and 14-18 g/dL in males. Therefore a Hgb of 10 g/dL indicates anemia; this is associated with fatigue, pallor, and shortness of breath. The normal range of serum phosphorous is 3-4.5 mg/dL. Therefore a phosphorous value of 7 mg/dL indicates hyperphosphatemia, which is associated with hypocalcemia and demineralization of bone.

Which part of the kidney produces the hormone bradykinin?

1
Juxtaglomerular cells of the arterioles

2
Kidney tissues

3
Kidney parenchyma

4
Renin-producing granular cells

1 The juxtaglomerular cells of the arterioles produce the hormone bradykinin, which increases blood flow and vascular permeability. The kidney tissues produce prostaglandins that regulate internal blood flow by vasodilation or vasoconstriction. The kidney parenchyma produces erythropoietin that stimulates the bone marrow to make red blood cells. The renin-producing granular cells produce the renin hormone that raises blood pressure as a result of angiotensin and aldosterone secretion.

The nurse is performing a physical examination of a client by placing the left hand on the back and supporting the client's right side between the rib cage and the iliac crest. Which physical assessment maneuver is the nurse performing on this client?

1
Palpation

2
Percussion

3
Auscultation

4
Inspection

1 The physical assessment involves inspection, palpation, percussion, and auscultation. During palpation of the right kidney, the nurse places the left hand behind and supports the client's right side between the ribcage and the iliac crest. During an inspection, the nurse assesses the client for changes in skin, abdomen, weight, face, and extremities. During percussion, the nurse strikes the fist of one hand against the dorsal surface of the other hand, which is placed flat along the post costovertebral angle (CVA) margin. While performing auscultation, the nurse uses the bell of the stethoscope over both CVAs and in the upper abdominal quadrants.

A client who has been told she needs a hysterectomy for cervical cancer is upset about being unable to have a third child. Which action should the nurse take next?

1
Emphasize that she does have two children already.

2
Encourage her to focus on her own recovery.

3
Ensure that other treatment options for her will be explored.

4
Evaluate her willingness to pursue adoption.

3 Although a hysterectomy may be performed, conservative management may include cervical conization and laser treatment that do not preclude future pregnancies; clients have a right to be informed by their primary healthcare provider of all treatment options. Willingness to pursue adoption currently is not the issue for this client. Encouraging her to focus on her own recovery and emphasizing that she does have two children already negate the client's feelings.

A client is admitted to the hospital from the emergency department with a diagnosis of urolithiasis. The nurse reviews the client's clinical record and performs an admission assessment. Which is the priority nursing action?

1
Collect a urine specimen for culture and sensitivity.

2
Administer the prescribed morphine.

3
Strain the client's urine.

4
Place the client in the high-Fowler position.

2 Pain relief is the priority. Clients report that ureteral colic is excruciatingly painful. Once pain is under control and the client is comfortable, other medical and nursing interventions can be implemented. Although straining all urine is required, pain relief is the priority. Once the client is medicated for pain, the urine that was set aside can be strained. The high-Fowler position is not necessary. The client can be assisted to assume a position of comfort. The urine was sent for a culture and sensitivity in the emergency department.

A client is taught how to change the dressing and how to care for a recently inserted nephrostomy tube. On the day of discharge the client states, "I hope I can handle all this at home; it's a lot to remember." Which is the best response by the nurse?

1
"Oh, a family member can do it for you."

2
"Perhaps you can stay in the hospital another day."

3
"I'm sure you can do it."

4
"You seem to be nervous about going home."

4 4 The response "You seem to be nervous about going home" is the best reply. Reflection conveys acceptance and encourages further communication. The response "I'm sure you can do it" is false reassurance that does not help to reduce anxiety. The response "Oh, a family member can do it for you" provides false reassurance and removes the focus from the client's needs. The response "Perhaps you can stay in the hospital another day" is unrealistic, and it is too late to suggest this

A client with ascites has a paracentesis, and 1500 mL of fluid is removed. For which immediate response is it most important for the nurse to monitor?

1
Respiratory congestion

2
Increase in temperature

3
Rapid, thready pulse

4
Decreased peristalsis

...3 Fluid shifts from the intravascular compartment into the abdominal cavity, causing hypovolemia. A rapid, thready pulse , which is indicative of shock, is a compensatory response to this shift. Decreased peristalsis is not likely to occur in the immediate period. After a paracentesis, intravascular fluid shifts into the abdominal cavity, not into the lungs. Increase in temperature is not the priority; body temperature usually is not affected immediately; an infection will take several days

The nurse is performing an assessment of a client's reproductive system. Which action should the nurse take?

1
Maintain friendly demeanor with the client during assessment

2
Ask about sexual practices at the beginning of assessment

3
Ask about menstrual history at the beginning of assessment

4
Maintain gender-specific terms while questioning during assessment

...3 It is necessary to gather health information as part of an assessment of the reproductive system. The nurse should always start the questioning with minimally sensitive information such as menstrual history. This will help the client adjust gradually. The nurse should maintain a professional demeanor while assessing or taking a reproductive health history. Sensitive information, such as client's sexual practices, should be asked after basic and less-sensitive topics. The nurse should make use of gender-neutral terms while questioning the client regarding their sexual partners.

Which hormone is crucial for ovulation and complete maturation of a client's ovarian follicles?

Incorrect 1

Gonadotropin releasing hormone

Correct 2

Luteinizing hormone

3

Follicle stimulating hormone

4

Human chorionic gonadotropin hormone

Ovulation and complete maturation of ovarian follicles can only take place in the presence of luteinizing hormone. However, follicle stimulating hormone initiates maturation of the follicles. Gonadotropin releasing hormone stimulates the pituitary gland to release follicle stimulating hormone and luteinizing hormone. Human chorionic gonadotropin hormone is released after implantation and is responsible for secretion of progesterone and estrogen during pregnancy.

Test-Taking Tip: You have at least a 25% chance of selecting the correct response in multiple-choice items. If you are uncertain about a question, eliminate the choices that you believe are wrong and then call on your knowledge, skills, and abilities to choose from the remaining responses.

...

A female client has a history of recurrent urinary tract infections. What should the nurse include in the teaching plan when educating the client about health practices that may help decrease future urinary tract infections?

1

"Void at least every 6 hours."

2

"Wipe from back to front after toileting."

Correct 3

"Wear cotton underpants."

4

"Increase foods containing alkaline ash in the diet."

Cotton allows air to circulate and does not retain moisture the way synthetic fabrics do; microorganisms multiply in warm, moist environments. Voiding frequently helps to flush ascending microorganisms from the bladder, thereby reducing the risk for urinary tract infections; holding urine for 6 hours can lead to urinary tract infections. Foods high in acid, not alkaline, ash help to acidify urine; this urine is less likely to support bacterial growth. Alkaline urine promotes bacterial growth. Wiping from back to front after toileting may transfer bacteria from the perianal area toward the urinary meatus, which will increase the risk for urinary tract infection.

...

A nurse is providing client teaching to a woman who has recurrent urinary tract infections. Which information should the nurse include concerning the reason why women are more susceptible to urinary tract infections than men?

Correct 1

The length of the urethra

2

The continuity of mucous membranes

3

Inadequate fluid intake

4

Poor hygienic practices

The length of the urethra is shorter in women than in men; therefore, microorganisms have a shorter distance to travel to reach the bladder. The proximity of the meatus to the anus in women also increases the incidence of urinary tract infections. Fluid intake may or may not be adequate in both men and women and does not account for the difference. Hygienic practices can be inadequate in men or women. Mucous membranes are continuous in both men and women

...

The nurse provides discharge instructions to a male client who had an ureterolithotomy. The client has a history of recurrent urinary tract infections (UTIs). For which indicators of a UTI should the nurse instruct the client?

1

Pain radiating to the external genitalia

Correct 2

Urgency or frequency of urination

3

The inability to maintain an erection

4

An increase of ketones in the urine

Urgency or frequency of urination occur with a urinary tract infection because of bladder irritability; burning on urination and fever are additional signs of a UTI. Increase of ketones is associated with diabetes mellitus, starvation, or dehydration. The inability to maintain an erection is not related to a UTI. Pain radiating to the external genitalia is a symptom of a urinary calculus, not infection.

...

Radium inserted in the vagina of a client now is being removed. Which safety precaution should the nurse employ when assisting with the radium removal?

1

Document how long the radium was in place and when it was removed.

2

Clean the radium in ether or alcohol.

Correct 3

Ensure that long forceps are available for removing the radium.

4

Wear foil-lined rubber gloves while handling the radium.

Radium must be handled with long forceps because distance helps limit exposure. A nurse does not clean radium implants. Foil-lined rubber gloves do not provide adequate shielding from the gamma rays emitted by radium. The amount and duration of exposure are important in assessing the effect on the client; however, documentation will not affect safety during removal.

...

Which part of the nephron secretes creatinine required for elimination?

Correct 1

Proximal tubule

2

Glomerulus

Incorrect 3

Loop of Henle

4

Collecting duct

The proximal tubule of the nephron secretes creatinine and hydrogen ions. It also reabsorbs water and electrolytes. The glomerulus filters the blood selectively. The ascending loop of Henle reabsorbs sodium and chloride, whereas the descending loop of Henle concentrates the filtrate. The collecting duct reabsorbs water.

...

Which phase of the woman's sexual response is characterized by elevation of the uterus?

1

Orgasmic phase

Correct 2

Plateau phase

3

Excitation phase

4

Resolution phase

The plateau phase occurs after the excitation phase, and excitation is maintained through the plateau phase, wherein the vagina expands and the uterus is elevated. Therefore elevation of the uterus is a characteristic of the plateau phase of a woman's sexual response. The orgasmic phase is characterized by uterine and vaginal contractions. In the excitation phase, the clitoris is congested and vaginal lubrication increases. The resolution phase is characterized by returning to the preexisting state.

...

Which structures are included in the external genitalia in males? Select all that apply.

Correct 1

Scrotum

2

Urethra

Correct 3

Penis

4

Seminal vesicles

Incorrect 5

Testes

The male reproductive system is divided into primary reproductive organs and secondary reproductive organs. Secondary reproductive organs include ducts, sex glands, and external genitalia. The external genitalia consists of the penis and the scrotum. Testes are the primary reproductive organs. The urethra is the duct, and the seminal vesicles are sex glands.

...

A nurse writes a goal of preventing renal calculi in a care plan for a client with paraplegia. Which information most likely caused the nurse to write this goal?

Correct 1

Accelerated bone demineralization

Incorrect 2

High fluid intake

3

Inadequate kidney function

4

Increased intake of calcium

Calcium that has left the bones as a response to prolonged inactivity enters the blood and may precipitate in the kidneys, forming calculi. Increased fluid intake is helpful in preventing this condition by preventing urinary stasis. Calcium intake usually is limited to prevent the increased risk for calculi. Calculi may develop despite adequate kidney function; kidney function may be impaired by the presence of calculi and urinary tract infections associated with urinary stasis or repeated catheterizations.

...

A pathology report states that a client's urinary calculus is composed of uric acid. Which food item should the nurse instruct the client to avoid?

Correct 1

Liver

2

Vegetables

3

Milk

4

Cheese

Uric acid stones are controlled by a low-purine diet. Foods high in purine, such as organ meats and extracts, should be avoided. Milk should be avoided with calcium, not uric acid, stones. Cheese or animal protein should be avoided with cystine, not uric acid, stones. Vegetables do not have to be avoided.

Test-Taking Tip: Avoid spending excessive time on any one question. Most questions can be answered in 1 to 2 minutes.

...

A client with chronic renal failure has been on hemodialysis for 2 years. The client communicates with the nurse in the dialysis unit in an angry, critical manner and is frequently noncompliant with medications and diet. The nurse can best intervene by first considering that the client's behavior is most likely for which reason?

1

A constructive method of accepting reality

Correct 2

A defense against underlying depression and fear

3

An attempt to punish the nursing staff

4

An effort to maintain life and to live it as fully as possible

Both hostility and noncompliance are forms of anger that are associated with grieving. The client's behavior is not a conscious attempt to hurt others but a way to relieve and reduce anxiety within the self. The client's behavior is a self-destructive method of coping, which can result in death. The client's behavior is an effort to maintain control over a situation that is really controlling the client; it is an unconscious method of coping, and noncompliance may be a form of denial.

...

Which statement indicates the nurse has a correct understanding about kidney ultrasonography?

1
Kidney ultrasonography primarily makes use of iodinated contrast dye.

2
Kidney ultrasonography gives three-dimensional information regarding kidneys.

3
Kidney ultrasonography makes use of sound waves and has minimal risk.

4
Kidney ultrasonography is performed on the client with an empty bladder.

3 Kidney ultrasonography is a minimal risk diagnostic procedure. Ultrasonography makes use of sound waves which, when reflected from internal organs of varying density, will produce the images of the kidneys, bladder, and associated structures on the display screen. While a dye can be used in computed tomography (CT), it is not the primary method. Generally kidney ultrasonography is performed on the client with full bladder. A CT gives three-dimensional information about the kidney and associated structures.

A student nurse is caring for a client with chronic kidney failure who is to be treated with continuous ambulatory peritoneal dialysis (CAPD). Which statement by the student nurse indicates to the primary nurse that the student nurse understands the purpose of this therapy?

Incorrect 1

"It decreases the need for immobility because it clears toxins in short and intermittent periods."

2

"It provides continuous contact of dialyzer and blood to clear toxins by ultrafiltration."

3

"It exchanges and cleanses blood by correction of electrolytes and excretion of creatinine."

Correct 4

"It uses the peritoneum as a semipermeable membrane to clear toxins by osmosis and diffusion."

Diffusion moves particles from an area of greater concentration to an area of lesser concentration; osmosis moves fluid from an area of lesser to an area of greater concentration of particles, thereby removing waste products into the dialysate, which is then drained from the abdomen. The principle of ultrafiltration involves a pressure gradient, which is associated with hemodialysis, not peritoneal dialysis. Peritoneal dialysis uses the peritoneal membrane to indirectly cleanse the blood. Dialysate does not clear toxins in a short time; exchanges may occur four or five times a day.

...

Which statement is true regarding the functions of kidney hormones?

1

Prostaglandin increases blood flow and vascular permeability.

2

Erythropoietin promotes the absorption of calcium in the gastrointestinal tract (GI) tract.

3

Bradykinin regulates intrarenal blood flow via vasodilation or vasoconstriction.

Correct 4

Renin raises blood pressure because of angiotensin and aldosterone secretion.

Renin is a kidney hormone that raises blood pressure as a result of angiotensin and aldosterone secretion. Prostaglandin is a kidney hormone that regulates intrarenal blood flow via vasodilation or vasoconstriction. Bradykinin is a kidney hormone that increases blood flow and vascular permeability. Erythropoietin is a kidney hormone that stimulates the bone marrow to make red blood cells.

...

After prostate surgery a client's indwelling catheter and continuous bladder irrigation (CBI) are to be removed. The nurse discusses with the client the procedure and what to expect after the removal. Which statement by the client indicates teaching by the nurse is understood?

Correct 1

"I probably will experience some burning on urination."

2

"I probably will be unable to urinate."

3

"I probably will produce dark red urine."

4

"I probably will have dilute urine."

Because of the trauma to the mucous membranes of the urinary tract, burning on urination is an expected response that should subside gradually. The urine should no longer be dilute after the continuous bladder irrigation is discontinued and removed. However, the urine may have a slight pink tinge because of the trauma from the surgery and the presence of the catheter. An inability to urinate should not occur unless the indwelling catheter is removed too soon and there is still edema of the urethra. Production of dark red urine is a sign of hemorrhage, which should not occur.

...

A nurse is evaluating a client's understanding of peritoneal dialysis. Which information in the client's response indicates an understanding of the purpose of the procedure?

1
Providing fluid for intracellular spaces

2
Cleaning the peritoneal membrane

3
Reestablishing kidney function

4
Removing toxins in addition to other metabolic wastes

4 Peritoneal dialysis uses the peritoneum as a selectively permeable membrane for diffusion of toxins and wastes from the blood into the dialyzing solution. Peritoneal dialysis acts as a substitute for kidney function; it does not reestablish kidney function. The dialysate does not clean the peritoneal membrane; the semipermeable membrane allows toxins and wastes to pass into the dialysate within the abdominal cavity. Fluid in the abdominal cavity does not enter the intracellular compartment

A client who had a transurethral resection of the prostate is transferred to the postanesthesia care unit with an intravenous (IV) line and a urinary retention catheter. For which major complication is it most important for the nurse to assess during the immediate postoperative period?

Correct 1

Hemorrhage

2

Phlebitis

3

Leakage around the IV catheter

4

Sepsis

After transurethral surgery , hemorrhage is common because of venous oozing and bleeding from many small arteries in the area. Sepsis is unusual, and if it occurs it will manifest later in the postoperative course. Phlebitis is assessed for, but it is not the most important complication. Hemorrhage is more important than phlebitis. Leaking around the IV catheter is not a major complication.

Test-Taking Tip: Work with a study group to create and take practice tests. Think of the kinds of questions you would ask if you were composing the test. Consider what would be a good question, what would be the right answer, and what would be other answers that would appear right but would in fact be incorrect.

...

Which hormone is crucial in maintaining the implanted egg at its site?

1

Testosterone

Correct 2

Progesterone

3

Estrogen

4

Inhibin

Progesterone is necessary to maintain an implanted egg. Inhibin regulates the release of follicle-stimulating hormone (FSH) and gonadotropin-releasing hormone (GnRH). Estrogen plays a vital role in the development and maintenance of secondary sexual characteristics. Testosterone is important for bone strength and development of muscle mass.

...

Which urinalysis finding indicates a urinary tract infection?

1
Presence of leukoesterase

2
Presence of crystals

3
Presence of ketones

4
Presence of bilirubin

1 Leukoesterases are released by white blood cells as a response to an infection or inflammation. Therefore, the presence of this chemical in urine indicates a urinary tract infection. The presence of crystals in the urine indicates that the specimen had been allowed to stand. Presence of bilirubin in the urine indicates anorexia nervosa, diabetic ketoacidosis, and prolonged fasting. The presence of ketones indicates diabetic ketoacidosis.

To help prevent a cycle of recurring urinary tract infections in a female client, which instruction should the nurse share?

1

"Take bubble baths regularly."

Correct 2

"Urinate as soon as possible after intercourse."

3

"Douche regularly with alkaline agents."

4

"Increase your daily intake of citrus juice."

Intercourse may cause urethral inflammation, increasing the risk of infection; voiding clears the urinary meatus and urethra of microorganisms. Most fruit juices, with the exception of cranberry juice, cause alkaline urine, which promotes bacterial growth. Douching is no longer recommended because it alters the vaginal flora. Bubble baths can promote urinary tract infections.

STUDY TIP: The old standbys of enough sleep and adequate nutritional intake also help keep excessive stress at bay. Although nursing students learn about the body's energy needs in anatomy and physiology classes, somehow they tend to forget that glucose is necessary for brain cells to work. Skipping breakfast or lunch or surviving on junk food puts the brain at a disadvantage.

...

A nurse is counseling a woman who had recurrent urinary tract infections. Which factor should the nurse explain is the reason why women are at a greater risk than men for contracting a urinary tract infection?

1

Altered urinary pH

Correct 2

Proximity of the urethra to the anus

3

Hormonal secretions

4

Juxtaposition of the bladder

Because a woman's urethra is closer to the anus than a man's, it is at greater risk for becoming contaminated. Urinary pH is within the same range in both men and women. Hormonal secretions have no effect on the development of bladder infections. The position of the bladder is the same in men and women.

...

A client in a nursing home is diagnosed with urethritis. What should the nurse plan to do before initiating antibiotic therapy prescribed by the primary healthcare provider?

Correct 1

Obtain a urine specimen for culture and sensitivity.

Incorrect 2

Teach the client how to perform perineal care.

3

Prepare for urinary catheterization.

4

Start a 24-hour urine collection.

The causative organism should be isolated before starting antibiotic therapy; a culture and sensitivity should be obtained before starting the antibiotic. A 24-hour urine test will not determine the infective organism causing the problem. Catheterization is not a routine intervention for urethritis. Although client teaching is important, it is not the priority at this time.

STUDY TIP: Determine whether you are a "lark" or an "owl." Larks, day people, do best getting up early and studying during daylight hours. Owls, night people, are more alert after dark and can remain up late at night studying, catching up on needed sleep during daylight hours. It is better to work with natural biorhythms than to try to conform to an arbitrary schedule. You will absorb material more quickly and retain it better if you use your most alert periods of each day for study. Of course, it is necessary to work around class and clinical schedules. Owls should attempt to register in afternoon or evening lectures and clinical sections; larks do better with morning lectures and day clinical sections.

...

Which hormone influences kidney function?

Incorrect 1

Erythropoietin

Correct 2

Aldosterone

3

Renin

4

Bradykinin

Released from the adrenal cortex, aldosterone influences kidney function. Renin, bradykinin, and erythropoietin are kidney hormones.

...

Which client's urine specific gravity level is abnormal?

1

1.028

2

1.006

Correct 3

1.041

4

1.012

The normal specific gravity of urine lies between 1.005 and 1.030. A specific gravity value of 1.041 is higher than the normal range; therefore, it's abnormal. The specific gravity values of urine such as 1.006, 1.012, and 1.028 lie in the normal range.

...

Which diagnostic tests are used to measure the kidney size of a client with kidney dysfunction? Select all that apply.

Correct 1

Computed tomography (CT)

Incorrect 2

Cystography

Correct 3

Radiography

Incorrect 4

Cystoscopy

Incorrect 5

Cystourethrography

A radiography and a computed tomography (CT) are diagnostic tests used to measure kidney size in clients with kidney dysfunction. A cystoscopy is used to identify abnormalities of the bladder wall in clients with kidney dysfunction. A cystography and a cystourethrography are used to examine the structure of the urethra and to detect backward flow of urine

...

A client who is recovering from deep partial-thickness burns develops chills, fever, flank pain, and malaise. The primary healthcare provider makes a tentative diagnosis of urinary tract infection. Which diagnostic tests should the nurse expect the primary healthcare provider to prescribe to confirm this diagnosis?

1

Creatinine clearance and albumin/globulin (A/G) ratio

Correct 2

Urinalysis and urine culture and sensitivity

3

Cystoscopy and bilirubin level

4

Specific gravity and pH of the urine

The client's manifestations may indicate a urinary tract infection; a culture of the urine will identify the microorganism, and sensitivity will identify the most appropriate antibiotic. A cystoscopy is too invasive as a screening procedure; altered bilirubin results indicate liver or biliary problems, not urinary signs and symptoms. Creatinine clearance reflects renal function; A/G ratio reflects liver function. Although an increased urine specific gravity may indicate red blood cells (RBCs), white blood cells (WBCs), or casts in the urine, which are associated with urinary tract infection, it will not identify the causative organism.

...

A male client has discharge from the penis. Gonorrhea is suspected. To obtain a specimen for a culture, what should the nurse do?

1

Swab the discharge when it appears on the prepuce.

2

Instruct the client to provide a semen specimen.

3

Instruct the client how to obtain a clean catch specimen of urine.

Correct 4

Swab the drainage directly from the urethra to obtain a specimen.

Swabbing the drainage directly from the urethra obtains a specimen uncontaminated by environmental organisms. Instructing the client to provide a semen specimen is not as accurate as obtaining the purulent discharge from the site of origin. Swabbing the discharge when it appears on the prepuce will contaminate the specimen with organisms external to the body. Teaching the client how to obtain a clean catch specimen of urine will dilute and possibly contaminate the specimen.

...

What instruction regarding sample collection should the nurse give a client who is ordered a clean-catch urine specimen?

1

Collect the last urine sample voided in the night

2

Keep the urine sample in dry warm area if delay is anticipated

Correct 3

Urinate small amount, stop flow, fill half of cup

4

Send the urine sample to the laboratory within 6 hours of collection

The nurse instructs the client to always collect the midstream urine to send as a test specimen. The client should be instructed to cleanse the perineum with the wipe provided, urinate a small amount, and then stop the flow. The client should then position the specimen cup a few inches from the urethra and resume urination, filling the cup at least half way. The client is asked to collect the first sample voided in the morning because the urine is highly concentrated in the morning. Keeping the urine sample in the refrigerator helps reduce bacterial growth due to alkaline environment. The cells in the urine sample begin to break down in alkalinity, and therefore the client is instructed to send the sample to the laboratory as soon as collected.

...

The primary healthcare provider suspects pituitary gland dysfunction in a female client. Which diagnostic test would the primary healthcare provider suggest to the client?

Incorrect 1

Estradiol test

2

Sims-Huhner test

Correct 3

Prolactin test

4

Papanicolaou (Pap) test

A prolactin test is used to detect pituitary gland dysfunction that causes amenorrhea. Therefore the primary healthcare provider would suggest that the client have a prolactin test to determine if the client does or does not have any pituitary gland dysfunction. Estradiol is tested to determine functioning of the ovaries. In men, the estradiol test is used to detect testicular tumors. The Sims-Huhner test is used to evaluate the hostility of the cervix for passage of sperm from the vagina into the uterus. The Papanicolaou (Pap) test detects malignancies, particularly cervical cancer.

...

The nurse is educating student nurses about the anatomy and physiology of the kidneys. What term does the nurse explain is used for the tip of the pyramid of a kidney?

Incorrect 1

Calyx

Correct 2

Papilla

3

Renal column

4

Renal pelvis

Pyramids are components of renal medulla, and the tip of each pyramid is called a papilla. A calyx is a structure that collects the urine at the end of each pyramid. The renal calices join together to form the renal pelvis. A renal column is a cortical tissue that separates the pyramids.

...

Which part of the female reproductive system produces testosterone in females?

Correct 1

Ovary

2

Ovarian follicle

3

Uterus

4

Fallopian tube

Testosterone is an androgen, and in females, androgens are produced by the ovaries and adrenal glands. The uterus holds the fetus during pregnancy. Fallopian tubes facilitate fertilization of oocyte and sperm. An ovarian follicle is a collection of oocytes in the ovary.

...

Which electrolyte deficiency triggers the secretion of renin?

1

Calcium

2

Potassium

Correct 3

Sodium

4

Chloride

Low sodium ion concentration causes decreased blood volume, thereby resulting in decreased perfusion. Decreased blood volume triggers the release of renin from the juxtaglomerular cells. Deficiencies of calcium, chloride, and potassium do not stimulate the secretion of renin.

...

A client is admitted to the hospital with a diagnosis of cancer of the liver with ascites and is scheduled for a paracentesis. Which nursing intervention is appropriate to include in the client's plan of care?

1
Marking the anesthetic insertion site

2
Cleansing the intestinal tract

3
Discussing the operating room set-up

4
Having the client void before the procedure

4 Because the trocar is inserted below the umbilicus, having the client void decreases the danger of puncturing the bladder. Cleansing the intestinal tract is not necessary because the gastrointestinal tract is not involved in a paracentesis. The primary healthcare provider, not the nurse, uses a local anesthetic to block pain during the insertion of the aspirating needle; marking the site usually is not done. A paracentesis usually is performed in a treatment room or at the client's bedside, not in the operating room.

A nurse is reviewing the clinical record of a client with a diagnosis of benign prostatic hyperplasia (BPH). Which test result will the nurse check to confirm the diagnosis?

Correct 1

Biopsy of prostatic tissue

2

Rectal examination

3

Serum phosphatase level

4

Massage of prostatic fluid

A definitive diagnosis of the cellular changes associated with benign prostatic hyperplasia (BPH) is made by biopsy, with subsequent microscopic evaluation. Palpation of the prostate gland through rectal examination is not a definitive diagnosis; this only reveals size and configuration of the prostate. The serum phosphatase level will provide information for prostatic cancer; a definitive diagnosis cannot be made with this test for BPH. A sample of prostatic fluid helps to diagnosis prostatitis.

...

A nurse is assessing the urine of a client with a urinary tract infection. For which characteristic should the nurse assess each specimen of urine?

1
Viscosity

2
Clarity

3
Specific gravity

4
Glucose level

Cloudy urine usually indicates drainage associated with infection. Viscosity is a characteristic that is not measurable in urine. Urinary glucose levels are not affected by urinary tract infections. Specific gravity yields information related to fluid balance.

2 Cloudy urine usually indicates drainage associated with infection. Viscosity is a characteristic that is not measurable in urine. Urinary glucose levels are not affected by urinary tract infections. Specific gravity yields information related to fluid balance.

What does the presence of ketones in the urine of a client with renal dysfunction indicate?

Correct 1

Anorexia nervosa

2

Cystitis

3

Heart failure

Incorrect 4

Urinary calculi

The body of a client with anorexia nervosa produces ketones as an alternate source of fuel for muscles and organs. Increased red blood cells (RBCs) in the urine indicate cystitis. Increased specific gravity of the urine indicates heart failure. The presence of casts in the urine indicates urinary calculi.

...

Which urodynamic study provides information on bladder capacity, bladder pressure, and voiding reflexes?

Correct 1

Cystometrography (CMG)

2

Renal arteriography

3

Radiography

Incorrect 4

Electromyography (EMG)

Cystometrography (CMG) is an urodynamic study that provides information on bladder capacity, bladder pressure, and voiding reflexes. Radiography is a diagnostic test for clients with disorders of kidney and urinary system to screen for the presence of two kidneys, to measure kidney size, and to detect gross obstruction in kidneys or urinary tract. Renal arteriography is a diagnostic study used to determine renal blood vessel size and abnormalities. Electromyography (EMG) is an urodynamic study used to test the strength of perineal muscles in voiding.

...

The nurse is educating new parents about circumcision. Which structure of the penis would this nurse tell the parents is removed during circumcision?

Correct 1

Prepuce

2

Vas deferens

3

Glans

4

Epididymis

Circumcision is a procedure that involves removal of the prepuce, a skin fold over the glans. The glans is the tip of the penis. The epididymis is the internal structure that promotes transportation of the sperm. The vas deferens carries the sperm from the epididymis to the ejaculatory duct.

...

A 55-year-old client reports cessation of menstrual periods. Which term best describes the client's condition?

1

Menarche

Correct 2

Menopause

3

Amenorrhea

4

Dyspareunia

Cessation of menstruation is called menopause; this is an aging process and occurs due to functional decline of the ovaries. The first episode of menstrual bleeding is called menarche. Dyspareunia refers to painful sexual intercourse. The absence of menstruation is called amenorrhea.

...

A client has a kidney transplant. The nurse should monitor for which assessment findings associated with rejection of the transplant? Select all that apply.

Correct 1

Oliguria

2

Polydipsia

Correct 3

Fever

Correct 4

Weight gain

Incorrect 5

Jaundice

Fever is a characteristic of the systemic inflammatory response to the antigen (transplanted kidney). Oliguria or anuria occurs when the transplanted kidney is rejected and fails to function. Weight gain can occur from fluid retention when the transplanted kidney fails to function or as a result of steroid therapy; this response must be assessed further. Jaundice is unrelated to rejection. Polydipsia is associated with diabetes mellitus; it is not a clinical manifestation of rejection.

...

What is the cup-like structure that collects a client's urine and is located at the end of each papilla?

Correct 1

Calyx

2

Capsule

3

Renal cortex

4

Renal columns

The calyx is a cup-like structure that collects urine and is located at the end of each papilla. The outer surface of the kidney consists of fibrous tissue and is called the capsule. The renal cortex is the outer tissue layer. The renal columns are the cortical tissue that dip down into the interior of the kidney and separate the pyramids.

...

Which is a primary glomerular disease?

Correct 1

Chronic glomerulonephritis

2

Diabetic glomerulopathy

3

Systemic lupus erythematosus (SLE)

4

Hemolytic-uremic syndrome

Chronic glomerulonephritis is a primary glomerular disease. Diabetic glomerulopathy, hemolytic-uremic syndrome, and systemic lupus erythematosus (SLE) are secondary glomerular diseases.

...

The nurse is caring for a client with ureteral colic. To prevent the development of renal calculi in the future, which strategy should be included in the client's plan of care?
1
Excluding milk products from the diet
2
Interventions to decrease the serum creatinine level
Correct 3
Instructing the client to drink 8 to 10 glasses of water daily
4
A urinary output goal of 2000 mL per 24 hours

Correct 3 Increasing fluid intake dilutes the urine, and crystals are less likely to coalesce and form calculi. An elevated serum creatinine has no relationship to the formation of renal calculi. Calcium restriction is necessary only if calculi have a calcium phosphate basis. Producing only 2000 mL of urine per 24 hours is inadequate. STUDY TIP: Answer every question. A question without an answer is the same as a wrong answer. Go ahead and guess. You have studied for the test and you know the material well. You are not making a random guess based on no information. You are guessing based on what you have learned and your best assessment of the question

Which drug prescribed to a client with a urinary tract infection (UTI) turns urine reddish-orange in color?

1
Nitrofurantoin
2
Ciprofloxacin
3
Phenazopyridine
4
Amoxicillin

correct:3 Phenazopyridine is a topical anesthetic that is used to treat pain or burning sensation associated with urination. It also imparts a characteristic orange or red color to urine. Amoxicillin is a penicillin form that could cause pseudomembranous colitis as a complication; it is not associated with reddish-orange colored urine. Ciprofloxacin is a quinolone antibiotic used for treating UTIs and can cause serious cardiac dysrhythmias and sunburns. It is not, however, responsible for reddish-orange colored urine. Nitrofurantoin is an antimicrobial medication prescribed for UTIs. This drug may affect the kidneys but is not associated with reddish-orange colored urine.

When assessing a client during peritoneal dialysis, a nurse observes that drainage of the dialysate from the peritoneal cavity has ceased before the required volume has returned. What should the nurse instruct the client to do?
1
Turn from side to side
2
Deep breathe and cough
3
Drink a glass of water
4
Rotate the catheter periodically

CORRECT 1 Turning from side to side will change the position of the catheter, thereby freeing the drainage holes of the tubing, which may be obstructed. Drinking a glass of water and deep breathing and coughing do not influence drainage of dialysate from the peritoneal cavity. The position of the catheter should be changed only by the primary healthcare provider

A client arrives at a health clinic reporting hematuria, frequency, urgency, and pain on urination. Which diagnosis will the nurse most likely observe written in the client's medical record?
1
Pyelonephritis
2
Nephrotic syndrome
3
Cystitis
4
Chronic glomerulonephritis

3 CORRECT Cystitis is an inflammation of the bladder that causes frequency and urgency of urination, pain on micturition, and hematuria. Chronic glomerulonephritis is a disease of the kidney that is associated with manifestations of systemic circulatory overload. Nephrotic syndrome is a condition of increased glomerular permeability characterized by severe proteinuria. Pyelonephritis is a diffuse, pyogenic infection of the pelvis and parenchyma of the kidney that causes flank pain, chills, fever, and weakness

The nurse providing postoperative care for a client who had kidney surgery reviews the client's urinalysis results. Which urinary finding should the nurse conclude needs to be reported to the primary healthcare provider?
1
Acidic pH
2
Glucose negative
3
Presence of large proteins
4
Bacteria negative

3 The glomeruli are not permeable to large proteins such as albumin or red blood cells (RBCs), and it is abnormal if albumin or RBCs are identified in the urine; their presence should be reported. The urine can be acidic; normal pH is 4.0 to 8.0. Glucose and bacteria should be negative; these are normal findings.

A nurse is teaching a birthing/prenatal class about breast-feeding. Which hormone stimulates the production of milk during lactation?
1
Progesterone
2
Prolactin
3
Inhibin
4
Estrogen

answer 2 Prolactin is the hormone that initiates and produces milk during lactation. Inhibin prevents the secretions of follicle stimulating hormone and gonadotropin releasing hormone. Estrogen and progesterone are the sex hormones produced by the ovaries.

What is the function of the structure labeled in the given figure?

picture wont copy onto here....

1
Holds the fetus
2
Secretes ovum
3
Serves as entry to the sperm
4
Massages the ovaries

correct 4 The structure labeled in the figure represents the fallopian tubes, fingerlike projections that massage the ovaries to facilitate ovum extraction. The ovaries produce ovum. The uterus accommodates the fetus. The cervix serves as an entry to the sperm and is also involved in expulsion of menses.

Tests reveal that a client has phosphatic renal calculi. The nurse teaches the client that the diet may include which food item?
1
Chocolate
2
Apples
3
Cheddar cheese
4
Rye bread

answer: 2 Apples are low in phosphate; fresh fruit is low in phosphorus. Chocolate contains more phosphate than apples. Rye bread contains more phosphate than apples. Cheese is made with milk, which contains phosphate and should be avoided. Dairy products are high in phosphorus. Test-Taking Tip: If you are unable to answer a multiple-choice question immediately, eliminate the alternatives that you know are incorrect and proceed from that point. The same goes for a multiple-response question that requires you to choose two or more of the given alternatives. If a fill-in-the-blank question poses a problem, read the situation and essential information carefully and then formulate your response.

A nurse educates the client about the relationship between the kidneys and blood pressure. Which term should the nurse use to describe the part of the kidney that senses changes in blood pressure?

1
Macula densa
2
Calices
3
Glomerulus
4
Juxtaglomerular cells

correct 1 The macula densa, a part of the distal convoluted tubule, consists of cells that sense changes in the volume and pressure of blood. Calices are cup-like structures, present at the end of each papilla that collect urine. The glomerulus is the initial part of the nephron, which filters blood to make urine. Juxtaglomerular cells secrete renin. Renin is produced when sensing cells in the macula densa sense changes in blood volume and pressure.

The nurse is caring for a client in the postanesthesia care unit. The client had a suprapubic prostatectomy for cancer of the prostate and has a continuous bladder irrigation (CBI) in place. Which primary goal is the nurse trying to achieve with the CBI?
1
Provide continuous pressure on the prostatic fossa.
2
Stimulate continuous formation of urine.
3
Facilitate the measurement of urinary output.
4
Prevent the development of clots in the bladder.

4 A continuous flushing of the bladder dilutes the bloody urine and empties the bladder, preventing clots. Fluid instilled into the bladder does not affect kidney function. Urinary output can be measured regardless of the amount of fluid instilled. The urinary retention catheter is not designed to exert pressure on the prostatic fossa.

A client is diagnosed as having invasive cancer of the bladder, and brachytherapy is scheduled. What should the nurse expect the client to demonstrate that indicates success of this therapy?
1
Shrinkage of the tumor on scanning
2
Increase in pulse strength
3
Increase in the quantity of white blood cells (WBCs)
4
Decrease in urine output

1 Brachytherapy, in which isotope seeds are implanted in the tumor, interferes with cell multiplication, which should control the growth and metastasis of cancerous tumors. Radiation affects healthy as well as abnormal cells; urinary output will increase with successful therapy. With brachytherapy of the bladder, increase in pulse strength is not a sign of success. Bone marrow sites may be affected by radiation, resulting in a reduction of WBCs.

What is the concentration of estradiol in the blood during the follicular phase of the menstrual cycle?
1
159 pg/mL
2
165 pg/mL
3
171 pg/mL
4
130 pg/mL

4 In the follicular phase of the menstrual cycle, 20-150 pg/mL of estradiol is released. Therefore 130 pg/mL of estradiol would be its concentration during the follicular phase of the menstrual cycle. Concentrations of 159, 165, and 171 pg/mL are greater than the reference range.

A client with an indwelling catheter is prescribed a urinalysis test. Arrange the steps involved in the collection of the urine sample in correct order.

1.Clamp drainage tubing

2.Attach a sterile syringe

3.Aspirate the urine

4. Remove the clamp

In a client with an indwelling catheter, urine sample is collected by first applying a clamp, distal to the injection port, on to the drainage tubing. Then the injection port cap of the catheter drainage tubing is cleaned with alcohol. The next step is to attach a 5-mL sterile syringe into the port and aspirate the urine sample required. Finally the clamp is removed so that the drainage is resumed.

A nurse is notified that the latest potassium level for a client in acute kidney injury is 6.2 mEq (6.2 mmol/L). Which action should the nurse take first?
1
Call the laboratory to repeat the test.
2
Take vital signs and notify the primary healthcare provider.
3
Obtain an electrocardiogram (ECG) strip and obtain an antiarrhythmic medication.
4
Alert the cardiac arrest team.

2 Vital signs monitor the cardiopulmonary status; the primary healthcare provider must treat this hyperkalemia to prevent cardiac dysrhythmias. The cardiac arrest team responds to a cardiac arrest; there is no sign of arrest in this client. A repeat laboratory test will take time and probably reaffirm the original results; the client needs medical attention. Although obtaining an ECG strip is appropriate, obtaining an antiarrhythmic is premature; vital signs and medical attention is needed first.

A client is transferred to the postanesthesia care unit after undergoing a pyelolithotomy. The client's urinary output is 50 mL/hr. What should the nurse do?
1
Record the output as an expected finding.
2
Milk the client's nephrostomy tube.
3
Encourage the client to drink oral fluids.
4
Notify the primary healthcare provider.

1 An output of 50 mL/hr is adequate; when urine output drops below 20 to 30 mL/hr, it may indicate renal failure, and the primary healthcare provider should be notified. Encouraging the client to drink oral fluids is contraindicated; the client probably still will be under the influence of anesthesia, and the gag reflex may be depressed. Milking the client's nephrostomy tube is unnecessary because the output is adequate.

How long will a client's ovum stay viable after its release to get fertilized?
1
72 hours
2
78 hours
3
76 hours
4
74 hours

Ovum can be fertilized up to 72 hours after its release. The ovum disintegrates after 72 hours, and menstruation begins soon after. Therefore the ovum cannot be viable for 74, 76, or 78 hours, and fertilization will not occur.

Which hormone is released in response to low serum levels of calcium?
1
Atrial natriuretic peptide
2
Renin
3
Parathyroid hormone
4
Erythropoietin

3 If serum calcium levels decline, the parathyroid gland releases parathyroid hormone to maintain calcium homeostasis. Renin is a hormone released in response to decreased renal perfusion; this hormone is responsible for regulating blood pressure. Erythropoietin is released by the kidneys in response to poor blood flow to the kidneys; it stimulates the production of red blood cells. Atrial natriuretic peptide is produced by the right atrium of the heart in response to increased blood volume. This hormone then acts on the kidneys to promote sodium excretion, which decreases the blood volume.

Which test helps to identify fibroids, tumors, and fistulas while performing a reproductive tract examination?
1
Mammography
2
Computed tomography
3
Ultrasonography
4
Hysterosalpingography

4 A hysterosalpingogram is an X-ray used to evaluate tubal anatomy and patency and used to identify uterine problems such as fibroids, tumors, and fistulas. A mammography is an X-ray of the soft tissue of the breast. An ultrasonography (US) is a technique used to assess fibroids, cysts, and masses. Computer tomography is used to detect and evaluate masses and identify lymphatic enlargement from metastasis. Test-Taking Tip: Identifying content and what is being asked about that content is critical to your choosing the correct response. Be alert for words in the stem of the item that are the same or similar in nature to those in one or two of the options.

The nurse finds that a client with a urinary disorder has very pale-yellow-colored urine. What is the significance of this abnormal finding?
1
It indicates dilute urine.
2
It indicates concentrated urine.
3
It indicates the presence of myoglobin.
4
It indicates blood in the urine.

Dilute urine tends to appear very pale-yellow in color. Dark-red or brown color urine indicates the presence of blood in the urine. Dark-amber color urine indicates concentrated urine. Red color urine may indicate the presence of myoglobin.

What are the functions of antidiuretic hormone (ADH)? Select all that apply.
1
Promoting the reabsorption of sodium in the distal convoluted tubule (DCT)
2
Increasing tubular permeability to water
3
Increasing arteriole constriction
4
Controlling calcium balance
5
Stimulating the bone marrow to make red blood cells

2, 3 Antidiuretic hormone (ADH), also known as vasopressin, is a hormone released from the posterior pituitary gland. ADH increases arteriole constriction and tubular permeability to water. Calcium balance is controlled by blood levels of calcitonin and the parathyroid hormone (PTH). Erythropoietin stimulates the bone marrow to make red blood cells. Aldosterone promotes the reabsorption of sodium in the distal convoluted tubule (DCT).

The nurse is aware that the Cowper gland is also often referred to by which other term?
1
Skene gland
2
Prostate gland
3
Bartholin gland
4
Bulbourethral gland

4 Cowper glands are accessory glands of the male reproductive system; they are also referred to as the bulbourethral glands. Skene glands are a part of the female reproductive system. The prostate gland is also a gland of the male reproductive system. Bartholin glands are part of the female reproductive system.

The nurse is teaching a client receiving peritoneal dialysis about the reason dialysis solution is warmed before it is instilled into the peritoneal cavity. Which information will the nurse share with the client?
1
Because it adds extra warmth to the body because metabolic processes are disturbed
2
Because it helps prevent cardiac dysrhythmias by speeding up removal of excess potassium
3
Because it encourages removal of serum urea by preventing constriction of peritoneal blood vessels
4
Because it forces potassium back into the cells, thereby decreasing serum levels

3 Encouraging the removal of serum urea by preventing constriction of peritoneal blood vessels promotes vasodilation so that urea, a large-molecular substance, is shifted from the body into the dialyzing solution. Heat does not affect the shift of potassium into the cells. The removal of metabolic wastes is affected in kidney failure, not the metabolic processes. Heating dialysis solution does not affect cardiac dysrhythmias.

Which diagnostic procedure helps in the detection of uropathologic features in a client who has a urinary pouch or ileal conduit?
1
Loopogram
2
Cystogram
3
Computed tomography urogram
4
Urethrogram

1 Loopogram helps in the detection of uropathologic features in a client who has a urinary pouch or ileal conduit. Cystogram helps to visualize the bladder and evaluates vesicoureteral reflux. A computed tomography (CT) urogram provides excellent visualization of kidneys and kidney size can be evaluated. When urethral trauma is suspected, an urethrogram is done before catheterization.

What are the general manifestations associated with clients who have urinary system disorders? Select all that apply.
1
Nausea and vomiting
2
Facial edema
3
Excessive thirst
4
Elevated blood pressure
5
Stress incontinence

1,3,4 The general manifestations associated with urinary system disorders include excessive thirst, nausea and vomiting, and elevated blood pressure. The specific manifestations associated with urinary system disorders include facial edema and stress incontinence.

Which component of the client's nephron acts as a receptor site for the antidiuretic hormone and regulates water balance?
1
Proximal convoluted tubule
2
Distal convoluted tubule
3
Bowman's capsule
4
Collecting ducts

4 The collecting ducts regulate water balance and act as a receptor site for antidiuretic hormone. The Bowman's capsule collects glomerular filtrate and funnels it into the tubule. The distal convoluted tubule acts as a site for additional water and electrolyte reabsorption. The proximal convoluted tubule is the site for reabsorption of sodium, chloride, water, and urea.

Which urinary diagnostic test does not require any dietary or activity restrictions for the client before or after the test?
1
Renal scan
2
Concentration test
3
Renal arteriogram
4
Renal biopsy

1 A renal scan does not require any dietary or activity restrictions. A renal biopsy requires bed rest for 24 hours after the procedure. A renal arteriogram requires the client to maintain bed rest with affected leg straight. A concentration test requires the client to fast after a given time in the evening.

A client reports to a health clinic because a sexual partner recently was diagnosed as having gonorrhea. The health history reveals that the client has engaged in receptive anal intercourse. What should the nurse assess for in this client?
1
Anal itching
2
Ribbon-shaped stools
3
Melena
4
Constipation

1 Anal itching and irritation can occur from having anal intercourse with a person infected with gonorrhea. Frank rectal bleeding, not upper gastrointestinal bleeding (melena), occurs. Painful defecation, not constipation, occurs. The shape of formed stool does not change; however, defection can be painful.

Which is an abnormal finding of the urinary system?
1
Pain in the flank region upon hitting
2
Presence of bowel sounds
3
Nonpalpable urinary bladder
4
Nonpalpable left kidney

1 Normally, a blow in the flank region should not elicit pain. Pain in the flank region upon hitting indicates kidney infection or polycystic kidney disease. But the client experiences pain when his/her flank area is hit; therefore, this is an abnormal finding. The left kidney is covered by the spleen and is not palpable, which is a normal finding. The client has bowel sounds. However, no alteration of bowel sounds is seen. Therefore it is a normal finding. The urinary bladder is not normally palpable, unless it is distended with urine.

A nurse is caring for a client who had a nephrectomy because of cancer of the kidney. Which factor will influence the client's ability to deep breathe and cough postoperatively?
1
Pulmonary congestion from preoperative medications
2
Location of the surgical incision
3
Inflammatory process associated with surgery
4
Increased anxiety about the prognosis

2 The location of the surgical site in relation to the diaphragm increases incisional pain when deep breathing or coughing. Anxiety about the prognosis should not interfere with the ability to deep breathe and cough, especially when encouraged by the nurse. Inflammatory changes will cause discomfort in the area of any incision but are not necessarily the prime factor preventing deep breathing after a nephrectomy. The client will need to cough and deep breathe if there is congestion in the lungs.

A client scheduled for a hemicolectomy because of ulcerative colitis asks if having a hemicolectomy means wearing a pouch and having bowel movements in an abnormal way. Which is the best response by the nurse?
1
"Yes, but it will be temporary until the colitis is cured."
2
"No, that is necessary when a tumor is blocking the rectum."
3
"Yes, hemicolectomy is the same as a colostomy."
4
"No, only part of the colon is removed and the rest reattached."

4 Hemicolectomy is removal of part of the colon with an anastomosis between the ileum and transverse colon; a colostomy is not necessary. With a colostomy the intestine opens on the abdomen, whereas in a hemicolectomy a portion of the intestine is resected and the ends reconnected. "Yes, but it will be temporary until the colitis is cured" is the description of a temporary colostomy; a cure occurs only when the entire colon is removed. A colostomy is done for a variety of reasons other than a tumor; a colectomy with a colostomy is only one intervention that may be used to treat a tumor.

A nurse is caring for an older bedridden male client who is incontinent of urine. Which action should the nurse take first?
1
Offer the urinal regularly.
2
Insert an indwelling urinary catheter.
3
Apply incontinence pants.
4
Restrict fluid intake.

1 Offering the urinal is the first step. Retraining the bladder includes a routine pattern of attempts to void, which may increase bladder muscle tone and produce a conditioned response. Restricting fluid intake can result in dehydration and a urinary tract infection in an older client. Applying incontinence pants does not address the cause of the incontinence; also it promotes skin breakdown and can lower self-esteem. Inserting an indwelling urinary catheter increases the risk of a urinary tract infection. Also, it requires a primary healthcare provider's prescription.

The nurse is performing bedside sonography for a female client who underwent a hysterectomy. Which nursing intervention needs correction?
1
Pointing the scan head so the ultrasound is projected towards the client's coccyx
2
Placing the midline of the probe over the abdomen about 1.5 inches (3.8 cm) above the pubic bone
3
Placing an ultrasound gel pad right above the pubic bone
4
Using the female icon on the bladder scanner

4 Before performing a bedside sonography, the male or female icon on the scanner should be selected. The male icon should be selected for men and for women who have undergone a hysterectomy. An ultrasound gel pad should be placed right above the pubic bone. The scan head should be pointed in such a way that the ultrasound is projected towards the client's coccyx. The midline of the probe should be placed over the abdomen about 1.5 inches (3.8 cm) above the pubic bone.

A client with acute kidney injury is to receive peritoneal dialysis and asks why the procedure is necessary. Which is the nurse's best response?
1
"It speeds recovery because the kidneys are not responding to regulating hormones."
2
"It removes toxic chemicals from the body so you will not get worse."
3
"It prevents the development of serious heart problems."
4
"It helps perform some of the work usually done by the kidneys."

4 Dialysis removes chemicals, wastes, and fluids usually removed from the body by the kidneys. The mention of heart problems is a threatening response and may cause increased fear or anxiety. Stating that peritoneal dialysis "removes toxic chemicals from the body so you will not get worse" is threatening and can cause an increase in anxiety. Dialysis helps maintain fluid and electrolytes; the nephrons are damaged in acute kidney injury, so it may or may not speed recovery.

Which part of the reproductive system secretes androgens in female clients?
1
Ovarian follicle
2
Uterus
3
Ovaries
4
Fallopian tube

3 The ovaries and adrenal glands produce androgens in women. The fetus develops in the uterus during pregnancy. Fallopian tubes facilitate fertilization of oocyte and sperm. Ovarian follicle is a collection of oocytes in the ovary.

A male client reports dysuria, nocturia, and difficulty starting the urinary stream. A cystoscopy and biopsy of the prostate gland have been scheduled. After the procedure the client reports an inability to void. Which action should the nurse take?
1
Insert a urinary retention catheter.
2
Palpate above the pubic symphysis.
3
Assure the client that this is expected.
4
Limit oral fluids until the client voids.

2 A full bladder is palpable with urinary retention and distention, which are common problems after a cystoscopy because of urethral edema. More conservative nursing methods, such as running water or placing a warm cloth over the perineum, should be attempted to precipitate voiding; catheterization carries a risk of infection and is used as the last resort. Fluids dilute the urine and reduce the chance of infection after cystoscopy and should not be limited. Although urinary retention can occur, it is not expected; the nurse must assess the extent of bladder distention and discomfort.

A nurse is performing peritoneal dialysis for a client. Which action should the nurse take?
1
Withhold the routine medications until after the procedure.
2
Infuse the dialysate solution slowly over several hours.
3
Warm the dialysate solution slightly before instillation.
4
Place the client in a side-lying position.

3 The infusion should be warmed to body temperature to decrease abdominal discomfort and promote dilation of peritoneal vessels. The side-lying position may restrict fluid inflow and prevent maximum urea clearance; the client should be placed in the semi-Fowler position. The infusion of dialysate solution should take approximately 10 to 20 minutes. Routine medications should not interfere with the infusion of dialysate solution.

The nurse is reviewing the urinalysis reports of four clients with renal disorders.
Which client's finding signifies the presence of excessive bilirubin?
1
Client 1
2
Client 2
3
Client 3
4
Client 4

yellow brown to olive green Client 3's urinalysis reports findings of the presence of yellow-brown to olive-green-colored urine which signifies excessive bilirubin. Client 1's urinalysis report findings of the presence of amber-yellow-colored urine signifies a normal finding. Client 2's urinalysis report findings of the presence of dark, smoky-colored urine signifies hematuria. Client 4's urinalysis report findings of orange-red or orange-brown-colored urine indicates the presence of phenazopyridine in the urine.

Which condition should be reported immediately to the primary healthcare provider?
1
Rectal bleeding for 2 days after prostate biopsy
2
Pelvic pain immediately after colposcopy
3
Light vaginal bleeding for 1 to 2 days following a hysterosalpingogram
4
Body temperature of 102° F with vaginal discharge 48 hours after cervical biopsy

4 The client with cervical biopsy should immediately report to the primary healthcare provider if experiencing a body temperature of 102° F with vaginal discharge. This is because fever and vaginal discharge that develops 48 hours after cervical biopsy may be the signs of infection related to the procedure. The client should take pain relievers for pelvic pain after colposcopy. Light vaginal bleeding for 1 to 2 days following hysterosalpingogram is common. If the amount of bleeding increases or extends beyond 2 days, the healthcare provider should be notified. Light rectal bleeding for a few days is common after prostate biopsy.

To prevent bleeding after a suprapubic prostatectomy, the client should be instructed to avoid straining on defecation. Which foods should the nurse encourage the client to eat to help prevent constipation during the recovery period? Select all that apply.
1
Scrambled eggs
2
Green peas
3
Milk
4
Apples
5
Oatmeal

2, 4, 5 Apples, oatmeal, and green peas are high in fiber, which helps prevent constipation. Milk and milk products can be constipating; they do not contain bulk. Scrambled eggs contain little dietary fiber and do not prevent constipation.

The nurse is preparing a client who is on metformin therapy and is scheduled to undergo renal computed tomography with contrast dye. What does the nurse anticipate the primary healthcare provider to inform the client regarding the procedure?
1
"Discontinue metformin a half-day prior to procedure."
2
"Discontinue metformin 7 days following the procedure."
3
"Discontinue metformin 1 day prior to procedure."
4
"Discontinue metformin 3 days following the procedure."

3 Metformin can react with the iodinated contrast dye that is given for a renal computed tomography (CT) and cause lactic acidosis. Therefore the nurse anticipates an instruction that the client should discontinue the metformin 1 day before the procedure. Stopping the metformin a half-day before the renal CT may not reduce the risk of lactic acidosis. The client is advised to discontinue the metformin for at least 48 hours after the procedure. It is not necessary to discontinue metformin for 3 to 7 days after a renal CT with contrast media.

A client is diagnosed with calcium oxalate renal calculi. Which foods should the nurse teach the client to avoid? Select all that apply.
1
Liver
2
Spinach
3
Rhubarb
4
Milk
5
Tea

2, 3, 5 Tea, rhubarb, and spinach are high in calcium oxalate. Limiting oxalate-rich foods limits oxalate absorption and the formation of calcium oxalate calculi. Milk is an acceptable calcium-rich protein and is avoided in calcium stones but not with oxalate stones. Liver is a purine-rich food that may be eaten. Test-Taking Tip: When using this program, be sure to note if you guess at an answer. This will permit you to identify areas that need further review. Also it will help you to see how correct your guessing can be.

A client has glomerulonephritis. To prevent future attacks of glomerulonephritis, the nurse planning discharge teaching includes which instruction?
1
"Avoid situations that involve physical activity."
2
"Restrict fluid intake."
3
"Seek early treatment for respiratory infections."
4
"Take showers instead of bubble baths."

3 A common cause of glomerulonephritis is a streptococcal infection. This infection initiates an antibody formation that damages the glomeruli. Any fluid restriction is moderated as the client improves; fluid is allowed to prevent urinary stasis. The alkalinity of bubble baths is linked to urinary tract infections, not glomerulonephritis. Moderate activity is helpful in preventing urinary stasis, which can precipitate urinary infection.

A client who has been on hemodialysis for several weeks asks the nurse what substances are being removed by the dialysis. Which substance removal should the nurse share with the client?
1
Sodium
2
Bacteria
3
Glucose
4
Blood

1 Sodium is an electrolyte that passes through the semipermeable membrane during hemodialysis. Red blood cells do not pass through the semipermeable membrane during hemodialysis. Glucose does not pass through the semipermeable membrane during hemodialysis. Bacteria do not pass through the semipermeable membrane during hemodialysis.

After a transurethral prostatectomy, a client returns to the postanesthesia care unit with a three-way indwelling catheter with continuous bladder irrigation. Which nursing action is the priority?
1
Observing the suprapubic dressing for drainage
2
Maintaining the client in the semi-Fowler position
3
Monitoring for bright red blood in the drainage bag
4
Encouraging fluids by mouth as soon as the gag reflex returns

3 Blood clots are normal 24 to 36 hours after surgery, but bright red blood can indicate hemorrhage. The surgery is performed through the urinary meatus and urethra; there is no suprapubic incision. It is unnecessary to keep the client in the semi-Fowler position. The client is initially allowed nothing by mouth and then advanced to a regular diet as tolerated. Continuous irrigation supplies enough fluid to flush the bladder.

A client with a history of chronic kidney disease is hospitalized. Which assessment findings will alert the nurse to kidney insufficiency?
1
Edema and pruritus
2
Facial flushing
3
Diminished force and caliber of stream
4
Dribbling after voiding and dysuria

1 The accumulation of metabolic wastes in the blood (uremia) can cause pruritus; edema results from fluid overload caused by impaired urine production. Pallor, not flushing, occurs with chronic kidney disease as a result of anemia. Dribbling after voiding is a urinary pattern that is not caused by chronic kidney disease; this may occur with prostate problems. Diminished force and caliber of stream occur with an enlarged prostate, not kidney disease.

The registered nurse is preparing to assess a client's renal system. Which statement by the nurse indicates effective technique?
1
"I must first palpate the client if a tumor is suspected."
2
"I must first auscultate the client and then proceed to percussion and palpation."
3
"I must first listen for normal pulse at the client's wrist region."
4
"I must first examine tender abdominal areas and then proceed to nontender areas."

2 Palpation and percussion can cause an increase in normal bowel sounds and hide abdominal vascular sounds. Therefore it is wise to perform auscultation prior to percussion and palpation during clinical assessment of the renal system. Palpation should be avoided if a client is suspected of having a tumor because it could harm the client. It is more important as part of clinical assessment of the renal system to listen for bruit by auscultating over the renal artery. Bruit indicates renal artery stenosis. The nontender areas should be examined prior to tender areas to avoid confusion regarding radiating pain from the tender area being percussed.

A nurse is assessing a client with a diagnosis of kidney failure for clinical indicators of metabolic acidosis. What should the nurse conclude is the reason metabolic acidosis develops with kidney failure?
1
Inability of the renal tubules to reabsorb water to dilute the acid contents of blood
2
Impaired glomerular filtration, causing retention of sodium and metabolic waste products
3
Depressed respiratory rate due to metabolic wastes, causing carbon dioxide retention
4
Inability of the renal tubules to secrete hydrogen ions and conserve bicarbonate

4 Bicarbonate buffering is limited, hydrogen ions accumulate, and acidosis results. The rate of respirations increases in metabolic acidosis to compensate for a low pH. The fluid balance does not significantly alter the pH. The retention of sodium ions is related to fluid retention and edema rather than to acidosis.

A nurse is caring for a client who is experiencing urinary incontinence. The client has an involuntary loss of small amounts (25 to 35 mL) of urine from an overdistended bladder. How should this be documented in the client's medical record?
1
Overflow incontinence
2
Stress incontinence
3
Functional incontinence
4
Urge incontinence

1 Overflow incontinence describes what is happening with this client; overflow incontinence occurs when the pressure in the bladder overcomes sphincter control. Urge incontinence describes a strong need to void that leads to involuntary urination regardless of the amount in the bladder. Stress incontinence occurs when a small amount of urine is expelled because of an increase in intraabdominal pressure that occurs with coughing, lifting, or sneezing. Functional incontinence occurs from other issues rather than the bladder, such as cognitive (dementia) or environmental (no toileting facilities). Test-Taking Tip: Do not panic while taking an exam! Panic will only increase your anxiety. Stop for a moment, close your eyes, take a few deep breaths, and resume review of the question.

Which instruction would be most beneficial for an aging African-American client with hypertension?
1
"Record blood pressure weekly."
2
"Check the pulse daily."
3
"Visit an ophthalmologist monthly."
4
"Have an annual urinalysis."

4 African-American clients have 20% less blood flow to the kidneys because of high sodium consumption. This causes anatomical changes in the blood vessels, thereby increasing the risk of kidney failure. Therefore instructing the client with hypertension to have an annual urine examination would be beneficial. If the client has protein in the urine, this is a sign of high blood pressure and can signify kidney damage. Checking the pulse daily poses no harm to the individual, but does not determine if the client has hypertension. Recording the blood pressure weekly is not a good indicator of an aging African-American client with hypertension. The client's blood pressure should be taken at least daily to determine if the client has problems. If the client has an eye-related problem, visiting an ophthalmologist should be suggested.

The nurse is assisting the primary healthcare provider during a renal ultrasonography. Arrange the steps involved in the procedure in correct sequence.
Incorrect
2
Apply gel over skin
Incorrect
4
Wipe cotton pad over gel
Correct
3.
Move transducer across skin
Incorrect
1
Place client in prone position

The client undergoing renal ultrasonography should first be placed in the prone position. Then the sonographic gel should be applied on the client's skin over the back and flank regions. Then the transducer is moved across the client's skin to measure the echoes. The images are visualized on the display screen. At the end of the procedure the gel is removed from the client's skin by using a piece of wet cotton or cloth over the gel.

A client has a permanent colostomy. During the first 24 hours there is no drainage from the colostomy. How should the nurse interpret this finding?
1
Effective functioning of the nasogastric tube is causing this.
2
Absence of intestinal peristalsis is causing this.
3
Edema after the surgery is causing this.
4
Decrease in fluid intake before surgery is causing this.

2 Absence of peristalsis is caused by manipulation of abdominal contents and the depressant effects of anesthetics and analgesics. Edema will not interfere with peristalsis; edema may cause peristalsis to be less effective, but some output will result. An absence of fiber has a greater effect on decreasing peristalsis than does decreasing fluids. A nasogastric tube decompresses the stomach; it does not cause cessation of peristalsis.

A client scheduled for a transurethral prostatectomy expresses concern about the effect the surgery will have on sexual ability. Which information should the nurse share with the client?
1
Will have prolonged erections
2
Will be impotent
3
May have a diminished sex drive
4
May experience retrograde ejaculations

4 Ejection of semen into the bladder instead of the urethra is common after a transurethral prostatectomy. The surgery should not interfere with the libido and will not cause prolonged erections. Impotence is not typical with this approach; it may occur with the retroperitoneal approach.

After reviewing the urinalysis reports of a client with kidney dysfunction, the nurse suspects the presence of myoglobin. Which finding in the test reports supports the nurse's suspicion?
1
Very pale yellow colored urine
2
Dark amber colored urine
3
Red-colored urine
4
Brown-colored urine

3 Red-colored urine in clients with kidney dysfunction indicates the presence of myoglobin. Brown-colored urine indicates increased bilirubin levels. Dark amber urine indicates concentrated urine. Very pale yellow urine indicates dilute urine.

A nurse is caring for a client with a ureteral calculus. Which are the most important nursing actions? Select all that apply.
1
Limiting fluid intake at night
2
Administering the prescribed analgesic
3
Monitoring intake and output
4
Recording the client's blood pressure
5
Straining the urine at each voiding

2, 3, 5 A urinary calculus may obstruct urine flow, which will be reflected in a decreased output; obstruction may result in hydronephrosis . Urine is strained to determine whether any calculi or calcium gravel is passed. Reduction of pain is a priority. A calculus obstructing a ureter causes flank pain that extends toward the abdomen, scrotum and testes, or vulva; the pain begins suddenly and is severe (renal colic). Fluids should be encouraged to promote dilute urine and facilitate passage of the calculi. Recording the blood pressure is not critical.

Which vascular component of the client's nephron delivers arterial blood from the glomerulus into the peritubular capillaries or the vasa recta?
1
Afferent arteriole
2
Interlobular artery
3
Efferent arteriole
4
Arcuate artery

3 The efferent arteriole is the vascular component of the nephron that delivers arterial blood from the glomerulus into the peritubular capillaries or the vasa recta. The arcuate artery is a curved artery of the renal system that surrounds the renal pyramids. The afferent arteriole is the vascular component of the nephron that delivers arterial blood from the branches of the renal artery into the glomerulus. The interlobular artery feeds the lobes of the kidney.

The nurse is preparing a blood transfusion for a client with renal failure. Why does anemia often complicate renal failure?
1
Increase in blood pressure
2
Decrease in serum sodium concentration
3
Decrease in erythropoietin
4
Increase in serum phosphate levels

3 The hormone erythropoietin, produced by the kidneys, stimulates the bone marrow to produce red blood cells. In renal failure there is a deficiency of erythropoietin that often results in the client developing anemia. Therefore the nurse is instructed to administer blood. In renal failure, increased blood pressure is due to impairment of renal vasodilator factors and is not treated by administration of blood. Phosphate is retained in the body during renal failure, causing binding of calcium leading to done demineralization, not anemia. Increase in urinary sodium concentration and decrease in serum sodium concentration trigger the release of renin from the juxtaglomerular cells.

The nurse is providing education to a client with calculi in the calyces of the right kidney. The client is scheduled to have the calculi removed. Which information should the nurse include in the teaching?
1
The surgery will be performed transurethrally.
2
After surgery, a suprapubic catheter will be in place.
3
During the surgery, the right ureter will be removed.
4
After surgery, there will be a small incision in the right flank area.

4 If the calculi are in the renal pelvis, a percutaneous pyelolithotomy is performed; the calculi are removed via a small flank incision. Removal of the right ureter is not necessary. Performing surgery transurethrally is used for calculi in the ureters and renal pelvis. Placement of a suprapubic catheter usually is unnecessary.

What age-related changes are associated with the female genitalia? Select all that apply.
1
Firm breasts
2
Erected nipples
3
Graying of pubic hair
4
Dry vagina
5
Decreased size of the labia majora

3,4,5 An elderly female client may have dry, smooth, and thin vaginal walls due to atrophy of the vaginal tissue and secretory glands on the vaginal walls. Graying and thinning pubic hair and decreased size of the labia majora and clitoris are also normal signs of aging. The client may have increased flabbiness and fibrosis of the breasts, which hang lower on the chest wall, along with decreased erection of the nipples.

The nurse prepares a client for a Papanicolaou test (Pap test). What should the nurse instruct the client before conducting the test?
1
Douche the vagina with soap
2
Empty the bladder
3
Avoid sexual intercourse for at least 24 hours before the test
4
Avoid scheduling a Pap test to be performed during menses

3 The Papanicolaou test is a cytological study used to detect precancerous and cancerous cells within the cervix. The nurse should advise the client to avoid sexual intercourse at least 24 hours before the test to prevent test misinterpretations. A client undergoing a pelvic examination should empty her bladder immediately before the test. Douching the vagina with soap or applying deodorants may lead to false test results. The Papanicolaou test should be scheduled between the client's menstrual periods so that the menstrual flow does not interfere with laboratory analysis.

A client with a renal disorder is scheduled for an intravenous pyelogram (IVP). Which interventions should the nurse undertake prior to the procedure? Select all that apply.
1
Instruct the client to lie still during the procedure
2
Have the client remove all metal objects
3
Ensure that the consent form is signed
4
Administer an enema or cathartic to the client
5
Assess the client for iodine sensitivity

3,4,5 The presence, position, shape, and size of kidneys, ureters, and bladder can be evaluated using an intravenous pyelogram (IVP). The contrast medium used in the procedure may cause hypersensitivity reactions. Therefore, the nurse should assess the client for sensitivity to iodine prior to the procedure. The nurse should use a cathartic or enema to empty the colon of feces and gas. An IVP does need a consent form since the procedure is invasive. The nurse has the client remove all metal objects before performing a magnetic resonance imaging (MRI) procedure. The nurse instructs the client to lie still during a computed tomographic (CT) scan procedure; during an IVP the client may be asked to turn certain ways.

The nurse is reviewing blood screening tests of the immune system of a client with acquired immunodeficiency syndrome (AIDS). What does the nurse expect to find?
1
A decrease in the serum level of glucose-6-phosphate dehydrogenase
Incorrect2
An increase in immunoglobulin E
3
An increase in thymic hormones
Correct4
A decrease in CD4 T cells
The human immunodeficiency virus (HIV) infects helper T-cell lymphocytes; therefore 300 or fewer CD4 T cells per cubic millimeter of blood or CD4 cells accounting for less than 20% of lymphocytes is suggestive of AIDS. The thymic hormones necessary for T-cell growth are decreased. An increase in immunoglobulin E is associated with allergies and parasitic infections. A decrease in the serum level of glucose-6-phosphate dehydrogenase is associated with drug-induced hemolytic anemia and hemolytic disease of the newborn.

Test-Taking Tip: As you answer each question, write a few words about why you think that answer is correct; in other words, justify why you selected that answer. If an answer you provide is a guess, mark the question to identify it. This will permit you to recognize areas that need further review. It will also help you to see how correct your "guessing" can be. Remember: on the licensure examination you must answer each question before moving on to the next question.
Topics

Client Needs - Reduction of Risk Potential
Mastery Level 0
1 2 3

...

Why is a Neisseria gonorrhoeae infection particularly troublesome for a female client?
Correct1
Symptoms are often overlooked.
2
Treatment has many adverse effects.
3
It is difficult to treat with antibiotics.
4
The medication is expensive.
Many female clients who contract gonorrhea are asymptomatic or have minor symptoms that are often overlooked, making it possible for them to remain a source of infection. There is no evidence to support that the medication to treat the infection is expensive. The infection can be treated with one intramuscular injection of ceftriaxone. There is no evidence to support that the medication to treat this infection has many adverse effects.

...

Which type of cytokine is used to treat anemia related to chronic kidney disease?
Correct1
Erythropoietin
2
α-Interferon
3
Interleukin-2
4
Interleukin-11
Erythropoietin is used to treat anemia related to chronic kidney disease. α-Interferon is used to treat hairy cell leukemia or malignant melanoma. Interleukin-2 is used to treat metastatic renal carcinoma. Interleukin-11 is used to prevent thrombocytopenia after chemotherapy.

...

Which cytokine is used to treat multiple sclerosis?
1
Erythropoietin
2
Colony-stimulating factor
3
Interleukin-2
Correct4
β-Interferon
β-Interferon is a cytokine used to treat multiple sclerosis. Interleukin-2 is used to treat metastatic melanoma. Erythropoietin is a cytokine used to treat anemia related to chemotherapy. Colony-stimulating factor is a cytokine used to treat chemotherapy-induced neutropenia.

...

Which sexually transmitted disease is caused by the human papilloma virus?
1
Genital herpes
2
Chlamydia infection
Correct3
Condylomata acuminata
4
Gonorrhea
Condylomata acuminata is a sexually transmitted disease caused by the human papilloma virus. Gonorrhea is caused by Neisseria gonorrhoeae. Genital herpes is causes by the herpes simplex virus. Chlamydia infection is caused by Chlamydia trachomatis.

Test-Taking Tip: Identify option components as correct or incorrect. This may help you identify a wrong answer.

...

The echoviruses can cause which diseases in clients? Select all that apply.
1
Burkitt's lymphoma
Incorrect 2
Mononucleosis
Correct 3
Gastroenteritis
4
Parotitis
Correct 5
Aseptic meningitis
Echoviruses cause gastroenteritis and aseptic meningitis. Parotitis is caused by mumps. Burkitt's lymphoma and mononucleosis are caused by the Epstein-Barr virus.

...

Which parameter should the nurse consider while assessing the psychologic status of a client with acquired immune deficiency syndrome (AIDS)?
Correct1
Presence of anxiety
2
Sleep pattern
3
Severity of pain
4
Cognitive changes
Presence of anxiety should be considered while assessing the psychologic status of a client with AIDS. Sleep patterns and severity of pain are related to the assessment of activity and rest, a physical status. Cognitive changes are related to the assessment of neurologic status.

...

A client with localized redness and swelling due to a bee sting reports intense local pain, a burning sensation, and itching. What would be the most appropriate nursing action?
1
Monitoring for neurological and cardiac symptoms
2
Advising the client to launder all clothes with bleach
3
Ensuring the client keeps the skin clean and dry
Correct4
Applying cold compresses to the affected area
A client with a bee sting may have localized redness, swelling, pain, and itching due to an allergic reaction. The nurse should apply cold compresses to the affected area to reduce the pain in the client. A client with Candida albicans infection should keep his or her skin clean and dry to prevent further fungal infections. A client with a Borrelia burgdorferi infection may suffer from cardiac, arthritic, and neurologic manifestations. Therefore the nurse has to monitor for these symptoms. Direct contact may transmit a Sarcoptes scabiei infection; the nurse should make sure that the client's clothes are bleached to prevent the transmission of the infection.

Test-Taking Tip: Read the question carefully before looking at the answers: (1) Determine what the question is really asking; look for key words; (2) Read each answer thoroughly and see if it completely covers the material asked by the question; (3) Narrow the choices by immediately eliminating answers you know are incorrect.

...

A client scheduled for surgery has a history of methicillin-resistant Staphylococcus aureus (MRSA) since developing an infection in a surgical site 9 months ago. The site is healed, and the client reports having received antibiotics for the infection. What should the nurse do to determine if the infecting organism is still present?
1
Call the surgeon for an infectious disease consultation.
Incorrect2
Inform the operating room of the MRSA.
3
Notify the infection control officer.
Correct4
Obtain an order to culture the client's blood.
Obtaining cultures is the most reliable method of determining the presence of an infecting microorganism. Although notifying the infection control officer should be done, the presence of an infecting microorganism should be identified first. Informing the operating room personnel of the MRSA is usual when an infecting microorganism is present; however, it is not yet confirmed that an infecting microorganism is present. Although calling the surgeon for an infectious disease consultation may be done, the presence of an infecting agent should be identified first.

Test-Taking Tip: Pace yourself while taking a quiz or exam. Read the entire question and all answer choices before answering the question. Do not assume that you know what the question is asking without reading it entirely.

...

Which is the first medication approved to reduce the risk of human immunodeficiency virus (HIV) infection in unaffected individuals?
1
Cromolyn
2
Methdilazine
Incorrect3
Abacavir
Correct4
Truvada
Truvada is the first medication approved to reduce the risk of HIV infection in unaffected individuals who are at a high risk of HIV infection. Abacavir is administered to treat HIV infection and is a reverse transcriptase inhibitor. Cromolyn is administered in the management of allergic

...

Which sexually transmitted disease is caused by a virus?
1
Syphilis
2
Chlamydial infection
Correct3
Genital warts
4
Gonorrhea
Genital warts are caused by a sexually transmitted virus. Bacteria cause syphilis, gonorrhea, and chlamydial infections.

...

Which is the first antibody formed after exposure to an antigen?
1
IgG
2
IgA
Correct3
IgM
4
IgE
IgM (immunoglobulin M) is the first antibody formed by a newly sensitized B-lymphocyte plasma cell. IgA has very low circulating levels and is responsible for preventing infection in the upper and lower respiratory tracts, and the gastrointestinal and genitourinary tracts. IgE has variable concentrations in the blood and is associated with antibody-mediated hypersensitivity reactions. IgG is heavily expressed on second and subsequent exposures to antigens to provide sustained, long-term immunity against invading microorganisms.

...

What are the mediators of injury in IgE-mediated hypersensitivity reactions? Select all that apply.
1
Neutrophils
Correct 2
Leukotrienes
Correct 3
Mast cells
Correct 4
Histamines
5
Cytokines
Mast cells, histamines, and leukotrienes are the mediators of injury in IgE-mediated hypersensitivity reactions. Cytokines are the mediators of injury in the delayed type of hypersensitivity reaction. Neutrophils are the mediators of injury in the immune-complex type of hypersensitivity reaction.

...

Which immunoglobulin crosses the placenta?
1
IgA
2
IgE
3
IgM
Correct4
IgG
IgG is the only immunoglobulin that crosses the placenta. IgE is found in the plasma and interstitial fluids. IgA lines the mucous membranes and protects body surfaces. IgM is found in plasma; this immunoglobulin activates due to the invasion of ABO blood antigens.

...

Which type of hypersensitivity reaction is associated with rheumatoid arthritis?
Incorrect1
Cytotoxic
2
Delayed
3
IgE-mediated
Correct4
Immune-complex
Rheumatoid arthritis is an autoimmune disorder associated with an immune-complex type of hypersensitivity reaction. Contact dermatitis caused by poison ivy is associated with a delayed type of hypersensitivity reaction. Goodpasture's syndrome is associated with a cytotoxic type of hypersensitivity reaction. Asthma is associated with an IgE-mediated type of hypersensitivity reaction.

...

Which disease is caused by Coronaviruses?
Correct1
Severe acute respiratory syndrome
2
Inhalation anthrax
3
Coccidioidomycosis
4
Pertussis
Severe acute respiratory syndrome is a respiratory infection caused by Coronaviruses. Pertussis is caused by the bacterium Bordetella pertussis. Inhalation anthrax is caused by Bacillus anthracis. Coccidioidomycosis is caused by Coccidioides.

...

Which disease is caused by Escherichia coli?
Incorrect1
Food poisoning
2
Diphtheria
Correct3
Peritonitis
4
Tetanus
Escherichia coli causes peritonitis. Clostridium tetani causes tetanus. Corynebacterium diphtheria causes diphtheria. Clostridium botulinum causes food poisoning.

...

A mother with the diagnosis of acquired immunodeficiency syndrome (AIDS) states that she has been caring for her baby even though she has not been feeling well. What important information should the nurse determine?
1
When the baby last received antibiotics
2
How long she has been caring for the baby
Correct3
If the baby is breast-feeding
4
If she has kissed the baby
Epidemiologic evidence has identified breast milk as a source of human immunodeficiency virus (HIV) transmission. Kissing is not believed to transmit HIV. When the baby last received antibiotics is unrelated to transmission of HIV. HIV transmission does not occur from contact associated with caring for a newborn.

...

What is the mechanism of action of penicillin?
Correct1
Inhibits cell wall synthesis of the pathogen
2
Prevents reproduction of the pathogen
3
Injures the cytoplasmic membrane of the pathogen
4
Inhibits nucleic acid synthesis of the pathogen
Penicillin is an antimicrobial medication that inhibits cell wall synthesis of the susceptible pathogen. Gentamicin is an antimicrobial medication that prevents the reproduction of the susceptible pathogen. Actinomycin is an antimicrobial medication that inhibits nucleic acid synthesis of the susceptible pathogen. Antifungal agents injure the cytoplasmic membrane of the susceptible pathogen.

...

Which drug treats hay fever by preventing leukotriene synthesis?
Incorrect1
Chlorpheniramine
2
Diphenhydramine
3
Cromolyn sodium
Correct4
Zileuton
Zileuton is a leukotriene antagonist drug; this substance prevents the synthesis of leukotrienes and helps in managing and preventing hay fever. Cromolyn sodium stabilizes mast cells and prevents the opening of mast cell membranes in response to allergens binding to immunoglobulin E.. Chlorpheniramine and diphenhydramine are antihistamines and prevent the binding of histamine to receptor cells and decrease allergic manifestations.

...

Which sexually transmitted infection (STI) is caused by Treponema pallidum?
1
Gonorrhea
2
Vulvovaginitis
3
Genital warts
Correct4
Syphilis
Syphilis is an STI caused by Treponema pallidum. Neisseria gonorrhoeae causes gonorrhea. Haemophilus ducreyi and Klebsiella granulomatis cause genital warts. Herpes simplex virus, Trichomonas vaginalis, and Candida albicans may cause vulvovaginitis.

...

Which drug treats hay fever by preventing leukotriene synthesis?
Incorrect1
Chlorpheniramine
2
Diphenhydramine
3
Cromolyn sodium
Correct4
Zileuton
Zileuton is a leukotriene antagonist drug; this substance prevents the synthesis of leukotrienes and helps in managing and preventing hay fever. Cromolyn sodium stabilizes mast cells and prevents the opening of mast cell membranes in response to allergens binding to immunoglobulin E.. Chlorpheniramine and diphenhydramine are antihistamines and prevent the binding of histamine to receptor cells and decrease allergic manifestations.

...

Which client organ is protected by microglial cells?
Correct1
Brain
2
Kidney
3
Lung
4
Liver
Microglial cells are macrophages present in the brain. The lungs are protected by alveolar macrophages. The liver is protected by Kupffer cells. Mesangial cells are present in the kidneys.

...

A nurse is teaching a health class about human immunodeficiency virus (HIV). Which basic methods are used to reduce the incidence of HIV transmission? Select all that apply.
1
Using separate toilets
2
Sterilizing the household utensils
Correct 3
Practicing sexual abstinence
4
Preventing direct casual contacts
Correct 5
Using condoms
HIV is found in body fluids such as blood, semen, vaginal secretions, breast milk, amniotic fluid, urine, feces, saliva, tears, and cerebrospinal fluid. Therefore a client should use condoms to prevent contact between the vaginal mucus membranes and semen. Practicing sexual abstinence is the best method to prevent transmission of the virus. The HIV virus is not transmitted by sharing the same toilet facilities, casual contacts such as shaking hands and kissing, or by sharing the same household utensils.

...

Which immunomodulatory is beneficial for the treatment of clients with multiple sclerosis?
1
Interleukin 2
2
Interleukin 11
3
Alpha interferon
Correct4
Beta interferon
Beta interferon is an immunomodulator that is administered in the treatment of multiple sclerosis. Interleukin 11 is used in the prevention of thrombocytopenia after chemotherapy. Interleukin 2 is used for the treatment of metastatic renal cell carcinoma and metastatic melanoma. Alpha interferon is administered for the treatment of hairy cell leukemia, chronic myelogenous leukemia, and malignant melanoma.

...

Which cytokine stimulates the liver to produce fibrinogen and protein C?
Correct1
Interleukin-6
Incorrect2
Interleukin-1
3
Tumor necrosis factor
4
Thrombopoietin
Interleukin-6 stimulates the liver to produce fibrinogen and protein C. Interleukin-1 stimulates the production of prostaglandins. Thrombopoietin increases the growth and differentiation of platelets. Tumor necrosis factor stimulates delayed hypersensitivity reactions and allergies.

Test-Taking Tip: Identify option components as correct or incorrect. This may help you identify a wrong answer.

...

Which cytokine medication is administered to treat chemotherapy-induced neutropenia?
1
Aldesleukin
2
Oprelvekin
3
Darbepoetin alfa
Correct4
Filgrastim
Colony-stimulating factors such as filgrastim are administered to treat chemotherapy-induced neutropenia. Oprelvekin is used to prevent thrombocytopenia. Aldesleukin is used to treat metastatic renal cell carcinoma. Darbepoetin alfa is administered to treat anemia related to chronic cancer and anemia related to chronic kidney disease.

...

A client's sputum smears for acid-fast bacilli (AFB) are positive, and transmission-based airborne precautions are prescribed. What should the nurse teach visitors to do?
Correct1
Wear a particulate respirator mask.
2
Limit contact with the client's nonexposed family members.
Incorrect3
Put on a gown and gloves.
4
Avoid touching objects in the client's room.
Tubercle bacilli are transmitted through air currents; therefore personal protective equipment, such as a particulate respirator that filters out organisms as small as 1 µm, is necessary. Gowns and gloves are not necessary. Tuberculosis is spread by airborne microorganisms; gloves are necessary only when touching articles contaminated with respiratory secretions. It is only necessary to avoid contact with objects in the client's room that are contaminated with respiratory secretions. Limiting contact with the client's nonexposed family members is unnecessary.

...

A client's laboratory report reveals a CD4+ T-cell count of 520 cells/mm3. According to the Centers for Disease Control and Prevention (CDC), which stage of human immunodeficiency virus (HIV) disease is present in the client?
Incorrect1
Stage 3
2
Stage 4
Correct3
Stage 1
4
Stage 2
According to the CDC, HIV disease is divided into four stages. A client with a CD4+ T-cell count of greater than 500 cells/mm3 is in the first stage of HIV disease. A client with a CD4+ T-cell count between 200 and 499 cells/mm3 is in the second stage of HIV disease. A client with a CD4+ T-cell count of less than 200 cells/mm3 is in the third stage of HIV disease. The fourth stage of HIV disease indicates a confirmed HIV infection with no information regarding the CD4+ T-cell counts.

...

What are the clinical manifestations of inhalation anthrax? Select all that apply.
Correct 1
Fatigue
Correct 2
Fever
Correct 3
Dry cough
4
Sore throat
5
Rhinitis
Inhalation anthrax is a bacterial infection caused by Bacillus anthracis. Clinical manifestations include fever, fatigue, and a dry cough. Rhinitis and a sore throat are upper respiratory manifestations not associated with inhalation anthrax.

...

Which is a leukotriene antagonist used to manage and prevent allergic rhinitis?
Correct1
Zileuton
2
Ephedrine
Incorrect3
Cromolyn sodium
4
Scopolamine
Zileuton is a leukotriene antagonist used to manage and prevent allergic rhinitis. Ephedrine is an ingredient in decongestants used to treat allergic rhinitis. Scopolamine is an anticholinergic used to reduce secretions. Cromolyn sodium is a mast cell stabilizing drug used to prevent mast cell membranes from opening when an allergen binds to IgE.

Test-Taking Tip: You have at least a 25% chance of selecting the correct response in multiple-choice items. If you are uncertain about a question, eliminate the choices that you believe are wrong and then call on your knowledge, skills, and abilities to choose from the remaining responses.

...

Which age-related effects on the immune system are seen in the older client?
1
Increased expression of IL-2 receptors
2
Increased delayed hypersensitivity reaction
Correct3
Increased autoantibodies
4
Increased primary and secondary antibody responses
The effects of aging on the immune system include increased autoantibodies. Expression of IL-2 receptors, delayed hypersensitivity reaction, and primary and secondary antibody responses decrease in older adults because of the effects of aging on the immune system.

...

A child is diagnosed with hepatitis A. The client's parent expresses concern that the other members of the family may get hepatitis because they all share the same bathroom. What is the nurse's best reply?
1
"I suggest that you buy a commode exclusively for your child's use."
Correct2
"All family members, including your child, need to wash their hands after using the bathroom."
3
"Your child may use the bathroom, but you need to use disposable toilet covers."
4
"You will need to clean the bathroom from top to bottom every time a family member uses it."
Hepatitis A is spread via the fecal-oral route; transmission is prevented by proper hand washing. Buying a commode exclusively for the child's use is unnecessary; cleansing the toilet and washing the hands should control the transmission of microorganisms. It is not feasible to clean "from top to bottom" each time the bathroom is used. The use of disposable toilet covers is inadequate to prevent the spread of microorganisms if the bathroom used by the child also is used by others. Hand washing by all family members must be part of the plan to prevent the spread of hepatitis to other family members.
Topics

...

Which organism is responsible for causing Lyme disease in clients?
Correct1
Borrelia burgdorferi
2
Sarcoptes scabiei
3
Pediculushumanus var. corporis
4
Phthirus pubis
Lyme disease is a bacterial infection caused by Borrelia burgdorferi, which is transmitted by ticks. Phthirus pubis causes pediculosis. Scabies is caused by Sarcoptes scabiei. Pediculushumanus var. corporis also causes pediculosis.

...

Which type of hypersensitivity reaction will occur when the client's T cytotoxic cells are involved as the mediators of injury?
Correct1
Type IV
2
Type I
Incorrect3
Type II
4
Type III
Type IV hypersensitivity reaction will occur when the T cytotoxic cells are involved as the mediators of injury. Type I IgE-mediated reaction will occur when histamine is involved as the mediators of injury. Type II cytotoxic reaction will occur when complement lysis is the mediator of injury. Type III immune complex reaction will occur when neutrophils are involved as the mediators of injury.

...

What finding in the client is a sign of allergic rhinitis?
Correct1
Reduced transillumination on the skin over the sinuses
2
Presence of pinkish nasal discharge
3
Presence of high-grade fever
Incorrect4
Reduced breathing through the mouth
Reduced transillumination on the skin overlying the sinuses indicates allergic rhinitis. This effect is caused by the sinuses becoming inflamed and blocked with thick mucoid secretions. Generally, fever does not accompany allergic rhinitis unless the client develops a secondary infection. In allergic rhinitis, the client is unable to breathe through the nose because it gets stuffy and blocked. Instead the client will resort to mouth breathing. Clients with allergic rhinitis will have clear or white nasal discharge.

...

What causes medications used to treat AIDS to become ineffective?
1
Taking medications from different classifications
2
Taking the medications 90% of the time
Correct3
Missing doses of the prescribed medications
Incorrect4
Developing immune reconstitution inflammatory syndrome (IRIS)
The most important reason for the development of drug resistance in the treatment of AIDS is missing doses of drugs. When doses are missed, the blood drug concentrations become lower than what is needed to inhibit viral replication. The virus replicates and produces new particles that are resistant to the drugs. Taking the medications 90% of the time prevents medications from becoming ineffective. Taking medications from different classes prevents the drugs from becoming ineffective. Immune reconstitution inflammatory syndrome (IRIS) occurs when T-cells rebound with medication therapy and become aware of opportunistic infections.

...

Which diseases may occur due to rickettsial infections? Select all that apply.
1
West Nile fever
2
Leprosy
Correct 3
Rocky Mountain spotted fever
4
Lyme disease
Correct 5
Typhoid fever
Typhoid fever and Rocky Mountain spotted fever are caused by rickettsial infections. Spirochetes and Mycobacterium leprae cause leprosy. Borrelia burgdorferi cause Lyme disease. The West Nile virus causes West Nile fever.

...

What should be used to clean needles and syringes between intravenous drug users (IDUs)?
1
Hot water
Correct2
Bleach
3
Ammonia
Incorrect4
Rubbing alcohol
Intravenous drug users (IDUs) should be instructed to fill syringes with household bleach and shake the syringe for 30 to 60 seconds. Hot water, ammonia, or rubbing alcohol are not used to disinfect used syringes.

...

What are the clinical manifestations during the fulminant stage in a client with inhalation anthrax? Select all that apply.
Correct 1
Pleural effusion
2
Harsh cough
3
Mild chest pain
Correct 4
Body temperature of 104 °F
Correct 5
Septic shock
Inhalation anthrax is a bacterial infection caused by Bacillus anthracis. Manifestations such as septic shock, pleural effusion, and body temperature above 103°F indicate the fulminant stage of inhalation anthrax. The prodromal stage is the early stage of inhalation anthrax; clinical manifestations include a harsh cough and mild chest pain.

...

A nurse discusses the potential for cross-contamination with the nursing assistants on a surgical unit. What does the nurse explain that standard precautions are designed to do?
1
Ensure that hygiene practices by clients are performed in a universal way
2
Be used when clients are suspected of having a communicable disease
3
Create categories in which certain additional precautions must be followed
Correct4
Decrease the risk of transmitting unidentified pathogens
Standard precautions are used for all clients in all settings, regardless of their diagnosis or presumed infectiousness. Practices associated with standard precautions require healthcare providers, not a client, to use hand washing and personal protective equipment to protect themselves and others from body fluids. Transmission-based precautions, known as airborne, droplet, and contact precautions, are based on a client's diagnosed infection.

...

Which autoantigens are responsible for the development of Crohn's disease?
Correct1
Crypt epithelial cells
2
Basement membranes of the lungs
3
Thyroid cell surface
4
Basement membranes of the glomeruli
Crypt epithelial cells are considered to be the autoantigens responsible for Crohn's disease. Thyroid cell surfaces are autoantigens responsible for Hashimoto's thyroiditis. The pulmonary and glomerular basement membranes act as autoantigens responsible for Goodpasture syndrome.

...

Why would a client with acquired immunodeficiency syndrome (AIDS) be administered pregabalin?
Correct1
To reduce muscle and joint pain
2
To reduce cognitive difficulty
Incorrect3
To reduce neuropathic pain
4
To reduce swallowing difficulty
Pregabalin is used to manage muscle and joint pain in clients with AIDS. Neuropathic pain can be managed with amitriptyline, which is a tricyclic antidepressant. Clients with AIDS generally exhibit emotional and behavioral changes, which can be managed with appropriate antidepressants and anxiolytics. AIDS clients who experience difficulty swallowing may have candidal esophagitis; this condition can be managed with antifungal mediations such as fluconazole or amphotericin B.

...

A nurse is caring for a client with pruritic lesions from an IgE-mediated hypersensitivity reaction. Which mediator of injury is involved?
1
Cytokine
Correct2
Histamine
3
Neutrophil
4
Macrophage
Histamine is one of the mediators of injury involving IgE-mediated injury that may cause pruritus. Cytokines are the mediators of injury in delayed hypersensitivity reaction. Neutrophils are involved in immune complex-mediated hypersensitivity reactions. Macrophages in tissues are involved in cytotoxic reactions.

...

The nurse suspects that a client is in the chronic persistent stage of Lyme disease. Which symptoms support the nurse's suspicion? Select all that apply.
1
Erythema migrans
2
Dyspnea
3
Dizziness
Correct 4
Chronic fatigue
Correct 5
Arthritis
Lyme disease is a systemic infectious disease caused by the spirochete Borrelia burgdorferi. The symptoms of the chronic persistent stage are arthritis and chronic fatigue. Dyspnea and dizziness are the symptoms of the early disseminated stage. Erythema migrans is observed in the localized stage.

Test-Taking Tip: Be alert for details about what you are being asked to do. In this Question Type, you are asked to select all options that apply to a given situation or client. All options likely relate to the situation, but only some of the options may relate directly to the situation.

...

A client who abused intravenous drugs was diagnosed with the human immunodeficiency virus (HIV) several years ago. What does the nurse explain to the client regarding the diagnostic criterion for acquired immunodeficiency syndrome (AIDS)?
Incorrect1
Is capable of transmitting the virus to others
2
Contracts HIV-specific antibodies
3
Develops an acute retroviral syndrome
Correct4
Has a CD4+T-cell lymphocyte level of less than 200 cells/µL (60%)
AIDS is diagnosed when an individual with human immunodeficiency virus (HIV) develops one of the following: a CD4+T-cell lymphocyte level of less than 200 cells/µL (60%), wasting syndrome, dementia, one of the listed opportunistic cancers (e.g., Kaposi sarcoma , Burkitt lymphoma), or one of the listed opportunistic infections (e.g., Pneumocystis jiroveci pneumonia, Mycobacterium tuberculosis). The development of HIV-specific antibodies (seroconversion), accompanied by acute retroviral syndrome (flulike syndrome with fever, swollen lymph glands, headache, malaise, nausea, diarrhea, diffuse rash, joint and muscle pain) 1 to 3 weeks after exposure to HIV reflects acquisition of the virus, not the development of AIDS. A client who is HIV positive is capable of transmitting the virus with or without the diagnosis of AIDS.

...

Which client is most likely to develop IgE antibodies?
1
A client undergoing a blood transfusion
Incorrect2
A client with bacterial infection
3
A client undergoing a poison ivy reaction
Correct4
A client with pollen allergy
A client with a pollen allergy develops IgE antibodies that may result in an anaphylactic reaction. A client with poison ivy develops delayed hypersensitivity, which is mediated by T lymphocytes. A client with a bacterial infection develops IgG and IgM antibodies. A client undergoing blood transfusion may develop IgG and IgM type II hypersensitivity reactions.

...

What is the function of a client's natural killer cells?
Correct1
Attack non-selectively on non-self cells, especially mutated and malignant cells
Incorrect2
Enhance immune activity through secretion of various factors, cytokines, and lymphokines
3
Heighten selectively and destroy non-self cells, including virally infected cells
4
Secrete immunoglobulins in response to the presence of a specific antigen
Natural killer cells attack non-selectively on non-self cells, especially body cells that have undergone mutation and become malignant. Plasma cells secrete immunoglobulins in response to the presence of a specific antigen. Cytotoxic T-cells attack selectively and destroy non-self cells, including virally infected cells. Helper T-cells enhance immune activity through secretion of various factors, cytokines, and lymphokines.

...

Which malnutrition condition may predispose a client to secondary immunodeficiency?
1
Hodgkin's lymphoma
2
Cirrhosis
Correct3
Cachexia
4
Diabetes mellitus
Cachexia is a nutrition disorder that may occur due to wasting of muscle mass and weight, resulting in secondary immunodeficiency disorder. Cirrhosis, diabetes mellitus, and Hodgkin's lymphoma also lead to secondary immunodeficiency disorder, but these are not malnutrition disorders.

...

Which autoantigens are responsible for the development of Crohn's disease?
1
Basement membranes of the glomeruli
2
Thyroid cell surface
Correct3
Crypt epithelial cells
4
Basement membranes of the lungs
Crypt epithelial cells are considered to be the autoantigens responsible for Crohn's disease. Thyroid cell surfaces are autoantigens responsible for Hashimoto's thyroiditis. The pulmonary and glomerular basement membranes act as autoantigens responsible for Goodpasture syndrome.

...

Which type of immunity is acquired through the transfer of colostrum from the mother to the child?
1
Artificial active immunity
2
Artificial passive immunity
Correct3
Natural passive immunity
4
Natural active immunity
Natural passive immunity is acquired through the transfer of colostrum from the mother to the child. Natural active immunity is acquired when there is a natural contact with an antigen through a clinical infection. Artificial active immunity is acquired through immunization with an antigen. Artificial passive immunity is acquired by injecting serum from an immune human.

...

Which autoimmune disease is directly related to the client's central nervous system?
Incorrect1
Rheumatic fever
2
Goodpasture syndrome
Correct3
Multiple sclerosis
4
Myasthenia gravis
Multiple sclerosis is a central nervous system-specific autoimmune disease. Rheumatic fever is related to the heart. Myasthenia gravis is a muscle-related autoimmune disease. Goodpasture syndrome is a kidney-related autoimmune disease.

...

Which drug can be administered via the intramuscular route to treat anaphylaxis?
1
Phenylephrine
2
Mycophenolate mofetil
Correct3
Epinephrine
4
Methdilazine
Epinephrine is administered through the intramuscular route to treat anaphylaxis. Methdilazine is administered to treat allergic reactions and pruritus. Phenylephrine is administered orally, not intramuscularly, to treat anaphylaxis. Mycophenolate mofetil is administered intravenously as an immunosuppressant agent.

...

client is treated with methyldopa for hypertension. For which side effect should the nurse monitor the client?
Correct1
Hemolytic anemia
2
Lupus-like syndrome
Incorrect3
Xerostomia
4
Thrombocytopenia
Methyldopa is used in the treatment of hypertension. It can be a precipitating factor in an autoimmune disease such as hemolytic anemia. Scopolamine transdermal, an anticholinergic, may cause dry mouth or xerostomia. Chemotherapy drugs, such as mycophenolate mofetil and azathiprine, can cause thrombocytopenia. Procainamide is an anti-arrhythmic agent that can induce the formation of antinuclear antibodies and cause a lupus-like syndrome.

...

A client's laboratory report shows severe neutropenia and thrombocytopenia. Which medication may have caused this condition?
Incorrect1
Cyclosporine
Correct2
Mycophenolate mofetil
3
Methylprednisolone
4
Daclizumab
Mycophenolate mofetil is a cytotoxic drug that may cause neutropenia and thrombocytopenia. Daclizumab may cause hypersensitivity reaction and anaphylaxis. Cyclosporine may cause neurotoxicity, nephrotoxicity, and hypertension. Methylprednisolone may cause peptic ulcers, osteoporosis, and hyperglycemia.

...

A client presents with red, inflamed skin covered with papules, vesicles, and bullae from a type IV hypersensitivity reaction. Which condition/disease will the nurse most likely observe written in the client's electronic medical record?
1
Systemic lupus erythematosus (SLE)
Correct2
Contact dermatitis
Incorrect3
Allergic rhinitis
4
Goodpasture syndrome
Contact dermatitis is a type of type IV delayed hypersensitivity reaction. Type IV hypersensitivity involves a cell-mediated response that may result in tissue damage and skin lesions. The skin lesions with redness of skin are characterized by the presence of papules, vesicles, and bullae. This indicates the presence of erythematosus resulting in contact dermatitis. Sneezing, lacrimation, swelling with airway obstruction, and pruritus around the eyes, nose, throat, and mouth are symptoms that occur due to hypersensitivity reaction resulting in allergic rhinitis, a type 1 hypersensitivity reaction mediated by immunoglobulin E. Goodpasture syndrome, a type II hypersensitivity reaction, is a disorder involving the lungs and kidneys that causes deposits of immunoglobulin G to form along the basement membranes of the lungs or kidneys. Systemic lupus erythematosus (SLE) is an autoimmune disorder that is characterized by damage to multiple organs such as kidneys, joints, and the brain; SLE is a type III hypersensitivity reaction.

...

The primary healthcare provider has prescribed fluticasone nasal spray for a client with sinusitis. Which of these instructions regarding the use of the spray are appropriate for the client? Select all that apply.
Correct 1
Use the drug on a regular basis, not PRN
2
Remember that driving may be dangerous because of the drug's sedative effect
3
Begin taking the drug 2 weeks before pollen season starts and use it throughout the season
Correct 4
Discontinue use of the drug if nasal infection develops
Correct 5
Clear the nasal passages before using the drug
Fluticasone is a corticosteroid that is prescribed as a nasal spray in cases of sinusitis and rhinitis. The nurse should instruct the client to use the drug on a regular basis, not as needed, to clear the nasal passages before using the drug, and to discontinue use of the drug if nasal infection develops. Warning the client that driving may be dangerous because of the sedative effect would be beneficial for a client prescribed an antihistamine such as brompheniramine. The nurse should tell a client prescribed a mast cell stabilizer such as cromolyn spray to begin the drug 2 weeks before pollen season starts and to use it throughout the pollen season.

...

A client with bubonic plague has a body temperature of 103° F associated with chills, swollen glands, headache, and weakness. Which microorganism is most likely responsible for the client's condition?
1
Bordetella pertusis
2
Mycobacterium tuberculosis
3
Corynebacterium diphtheria
Correct4
Yersinia pestis
Bubonic plague is a reemerging infection caused by Yersinia pestis, which is associated with elevated body temperature, chills, swollen glands, headache, and weakness. Bordetella pertusis causes pertussis, which is associated with acute, highly contagious respiratory disease characterized by loud whooping cough. Mycobacterium tuberculosis causes tuberculosis, which is transmitted by inhalation of infected droplets. Corynebacterium diphtheria causes a localized infection of mucous membranes or skin such as diphtheria.

...

After assessing an older client's medical report, the nurse finds that the client is at an increased risk for bacterial and fungal infections. Which change in immune function may have occurred?
Correct1
Decrease in circulating T-lymphocytes
2
Reduction of colony-forming B-lymphocytes
3
Decline in natural bodies
4
Reduction of neutrophil function
A decrease in circulating T-lymphocytes occurs with cell-mediated immunity, resulting in an increased risk of bacterial and fungal infections. A client would need booster shots for old vaccinations and immunizations when there is a decline in natural antibodies. A reduced neutrophil function may be an implication when neutrophil function is decreased. The older adult should receive immunizations, such as flu shots, when the number of colony-forming B-lymphocytes is diminished.

...

A nurse is developing a teaching plan for a client with scleroderma. What should the nurse include about skin care?
1
Use calamine lotion for pruritus.
Correct2
Keep skin lubricated with lotion.
3
Apply warm soaks to inflamed areas.
4
Take frequent baths to remove scaly lesions.
With scleroderma, the skin becomes dry because of interference with the underlying sweat glands. Pruritus, inflamed areas, and skin lesions are not associated with scleroderma.

...

The nurse suspects the Jarisch-Herxheimer reaction in a client with syphilis who is on antibiotic therapy. Which symptoms in the client support the nurse's suspicion? Select all that apply.
Correct 1
Generalized ache
Correct 2
Fever
3
Vasoconstriction
Correct 4
Pain at the injection site
5
Hypertension
Fever, generalized ache, and pain at the injection site are signs of the Jarisch-Herxheimer reaction in a client with syphilis receiving antibiotic therapy. This reaction is caused by the rapid release of products from the disruption of cells of the organism. Hypotension because of vasodilatation and declining peripheral resistance, not hypertension and vasoconstriction, are additional signs of Jarisch-Herxheimer reaction.

...

A client in the emergency department states, "I was bitten by a raccoon while I was fixing a water pipe in the crawl space of my basement." Which is the most effective first-aid treatment for the nurse to use for this client?
1
Applying a tourniquet proximal to the wound
Incorrect2
Administering an antivenin
Correct3
Cleansing the wound with soap and water
4
Maintaining a pressure dressing
Infection is caused by viral contact with the dermal layer of skin; cleansing the wound with soap and water helps remove superficial contaminants. Antivenins are not effective against microbiologic stresses. A pressure dressing will not prevent infection. Application of a tourniquet may impair circulation and will not prevent infection.

...

A client presents with cutaneous lesions with swelling in the face, eyelids, and lips from dilation and engorgement of the capillaries. No welts or vesicles are observed. Which condition most likely has occurred in the client?
Correct1
Angioedema
2
Atopic dermatitis
3
Urticaria
4
Systemic lupus erythematosus (SLE)
Angioedemais a localized cutaneous lesion similar to urticaria but involving deeper layers of the skin and the submucosa. The principal areas of involvement include the eyelids, lips, tongue, and face. It occurs due to the dilation and engorgement of the capillaries. Urticaria is a cutaneous reaction against systemic allergens occurring in atopic people. It is characterized by transient wheals that may vary in size and shape and occur all over the body. Atopic dermatitis is a chronic, inherited skin disorder that is characterized by exacerbations and remissions. The skin lesions are more generalized and involve vasodilation of blood vessels, resulting in interstitial edema with vesicle formation. SLE is an autoimmune disease that is characterized by damage to multiple organs (kidneys, skin, joints, lungs).

...

A client with P. jiroveci pneumonia (PCP) infection arrives at the hospital for a follow-up visit. The laboratory report shows the CD4+ T-cell count as 150. Which medication will be most beneficial for this client?
Correct1
Pentamidine isethionate
2
Piperacillin/tazobactam
3
Dapsone
Incorrect4
Atovaquone
Aerosolized pentamidine isethionate is used as prophylaxis for clients with CD4+ T-cell counts below 200 as well as for those who already have PCP. Dapsone and atovaquone are used as alternative therapies to trimethoprim/sulfamethoxazole for existing PCP or as prophylaxis. Piperacillin/tazobactam is used to treat certain pneumonias but not P. jiroveci.

...

A nurse in a public health clinic is teaching clients how to prevent toxoplasmosis. What should the nurse instruct the clients to avoid?
Incorrect1
Ingestion of freshwater fish
2
Exposure to heavy metals
Correct3
Contact with cat feces
4
Excessive radiation exposure
Toxoplasma gondii, a protozoan, can be transmitted by exposure to infected cat feces or by ingestion of undercooked, contaminated meat. Toxoplasmosis is not related to heavy metals. T. gondii is a parasite of warm-blooded animals; fish are not considered the source of contamination. Toxoplasmosis is not related to radiation.

...

Which immunodeficiency disorders affect polymorphonuclear leukocytes and monocytes? Select all that apply.
Correct 1
Chronic granulomatous disease
Correct 2
Job syndrome
3
Graft-versus-host disease
4
DiGeorge syndrome
5
Ataxia-telangiectasia
Job syndrome and chronic granulomatous disease are immunodeficiency disorders that affect polymorphonuclear leukocytes and monocytes. DiGeorge syndrome affects T-cells. Ataxia-telangiectasia affects B- and T-cells. Graft-versus-host disease affects B- and T-lymphocytes.

...

Which test result should a nurse review to determine if the antibiotic prescribed for the client will be effective?
Incorrect1
Sedimentation rate
2
Serologic test
3
Serum osmolality
Correct4
Sensitivity test
Infected body fluids are tested to determine the antibiotics to which the organism is particularly sensitive or resistant (sensitivity). The serologic test checks for antibody content. The serum osmolality test provides data about fluid and electrolyte balance. The erythrocyte sedimentation rate (ESR) is a nonspecific test for the presence of inflammation.

...

The T lymphocytes in a healthy older adult client are 120 cells/mm3 (120 cells/uL). The chest x-ray reveals shrinkage of the thymus gland. What might have led to the client's condition?
Incorrect1
Immunosuppression
2
Immunodeficiency
3
Immunocompetence
Correct4
Immunosenescence
Immunosenescence is the primary cause of thymic involution (shrinking) and decrease in number of T cells to 120 cells/mm3 (120 cells/uL) in an older client (normal range in healthy clients: 500 cells/mm3 to 1,200 cells/mm3 ). Immunodeficiency is characterized by lack of or diminished reaction to an antigen or a group of antigens. Immunosuppression is a reduction of the activation or efficacy of the immune system, which occurs due to adverse reactions to treatment of other conditions. Immunocompetence exists when the body's immune system can identify and inactivate or destroy foreign substances.

...

A client with acquired immunodeficiency syndrome (AIDS) is suspected to be infected with Cryptococcus neoforman. Which symptoms in the client support the nurse's suspicion? Select all that apply.
Incorrect 1
Diarrhea
Correct 2
Seizures
Correct 3
Confusion
Incorrect 4
Persistent dry cough
Correct 5
Fever
Cryptococcosis is caused by Cryptococcus neoformans and is a debilitating meningitis. It can be a widely spread infection in a client with AIDS. The symptoms of cryptococcosis are fever, seizures, and confusion because the disease affects the brain. Cryptosporidium infection causes diarrhea. Pneumocystis jiroveci pneumonia causes persistent dry cough.

...

Which leukocyte values should be assessed to determine the adequacy of a client's response to inflammation? Select all that apply.
1
Plasma cells
Correct 2
Monocytes
3
T-helper cells
Correct 4
Macrophages
Correct 5
Neutrophils
In response to inflammation, monocytes destroy bacteria and cellular debris; neutrophils ingest and phagocytize microorganisms and foreign protein; and macrophages destroy bacteria and cellular debris. Plasma cells are a part of antibody-mediated immunity and secrete immunoglobulins in response to the presence of a specific antigen. T-helper cells are a part of cell-mediated immunity and enhance immune activity through the secretion of various factors, cytokines, and lymphokines.

...

Arrange the sequence of steps required to stimulate antibody-mediated immunity in its correct sequence.

Antibody-mediated immunity reactions start when new antigens invade the human body. After exposure, the antigen is recognized by the macrophages. Next virgin B lymphocytes are sensitized to the new antigen. After this action, B lymphocytes produce antibodies. These antibodies then bind to the antigen and an immune complex is formed. The binding of antibodies causes cellular events and results in the elimination of the antigen.

...

A client with Lyme disease presents with dyspnea, dizziness, and facial paralysis. Which medication may be included in the prescription?
Correct1
Ceftriaxone
Incorrect2
Amoxicillin
3
Doxycycline
4
Erythromycin
Lyme disease is a systemic infectious disease caused by the spirochete Borrelia burgdorferi and results from the bite of an infected deer tick, also known as the blacklegged tick. Stage II of Lyme disease is characterized by dizziness, dyspnea, and facial paralysis, and may be treated with ceftriaxone. Amoxicillin, doxycycline, and erythromycin are prescribed to treat localized stage I of Lyme disease.

...

A client reports a cold and a whooping sound with the cough. Which organism is responsible for this condition?
1
Mycobacterium tuberculosis
2
Corynebacterium diphtheria
3
Yersinia pestis
Correct4
Bordetella pertussis
Pertussis is an acute highly contagious respiratory disease characterized by loud whooping inspirations (whooping cough). Pertussis is caused by Bordetella pertussis. Yersinia pestis causes plague. Corynebacterium diphtheria causes diphtheria. Mycobacterium tuberculosis causes tuberculosis.

...

Which pathogen is responsible for the condition illustrated in the image?
3202828073
1
Candida albicans
2
Human herpes virus-8
3
Coccidioides immitis

Correct 4
Epstein-Barr virus

Oral hairy leukoplakia is an infection in which painless, white, raised lesions on the lateral aspect of the tongue are present. Epstein-Barr virus causes this condition. Candida albicans causes oral thrush. Coccidioides immitis causes pneumonia and fever. Human herpes virus-8 causes Kaposi sarcoma.

...

What should the nurse explain as the reason a client prescribed corticosteroid therapy for a chronic health problem develops frequent infections?

1

"They enhance the inflammatory process."

2

"The medication makes the white blood cells work harder."

Correct 3

"They affect antigen-antibody immunity."

Incorrect 4

"They increase the number of inflammatory chemicals in the blood."

Corticosteroids reduce the number of circulating T-cells and suppress cell-mediated immunity. They also interfere with immunoglobulin G (IgG) production and reduce antibody-antigen binding. Corticosteroids suppress the inflammatory process, block the movement of white blood cells, and disrupt the synthesis of a variety of inflammatory

...

A pregnant woman presents with a body temperature of 103 °F, cough, headache, muscle aches, chest pain, severe joint pain and night sweats and is diagnosed with coccidioidomycosis. Which medication will the nurse most likely observe prescribed on the medication administration record?

Correct 1

Amphotericin B

Incorrect 2

Ciprofloxacin

3

Pyrazinamide

4

Doxycycline

An elevated body temperature of 103 °F, cough, headache, muscle aches, chest pain, severe joint pain and night sweats are symptoms of coccidioidomycosis, a fungal infection. Pregnant women can safely take amphotericin B because the drug will not affect the fetus. Doxycycline is a tetracycline that may lead to discoloration of the teeth in the newborn. Ciprofloxacin is a broad-spectrum antibiotic used to treat various bacterial infections and is ineffective with a fungal infection. Pyrazinamide is one of the first-line treatments for tuberculosis.

...

A client has been admitted to the hospital with a diagnosis of methicillin-resistant Staphylococcus aureus (MRSA) in the urine. The client has a urinary catheter in place. No private rooms are available. Which room assignment would be most appropriate for this client?

1

Roommate is bedridden and uses a bedpan for urination.

Correct 2

Roommate has MRSA in the urine. The roommate is ambulatory, but confused.

3

Roommate has a urinary catheter.

Incorrect 4

Roommate is alert and oriented with a diagnosis of pneumonia but practices good hygiene when using the bathroom.

If a private room is not available, the client may be placed in a room with another client who has the same active infection with the same microorganism if no other infection is present. The roommate with a urinary catheter, who is bedridden, or who has pneumonia does not indicate MRSA in the urine.

...

A client with tuberculosis asks the nurse about the communicability of the disease. Which is the best response by the nurse?

Correct 1

"Untreated active tuberculosis is communicable."

2

"With the newer long-term therapies, tuberculosis is not communicable."

3

"Tuberculosis is communicable during the primary stage."

4

"Tuberculosis is not communicable at this time."

The statement that untreated active tuberculosis is communicable is an accurate statement; treatment is necessary to stop communicability. The statement that tuberculosis is not communicable at this time is false reassurance; untreated active tuberculosis is communicable. Tuberculosis is not communicable when there is no active infection; the primary complex refers to the presence of a primary (Ghon) tubercle and enlarged lymph nodes and is the initial response to exposure; active disease may or may not occur. Tuberculosis is a communicable disease; close contacts should be screened via a skin test.

...

A nurse is assessing clients who are to be given the smallpox vaccination. Which client should the nurse remove from the immunization line for medical counseling?

1

75-year-old man with Parkinson disease

Correct 2

45-year-old woman with breast cancer

3

20-year-old healthy woman

4

50-year-old man with diabetes mellitus

The smallpox vaccine should not be given to individuals who may be immunocompromised as a result of therapy for cancer. There is no contraindication to giving the smallpox vaccination to a healthy woman, a client with diabetes mellitus, or a client with Parkinson disease.

...

In clients with Goodpasture syndrome, what is the pathophysiology of this syndrome?

1

Antigen-specific IgE binds to the surface of basophils and mast cells

Correct 2

Auto-antibodies attack the glomerular basement membrane and neutrophils

3

Rapid and systemic reactions of the organs within seconds after exposure to an allergen

4

Autoimmune destruction of the lacrimal and salivary mucus-producing glands

Goodpasture syndrome is an autoimmune disorder in which auto-antibodies attack the glomerular basement membrane and neutrophils, especially in the lungs and kidneys. A client may have allergies when an antigen-specific IgE binds to the surface of basophils and mast cells, which stimulates the production of vasoactive amines. Rapid and systemic reactions of the organs within seconds after exposure to an allergen may lead to anaphylactic shock. A client with Sjögren's syndrome may have the autoimmune destruction of the lacrimal and salivary mucus/moisture-producing glands.

...

Which medications act by binding with integrase enzyme and prevent human immunodeficiency virus (HIV) from incorporating its genetic material into the client's cell? Select all that apply.

I

Ritonavir

2

Nelfinavir

3

Tenofovir

Correct 4

Raltegravir

Correct 5

Elvitegravir

Raltegravir and elvitegravir are integrase inhibitors. They act by binding with integrase enzyme and prevent HIV from incorporating its genetic material into the client's cell. Ritonavir and nelfinavir are protease inhibitors. They act by preventing the protease enzyme from cutting HIV proteins into the proper lengths needed to allow viable virions to assemble and bud out from the cell membrane. Tenofovir is a nucleotide reverse transcriptase inhibitor. It acts by combining with reverse transcriptase enzyme to block the process needed to convert HIV ribose nucleic acid into HIV deoxyribose nucleic acid.

...

A client has been prescribed tacrolimus for immunosuppressant therapy. Which drug safety alert should the nurse mention?

Correct 1

Do not consume grapefruit or grapefruit juice.

2

Take acetaminophen before taking tacrolimus.

3

Once-a-day doses should be taken at bedtime (at least initially).

4

The client should make sure to report any vivid or bizarre dreams.

Tacrolimus is a calcineurin inhibitor prescribed in immunosuppressant therapy. The client should not consume grapefruit or grapefruit juice because a substance in the grapefruit prevents the metabolism of these medications and may increase their toxicity. Monoclonal antibodies may cause fever, chills, headache, and chest pain among the adverse effects; acetaminophen should be taken before this drug is administered. When a client is prescribed efavirenz, a once-a-day dose should be taken at bedtime to help the client cope with the side effects, which may include dizziness and confusion. The nurse should also suggest that the client report experiencing vivid and sometimes bizarre dreams.

...

Scabies

2

Pediculosis

3

Herpes zoster

Correct 4

Cutaneous anthrax

Cutaneous anthrax is an infection caused by the spores of Bacillus anthracis. The primary clinical manifestation is the presence of an ulcer with a vesicular ring, induration, and erythema on the skin. The clinical manifestation of scabies is the presence of visible horizontal white skin ridges. The clinical manifestations of pediculosis are visible white flecks attached to the hair shaft near the scalp. The clinical manifestations of herpes zoster are the presence of fluid-filled blisters on the skin (shingles).

...

Which medications prescribed to a client after a kidney transplant surgery may require the client to visit a dentist? Select all that apply.

1

Sirolimus

Correct 2

Everolimus

3

Prednisone

Correct 4

Cyclosporine

5

Prednisolone

Everolimus is an antiproliferative drug that inhibits cell division in activated lymphocytes. Everolimus could cause gingival hyperplasia as a side effect. Cyclosporine may also cause gingival hyperplasia. Therefore the client may have to visit a dentist because of these medications. Sirolimus may cause leukopenia and thrombocytopenia. Prednisone and prednisolone may cause hypertension and hyperglycemia.

...

Which type of hypersensitivity reaction is present in a client with a body temperature of 102 °F, severe joint pain, rashes on the extremities, and enlarged lymph nodes from serum sickness?

1

Delayed reaction

2

Cytotoxic reaction

3

Immediate reaction

4

Immune complex-mediated reaction

4 Serum sickness is a type III immune complex-mediated reaction. A delayed reaction is a type IV hypersensitivity reaction that may include poison ivy skin rashes, graft rejection, and sarcoidosis. A cytotoxic reaction is a type II hypersensitivity reaction that includes autoimmune hemolytic anemia, Goodpasture syndrome, and myasthenia gravis. An immediate reaction is a type I hypersensitive reaction that includes allergic asthma, hay fever, and anaphylaxis. Test-Taking Tip: Serum sickness is manifested by elevated body temperature, severe joint pain, rashes on limbs, and enlarged lymph nodes. Recall the type of hypersensitivity reaction to which serum sickness belongs.

A client with tuberculosis is prescribed isoniazid. What statements should the nurse tell the client? Select all that apply.

Correct 1

"Take the drug on an empty stomach."

Incorrect 2

"Report any changes in vision to your primary healthcare physician."

Correct 3

"Take daily multiple vitamins that contain B-complex."

4

"Wear protective clothing when going outdoors during the day."

Correct 5

"Report darkening of the urine or a yellowish skin discoloration."

Isoniazid should be taken on an empty stomach because food prevents absorption of the drug. Multiple vitamins that contain the vitamin B-complex should be taken along with isoniazid because the drug depletes vitamin B. A client on isoniazid should report darkening of the urine and yellowish skin discoloration because these conditions are signs of liver toxicity. A client on ethambutol should be taught to report changes in vision. A client on pyrazinamide is instructed to wear protective clothing if he or she will be exposed to sunlight.

...

A client with acquired immunodeficiency syndrome (AIDS) reports speech, gait, and vision difficulty. The nurse observes the client is confused and lethargic. Which microorganism is most likely responsible for this condition?

Incorrect 1

Candida albicans

Correct 2

Toxoplasma gondii

3

Pneumocystis jiroveci

4

Mycobacterium tuberculosis

Toxoplasmosis encephalitis is an opportunistic infection caused by Toxoplasma gondii. The symptoms of toxoplasmosis encephalitis are speech, gait, and vision difficulty along with confusion and lethargy. An overgrowth of Candida albicans causes mouth pain and difficulty swallowing. Pneumocystis jiroveci causes pneumonia in a client with acquired immunodeficiency syndrome (AIDS). Mycobacterium tuberculosis causes tuberculosis in a client with AIDS

...

Why would a primary healthcare provider recommend that a client with acquired immunodeficiency syndrome (AIDS) and Kaposi's sarcoma (KS) wear hats and long sleeves?

Correct 1

To maintain a normal appearance

2

To reduce pain

3

To promote healing

4

To prevent infection

Clients with KS lesions may be advised to wear hats, makeup, or long sleeves to maintain a normal appearance. Pain associated with KS lesions is treated with analgesics and comfort measures. Modified burrow's solution soaks may promote healing in some clients with KS. The cleaning and dressing of KS lesions will prevent infections.

...

When taking the blood pressure of a client who has acquired immunodeficiency syndrome (AIDS), what must the nurse do?

1

Don clean gloves.

2

Use barrier techniques.

3

Put on a mask and gown.

Correct 4

Wash hands thoroughly.

Because this procedure does not involve contact with blood or secretions, additional protection other than washing the hands thoroughly is not indicated. Donning clean gloves and using barrier techniques are necessary only when there is risk of contact with blood or body fluid. A mask and gown are indicated only if there is a danger of secretions or blood splattering on the nurse (for example, during suctioning).

...

A client has undergone scratch testing but the causative allergen is yet to be identified. What would be the next step to confirm a strongly suspected allergen?

1

Administer allergy shots

Correct 2

Begin intradermal testing

3

Request that a client take an oral food challenge

4

Begin radioallergosorbent testing

An intradermal test is administered when a strongly suspected allergen tests negative with the scratch test. A testing dose of sera is injected intradermally into the arm and any signs of allergy are observed to confirm the allergen. Allergy shots are a therapeutic method of reducing sensitivity to a known allergen when exposure cannot be avoided. An oral food challenge is done to confirm an ingested allergen if skin testing completely fails. Radioallergosorbent testing is useful to measure immunoglobulin E levels to ascertain the presence of an allergic reaction.

...

While assessing a client with acquired immunodeficiency syndrome (AIDS), the nurse suspects that the client has developed cryptococcosis. Which clinical manifestations support the nurse's suspicion of a cryptococcosis infection? Select all that apply.

Correct 1

Seizures

Incorrect 2

Dyspnea

Correct 3

Blurred vision

Correct 4

Neurologic deficits

5

Enlarged lymph nodes

Seizures, neurologic problems/deficits, and blurred vision are the manifestations of cryptococcosis. Cryptococcosis is a debilitating meningitis and can be a widely spread infection in clients who have AIDS. It is caused by Cryptococcus neoformans. Histoplasmosis is a respiratory infection caused by Histoplasma capsulatum, which progresses to widespread infection in a client with AIDS. The symptoms of histoplasmosis are dyspnea and enlarged lymph nodes

...

Which throat manifestations are the key features for a client with acute viral pharyngitis? Select all that apply.

Incorrect 1

Petechiae on the soft palate

Correct 2

Scant or no tonsillar exudate

Correct 3

Slight erythema of the pharynx and tonsils

4

Severe hyperemia of the pharyngeal mucosa

5

Erythema of the tonsils with yellow exudates

The throat manifestations of acute viral pharyngitis are scant or no tonsillar exudate and slight erythema of the pharynx and tonsils. Petechiae on the soft palate, severe hyperemia of the pharyngeal mucosa, and erythema of the tonsils with yellow exudates are the manifestations of acute bacterial pharyngitis.

...

Arrange in order the steps involved in skin testing for an allergen causing a type I allergic reaction.

The first step is to discontinue corticosteroid medications 2 weeks prior to the test to prevent a suppressed immune response. Then, on the day of testing, the test site (the inner side of the forearm) should be cleaned with soap and water to remove surface contaminants. Then drops of sera containing a known allergen should be placed on the skin. The skin is then scratched through the drop with the use of a skin testing needle. Control drops are also applied to compare reactions. Then positive signs for allergic reactions, such as erythema or wheal formation, should be observed. Finally the client should be discharged after a topical steroid is applied on the skin.

Test-Taking Tip: In this Question Type, you are asked to prioritize (put in order) the options presented. For example, you might be asked the steps of performing an action or skill such as those involved in medication administration.

...

The skin of a client with syphilis is illustrated in the image. Which stage of syphilis is present in the client?
3202827598

Correct 1

Late

2

Latent

Incorrect 3

Primary

4

Secondary

Syphilis is a sexually transmitted disease caused by Treponema pallidum. In the late stage of the disease, chronic destructive lesions (gummas) are present. Signs and symptoms are absent in the latent stage of syphilis. In the primary stage of syphilis, indurated lesions are present on the skin. In the secondary stage of syphilis, a mucous patch is present in the mouth.

...

Which antitubercular medications may increase a client's risk for gout? Select all that apply.

Incorrect 1

Rifampin

2

Isoniazid

3

Bedaquiline

Correct 4

Ethambutol

Correct 5

Pyrazinamide

Pyrazinamide and ethambutol increase uric acid formation and may increase the risk of gout. Rifampin and isoniazid may lead to liver toxicity. Bedaquiline does not increase the risk of gout; it is used for multidrug-resistant tuberculosis

...

A client undergoing tuberculosis therapy reports eye pain that worsens when moving the eyes with decreased color vision. Which medication most likely is responsible for the client's condition?

1

Rifampin

2

Isoniazid

3

Ethambutol

4

Pyrazinamide

Ethambutol Eye pain that is worsened when the eyes are moved with decreased color vision may be indicative of optic neuritis. Ethambutol, especially at high dosages, can cause optic neuritis, a condition that can result in blindness. Rifampin reduces the effectiveness of oral contraceptives, increasing the risk of an unplanned pregnancy, and can change bodily fluid orange. Isoniazid can deplete the body of the B-complex vitamins. Pyrazinamide increases uric acid formation and worsens gout.

A tuberculin skin test with purified protein derivative (PPD) tuberculin is performed as part of a routine physical examination. When does the nurse instruct the client to make an appointment so the test can be read?

1

1 week

2

12 hours

3

24 to 48 hours

4

48 to 72 hours

It takes 48 to 72 hours for antibodies to respond to the antigen and form an indurated area. The results of tuberculosis skin tests that are not read within this timeframe will not be accurate.

Which medication class helps to prevent human immunodeficiency virus (HIV) incorporating its genetic material into the client's cell?

1

Entry inhibitors

2

Protease inhibitors

3

Integrase inhibitors

4

Reverse transcriptase inhibitors

Integrase inhibitors such as raltegravir and dolutegravir bind with integrase enzymes and prevent HIV from incorporating its genetic material into the host (client's) cell. Entry inhibitors prevent the binding of HIV. Protease inhibitors prevent the protease enzyme from cutting HIV proteins into the proper lengths needed to allow viable virions to assemble. Reverse transcriptase inhibitors inhibit the action of reverse transcriptase

A client presents with a localized bacterial infection of mucous membranes. Which organism is most likely responsible for this condition in the client?

1

Giardia

2

Aspergillus fumigatus

3

Corynebacterium diphtheria

4

Mycobacterium tuberculosis

Corynebacterium diphtheria Diphtheria is a re-emerging infection that can be characterized by localized infection of mucous membranes or skin. It is caused by Corynebacterium diphtheria. Giardia, a parasite, causes giardiasis, a diarrheal illness known as traveler's diarrhea. Aspergillosis is a lung disease caused by Aspergillus fumigatus, a fungus. Mycobacterium tuberculosis causes tuberculosis.

Human herpes virus-8

The image illustrates lesions in Kaposi sarcoma, a condition caused by Human herpes virus-8. Kaposi sarcoma develops as small, purplish-brown, raised lesions that are usually not painful or itchy. The skin and mucous membrane lesions can occur anywhere on the body. Candida albicans causes oral thrush that affects the hard and soft palates. Epstein-Barr virus causes oral hairy leukoplakia, which is a painless, white, raised lesion on the lips and tongue. Toxoplasma gondii causes toxoplasmosis encephalitis, which results in changes in mental status.

...

A nurse is concerned about the public health implications of gonorrhea diagnosed in a 16-year-old adolescent. Which should be of most concern to the nurse?

1

Interviewing the client's parents

2

Finding the client's sexual contacts

3

Instructing the client about birth control measures

4

Determining the reasons for the client's promiscuity

2 Gonorrhea is a highly contagious disease transmitted through sexual intercourse. The incubation period varies, but symptoms usually occur 2 to 10 days after contact. Early effective treatment prevents complications such as sterility. The parents may be unaware that their child has gonorrhea. Most birth control measures do not protect against the transmission of sexually transmitted infections. Contracting venereal infection is not necessarily indicative of promiscuity.

While assessing a client with acquired immunodeficiency syndrome (AIDS), the nurse suspects that the client has developed cryptococcosis. Which clinical manifestations support the nurse's suspicion of a cryptococcosis infection? Select all that apply.

Correct 1

Seizures

Incorrect 2

Dyspnea

Correct 3

Blurred vision

Correct 4

Neurologic deficits

5

Enlarged lymph nodes

Seizures, neurologic problems/deficits, and blurred vision are the manifestations of cryptococcosis. Cryptococcosis is a debilitating meningitis and can be a widely spread infection in clients who have AIDS. It is caused by Cryptococcus neoformans. Histoplasmosis is a respiratory infection caused by Histoplasma capsulatum, which progresses to widespread infection in a client with AIDS. The symptoms of histoplasmosis are dyspnea and enlarged lymph nodes.

14%
of students nationwide answered this question correctly.

View Topics

ID: 3203094436

Confidence: Nailed it

Stats

Issue with this question?

3.

Which throat manifestations are the key features for a client with acute viral pharyngitis? Select all that apply.

Incorrect 1

Petechiae on the soft palate

Correct 2

Scant or no tonsillar exudate

Correct 3

Slight erythema of the pharynx and tonsils

4

Severe hyperemia of the pharyngeal mucosa

5

Erythema of the tonsils with yellow exudates

The throat manifestations of acute viral pharyngitis are scant or no tonsillar exudate and slight erythema of the pharynx and tonsils. Petechiae on the soft palate, severe hyperemia of the pharyngeal mucosa, and erythema of the tonsils with yellow exudates are the manifestations of acute bacterial pharyngitis.

14%
of students nationwide answered this question correctly.

View Topics

ID: 3202802806

Confidence: Just a guess

Stats

Issue with this question?

4.

Arrange in order the steps involved in skin testing for an allergen causing a type I allergic reaction.

Incorrect

1.

Use a skin testing needle to scratch the surface of the skin.

Incorrect

2.

Apply topical steroid on the skin.

Incorrect

3.

Observe for a wheal formation on the skin.

Incorrect

4.

Discontinue corticosteroid medications.

Incorrect

5.

Clean the inner side of the forearm with soap and water.

Incorrect

6.

Place drops of sera containing the known allergen on skin.

The first step is to discontinue corticosteroid medications 2 weeks prior to the test to prevent a suppressed immune response. Then, on the day of testing, the test site (the inner side of the forearm) should be cleaned with soap and water to remove surface contaminants. Then drops of sera containing a known allergen should be placed on the skin. The skin is then scratched through the drop with the use of a skin testing needle. Control drops are also applied to compare reactions. Then positive signs for allergic reactions, such as erythema or wheal formation, should be observed. Finally the client should be discharged after a topical steroid is applied on the skin.

Test-Taking Tip: In this Question Type, you are asked to prioritize (put in order) the options presented. For example, you might be asked the steps of performing an action or skill such as those involved in medication administration.

11%
of students nationwide answered this question correctly.

View Topics

ID: 3202827536

Confidence: Nailed it

Stats

Issue with this question?

5.

The skin of a client with syphilis is illustrated in the image. Which stage of syphilis is present in the client?
3202827598

Correct 1

Late

2

Latent

Incorrect 3

Primary

4

Secondary

Syphilis is a sexually transmitted disease caused by Treponema pallidum. In the late stage of the disease, chronic destructive lesions (gummas) are present. Signs and symptoms are absent in the latent stage of syphilis. In the primary stage of syphilis, indurated lesions are present on the skin. In the secondary stage of syphilis, a mucous patch is present in the mouth.

22%
of students nationwide answered this question correctly.

View Topics

ID: 3202809469

Confidence: Just a guess

Stats

Issue with this question?

6.

Which antitubercular medications may increase a client's risk for gout? Select all that apply.

Incorrect 1

Rifampin

2

Isoniazid

3

Bedaquiline

Correct 4

Ethambutol

Correct 5

Pyrazinamide

Pyrazinamide and ethambutol increase uric acid formation and may increase the risk of gout. Rifampin and isoniazid may lead to liver toxicity. Bedaquiline does not increase the risk of gout; it is used for multidrug-resistant tuberculosis.

17%
of students nationwide answered this question correctly.

View Topics

ID: 3203094061

Confidence: Just a guess

Stats

Issue with this question?

7.

A client undergoing tuberculosis therapy reports eye pain that worsens when moving the eyes with decreased color vision. Which medication most likely is responsible for the client's condition?

Incorrect 1

Rifampin

2

Isoniazid

Correct 3

Ethambutol

4

Pyrazinamide

Eye pain that is worsened when the eyes are moved with decreased color vision may be indicative of optic neuritis. Ethambutol, especially at high dosages, can cause optic neuritis, a condition that can result in blindness. Rifampin reduces the effectiveness of oral contraceptives, increasing the risk of an unplanned pregnancy, and can change bodily fluid orange. Isoniazid can deplete the body of the B-complex vitamins. Pyrazinamide increases uric acid formation and worsens gout.

Test-Taking Tip: Be alert for details. Details provided in the stem of the item, such as behavioral changes or clinical changes (or both) within a certain time period, can provide a clue to the most appropriate response or, in some cases, responses.

46%
of students nationwide answered this question correctly.

View Topics

ID: 3028952565

Confidence: Just a guess

Stats

Issue with this question?

8.

A tuberculin skin test with purified protein derivative (PPD) tuberculin is performed as part of a routine physical examination. When does the nurse instruct the client to make an appointment so the test can be read?

Incorrect 1

1 week

2

12 hours

3

24 to 48 hours

Correct 4

48 to 72 hours

It takes 48 to 72 hours for antibodies to respond to the antigen and form an indurated area. The results of tuberculosis skin tests that are not read within this timeframe will not be accurate.

Test-Taking Tip: As you answer each question, write a few words about why you think that answer is correct; in other words, justify why you selected that answer. If an answer you provide is a guess, mark the question to identify it. This will permit you to recognize areas that need further review. It will also help you to see how correct your "guessing" can be. Remember: on the licensure examination you must answer each question before moving on to the next question.

75%
of students nationwide answered this question correctly.

View Topics

ID: 3203024783

Confidence: Just a guess

Stats

Issue with this question?

9.

Which medication class helps to prevent human immunodeficiency virus (HIV) incorporating its genetic material into the client's cell?

1

Entry inhibitors

2

Protease inhibitors

Correct 3

Integrase inhibitors

Incorrect 4

Reverse transcriptase inhibitors

Integrase inhibitors such as raltegravir and dolutegravir bind with integrase enzymes and prevent HIV from incorporating its genetic material into the host (client's) cell. Entry inhibitors prevent the binding of HIV. Protease inhibitors prevent the protease enzyme from cutting HIV proteins into the proper lengths needed to allow viable virions to assemble. Reverse transcriptase inhibitors inhibit the action of reverse transcriptase.

33%
of students nationwide answered this question correctly.

View Topics

ID: 3202921073

Confidence: Just a guess

Stats

Issue with this question?

10.

A client presents with a localized bacterial infection of mucous membranes. Which organism is most likely responsible for this condition in the client?

1

Giardia

Incorrect 2

Aspergillus fumigatus

Correct 3

Corynebacterium diphtheria

4

Mycobacterium tuberculosis

Diphtheria is a re-emerging infection that can be characterized by localized infection of mucous membranes or skin. It is caused by Corynebacterium diphtheria. Giardia, a parasite, causes giardiasis, a diarrheal illness known as traveler's diarrhea. Aspergillosis is a lung disease caused by Aspergillus fumigatus, a fungus. Mycobacterium tuberculosis causes tuberculosis.

19%
of students nationwide answered this question correctly.

View Topics

ID: 3156449189

Confidence: Nailed it

Stats

Issue with this question?

11.

Which organism is responsible for the condition illustrated in the image?
3156449191

1

Candida albicans

Incorrect 2

Epstein-Barr virus

3

Toxoplasma gondii

Correct 4

Human herpes virus-8

The image illustrates lesions in Kaposi sarcoma, a condition caused by Human herpes virus-8. Kaposi sarcoma develops as small, purplish-brown, raised lesions that are usually not painful or itchy. The skin and mucous membrane lesions can occur anywhere on the body. Candida albicans causes oral thrush that affects the hard and soft palates. Epstein-Barr virus causes oral hairy leukoplakia, which is a painless, white, raised lesion on the lips and tongue. Toxoplasma gondii causes toxoplasmosis encephalitis, which results in changes in mental status.

46%
of students nationwide answered this question correctly.

View Topics

«
1
2
»

EAQ Home

Class Tools

Progress

Assignments

Grades

Custom Quiz

Elsevier Evolve

Credits

...

The nurse is preparing a teaching plan for clients receiving antitubercular medications. Which teaching plan needs correction?

1

Clients taking ethambutol should drink plenty of fluids.

Correct 2

Clients taking Isoniazid should take the drug with food.

3

Clients taking pyrazinamide should wear a hat while going out in the sun.

Incorrect 4

Clients taking rifampin should use other contraceptive methods even after stopping the medication.

Taking isoniazid with food should be corrected. The presence of food may slow or even prevent the absorption of isoniazid from the stomach. Therefore the client should take the medication on an empty stomach, either 1 hour before or 2 hours after eating. Ethambutol may increase uric acid formation. The client should drink plenty of water to reduce uric acid precipitation and kidney problems. Pyrazinamide is a photosensitive medication that may increase the risk of sunburn. Therefore the client should wear a hat and protect himself or herself from sun exposure. Rifampin may decrease the efficiency of oral contraceptives. Therefore the nurse should instruct the client to use an additional method of contraception even after stopping the medication.

...

The nurse teaches a group of clients that nutritional support of natural defense mechanisms indicates the need for a diet high in what nutrient or nutrients?

1

Essential fatty acids

2

Dietary cellulose and fiber

3

Tryptophan, an amino acid

Correct 4

Vitamins A, C, E, and selenium

Vitamins A, C, E, and selenium stimulate the immune system. The role of fatty acids in natural defense mechanisms is uncertain. Dietary cellulose and fiber have no known effect on natural defense mechanisms. Tryptophan has no known effect on natural defense mechanisms.

...

The image illustrates the toe of a client with a sexually transmitted infection (STI). Which other symptom should a nurse assess for in the client with this disease?
3203066447

1

Gummas on the skin

Incorrect 2

Loss of vision and perception

Correct 3

Mucopurulent cervical discharge

4

Unilateral scrotal pain and swelling

Gonorrhea is an STI caused by Neisseria gonorrhoeae. The image depicts skin lesions with disseminated gonococcal infection, which is also manifested by mucopurulent cervical discharge. Gummas on the skin accompanied with a loss of vision and perception are symptoms of tertiary syphilis. Unilateral scrotal pain and swelling are symptoms of a chlamydial infection.

...

A client receiving chemotherapy takes a steroid daily. The client has a white blood cell count of 12,000/mm3 (12 X 109/L) and a red blood cell count of 4.5 million/mm3 (4.5 X 1012/L). What is the priority instruction that the nurse should teach the client?

1

Omit the daily dose of prednisone.

Correct 2

Avoid large crowds and persons with infections.

Incorrect 3

Shave with an electric shaver rather than a safety razor.

4

Increase the intake of high-protein foods and red meats.

Moderate leukopenia increases the risk of infection; the client should be taught protective measures. Leukopenia is a side effect of cyclophosphamide, not prednisone. The platelet count has not been provided, so bleeding precautions are not indicated. Increasing the intake of high-protein foods and red meat are measures to correct anemia; protection from infection takes priority.

...

While assessing a client with presence of neurotoxicity, lymphoma, and hypertension, the medical history reveals that the client is on immunosuppressant drug therapy. Which drug class might have caused these conditions?

1

Corticosteroids

Incorrect 2

Cytotoxic drugs

3

Monoclonal antibodies

Correct 4

Calcineurin inhibitors

Calcineurin inhibitors such as cyclosporine act on T helper cells to prevent production and release of IL-2 and γ-interferon. This class of medications can cause adverse effects such as nephrotoxicity, lymphoma, hypertension, gingival hyperplasia, and hirsutism. Corticosteroids may cause peptic ulcer, osteoporosis, and hyperglycemia. Cytotoxic drugs may cause bone marrow suppression, hypertension, diarrhea, and nausea. Monoclonal antibodies may cause pulmonary edema, hypersensitivity reactions, fever/chills, and chest pain.

...

A client is admitted with full-blown anaphylactic shock that developed due to a type 1 latex allergic reaction. Which findings will the nurse observe upon assessment? Select all that apply.

Correct 1

Stridor

2

Fissuring

Correct 3

Hypotension

Correct 4

Dyspnea

5

Cracking of the skin

Full-blown anaphylactic shock produces stridor, hypotension, and dyspnea. Fissuring and cracking of the skin occurs in individuals with a type IV contact dermatitis.

...

The nurse educates a client on decreasing the risk of developing antibiotic-resistant infections. Which statement made by the nurse will be most significant?

Incorrect 1

"Wash your hands frequently."

Correct 2

"Do not skip any dose of your antibiotics."

3

"Save the unfinished antibiotics for later use."

4

"Stop taking the antibiotics when you feel better."

Antibiotic-resistant infection develops when the hardiest bacteria survive and multiply. This may happen when a client stops taking an entire course of antibiotics, which leads to infections that are resistant to many antibiotics. Therefore a client should not skip any dose of an antibiotic. Hand washing is required to prevent infections; it is not related to antibiotic-resistant infections. Antibiotics should not be stopped even if the client has started feeling better; the full course of treatment should be taken. Non-compliance in taking the full course of prescribed antibiotics can lead to an antibiotic-resistant infection. It is dangerous to take the unfinished antibiotics at a later time; it may prove fatal if the antibiotics are outdated.

...

Which organism causes Hansen's disease?

Incorrect 1

Clostridium tetani

2

Haemophilus pertusis

Correct 3

Mycobacterium leprae

4

Legionella pneumophila

Mycobacterium leprae causes Hansen's disease (leprosy). Clostridium tetani causes tetanus (lockjaw). Haemophilus pertusis causes pertussis (whooping cough) and Legionella pneumophila causes pneumonia (Legionnaires' disease).

...

A nurse is preparing to obtain a blood specimen for culture and sensitivity from a client with an elevated temperature for the last 2 days. Place in order of priority the nursing actions that should be taken.

First, the procedure should be explained to the client. Then the specimen should be collected and sent to the laboratory. The antibiotic should be administered last. If it were administered before the specimen was collected, the test results might not indicate the cause of the fever.

...

A client who works manufacturing latex gloves presents with dryness, pruritus, fissuring, and cracking of the skin followed by redness and inflammation about 24 hours after contact. The nurse identifies it as an allergic reaction. Which condition most likely has occurred?

Incorrect 1

Type I allergic reaction

Correct 2

Type IV contact dermatitis reaction

3

Immune complex reaction

4

Cytotoxic hypersensitivity reaction

Type IV contact dermatitis is caused by the chemicals used in the manufacturing process of latex gloves. It is characterized by dryness, pruritus, fissuring, and cracking of the skin and occurs within 6 to 48 hours of contact. Type I allergic reaction is a response to the natural rubber latex proteins and occurs within minutes of contact with the proteins; skin redness, urticaria, rhinitis, and conjunctivitis are the clinical manifestations. Immune-complex reaction is a type III hypersensitivity. The kidneys, skin, joints, blood vessels and lungs are common sites for deposit. Cytotoxic hypersensitivity reactions involvethe direct binding of IgG or IgM antibodies to an antigen on the cell surface and is a type II hypersensitivity.

Test-Taking Tip: Consider the types of latex allergies and recollect the symptoms associated with each condition to choose the correct answer.

...

A client with jaundice reports fatigue, abdominal pain, loss of appetite, dark urine, nausea, and vomiting. Which pathogen is most likely responsible for the client's condition?

Correct 1

Hepatitis C virus

2

Candida albicans

3

Varicella zoster virus

4

Cryptosporidium muris

A client with jaundice, fatigue, abdominal pain, loss of appetite, dark urine, nausea, and vomiting may have hepatitis, caused by hepatitis C virus. Candida albicans is a fungus that causes oral thrush, esophagitis, and vaginitis. Varicella zoster virus causes shingles, an erythematous maculopapular rash along dermatomal planes, pain, pruritis, and progressive outer retinal necrosis. Cryptosporidium muris causes gastroenteritis, watery diarrhea, abdominal pain, and weight loss.

...

A client with scleroderma reports having difficulty chewing and swallowing. What should the nurse recommend to safely facilitate eating?

Incorrect 1

Liquefy food in a blender.

Correct 2

Eat a mechanical soft diet.

3

Take frequent sips of water with meals.

4

Use a local anesthetic mouthwash before eating.

Scleroderma causes chronic hardening and shrinking of the connective tissues of any organ of the body, including the esophagus and face; a mechanical soft diet includes foods that limit the need to chew and are easier to swallow. Liquefied foods are difficult to swallow; esophageal peristalsis is decreased, and liquids are aspirated easily. Taking frequent sips of water with meals will not help; it is equally difficult to swallow solids and liquids, and aspiration may result. Using a local anesthetic mouthwash before eating is not necessary; oral pain is not associated with scleroderma.

STUDY TIP: Laughter is a great stress reliever. Watching a short program that makes you laugh, reading something funny, or sharing humor with friends helps decrease stress.

...

A client with mild diarrhea is diagnosed with a Clostridium difficile infection. Which is the first-line drug that would be used to treat this condition?

1

Rifaximin

2

Fidaxomicin

Incorrect 3

Vancomycin

Correct 4

Metronidazole

Metronidazole is the first line of treatment prescribed to clients with a Clostridium difficile infection. Rifaximin is used to treat traveler's diarrhea caused by Escherichia coli. Fidaxomicin is reserved for clients who are at risk for the relapse of or have recurrent Clostridium difficile infections. Vancomycin is preferred for serious Clostridium difficile infections.

...

While assessing the mouth of a client with acquired immunodeficiency syndrome (AIDS), the nurse finds the condition illustrated in the image. Which pathogen is responsible for the client's condition?
3202921058

1

Cryptosporidium

2

Candida albicans

3

Toxoplasma gondii

4

Histoplasma capsulatum

...

The nurse is caring for a client on antiretroviral therapy who has Pneumocystis jiroveci pneumonia. Which action is priority?

1

Assisting the client in eating and drinking

2

Maintaining fluid balance in the client

3

Providing adequate oxygenation for the client

4

Encouraging the client to perform breathing exercise

3

A client with radiation therapy for neck cancer reports, "I feel a lump while swallowing and foods get stuck." What does the nurse document in the client's medical history?

Incorrect 1

Dysgeusia

Correct 2

Dysphagia

3

Xerostomia

4

Odynophagia

Dysphagia is having difficulty while swallowing. This characterizes pharyngeal and esophageal involvement, which further impedes eating. In this condition, the client may report a feeling of having a "lump" when swallowing and feeling that "foods get stuck." Dysgeusia is the loss of taste; clients will report that all food has lost its flavor. While xerostomia may contribute to difficulty swallowing, it is not the term used; xerostomia is used to indicate dry mouth. Odynophagia is painful swallowing; clients will report severe pain while swallowing.

...

A client has colorectal cancer and is receiving cetuximab. Which process does cetuximab inhibit?

Incorrect 1

Proteasome activity

2

BCR-ABL tyrosine kinase (TK)

3

Anaplastic lymphoma kinase

Correct 4

Epidermal growth factor receptors (EGFRs)

Cetuximab is an EGFR-tyrosine TK inhibitor that acts by inhibiting EGFRs in clients with colorectal cancer. Bortezomib inhibits proteasome activity in clients with multiple myeloma. Dasatinib acts by inhibiting BCR-ABL TK in clients with chronic myeloid leukemia. Crizotinib acts by inhibiting anaplastic lymphoma kinase (ALK) in clients with locally advanced or metastatic non-small cell lung cancer that is ALK positive.

...

Place the pathophysiologic process of tuberculosis infection in its correct order.

tuberculosis is a highly communicable disease caused by Mycobacterium tuberculosis. The process of infection in tuberculosis starts by formation of granulomatous inflammation by tuberculosis bacillus in lungs. This granulomatous inflammation then becomes surrounded by collagen, fibroblasts, and lymphocytes. The necrotic tissue then turns into a granular mass, called a caseation necrosis, which occurs in the center of the lesion. Then the areas of caseation undergo resorption, degeneration, and fibrosis. Finally, the necrotic areas undergo calcification or liquefaction.

...

The nurse is teaching a client about automatic epinephrine injectors. Which statement made by the client indicates a need for additional education?
Correct1
"I will keep the device in the refrigerator."
2
"I will keep the device away from light."
3
"If the cap is loose, I will obtain a replacement device."
4
"I will have at least two drug-filled devices on hand at all times."
The device should be protected from extreme temperatures. Therefore the device should not be refrigerated. The device should be protected from light. If the cap is loose or comes off accidentally, the client should obtain a replacement device. The client should have at least two drug-filled devices on hand in case more than one dose is required.
76%of students nationwide answered this question correctly.
View Topics
3156442181
Confidence: Nailed it
Stats
Issue with this question?
4.
What should the nurse expect to assess in a client with a type IV delayed hypersensitivity reaction? Select all that apply.
Correct 1
Edema
2
Bruising
Correct 3
Ischemia
Correct 4
Induration
Correct 5
Tissue damage
A type IV delayed hypersensitivity reaction consists of edema, ischemia, induration, and tissue damage at the site. Bruising is not a typical occurrence in a type IV delayed hypersensitivity reaction.
14%of students nationwide answered this question correctly.
View Topics
3202830409
Confidence: Pretty sure
Stats
Issue with this question?
6.
A client reports nasal discharge, sneezing, lacrimation, and itching all over the body. During an assessment, the client reports that he or she was exposed to dust and molds. The nurse observes swelling and a pink color at the itching site. Which class of drugs is best used to treat this condition in the client?
Correct1
Antihistamines
2
Antipruritic drugs
3
Mast cell-stabilizing drugs
4
Leukotriene receptor antagonists
Allergic rhinitis is a hypersensitivity reaction that occurs due to airborne substances such as dust, pollen, and molds. The symptoms of allergic rhinitis are nasal discharge, sneezing, lacrimation, and pruritus around the eyes, nose, and throat. Urticaria is a cutaneous reaction characterized by pink, raised, edematous, pruritic areas that vary in size and shape and may occur all over the body. Antihistamines are the best drugs used to treat allergic rhinitis and urticaria. Antipruritic drugs are effective only in relieving itching. Mast cell-stabilizing drugs are used to manage allergic rhinitis. Leukotriene receptor antagonists are most effective in treating allergic rhinitis and asthma.
73%of students nationwide answered this question correctly.
View Topics
3156444017
Confidence: Pretty sure
Stats
Issue with this question?
7.
Which reason should the nurse request that the healthcare provider increase the intravenous fluid infusion for an older client with an infection?
1
Pruritus
2
Erythema
Correct3
Acute confusion
4
General malaise
The nurse should consider the development of dehydration if acute confusion occurs in an older client with an infection. Additional fluids would not be helpful if pruritus, erythema, or general malaise develop in a client with an infection.
90%of students nationwide answered this question correctly.
View Topics
3202675407
Confidence: Just a guess
Stats
Issue with this question?
9.
Chart/Exhibit 1

The nurse is reviewing the medication chart of four clients. Which client's medication chart indicates a need for correction?
Correct1
Client A
2
Client B
3
Client C
4
Client D
Enterococcus faecalis can cause a urinary tract infection, which is treated with penicillin G or ampicillin. E. faecalis in client A is resistant to medications such as streptomycin, vancomycin, and gentamicin and should be corrected by the nurse. All the rest are correct treatments. Staphylococcus epidermidis associated osteomyelitis in client B can be effectively treated with vancomycin. Streptococcus pneumoniae associated pneumococcal pneumonia in client C can be treated safely with cefotaxime and ceftriaxone. Klebsiella pneumoniae associated pneumonia in client D is treated with meropenem.

Test-Taking Tip: Apply your analytical thinking to recollect the medications that the pathogens are resistance to and select the correct answer.
55%of students nationwide answered this question correctly.
View Topics
3028949841
Confidence: Just a guess
Stats
Issue with this question?
11.
The nurse is caring for a client with sepsis who is hemodynamically stable. The client is complaining of abdominal pain. Which of these primary health care provider prescriptions should the nurse do first?
Correct1
Draw peripheral blood cultures.
2
Administer levofloxacin 500 mg intravenously over 30 minutes.
3
Administer 1 L intravenous bolus of Ringer's lactate over 30 minutes.
4
Take the client to x-ray for an abdominal computed tomography (CT) scan.
This question requires the learner to recall the priority treatments for clients with sepsis. Mortality in septic clients increases by 7.6% for every hour an antibiotic is delayed. Because this client is hemodynamically stable, the priority is to draw the blood cultures so the antibiotic can be initiated as soon as possible. Administering the antibiotic before obtaining blood cultures could mask the infection, delaying appropriate treatment. Taking the client to x-ray before obtaining the blood cultures would delay antibiotic initiation.

Test-Taking Tip: Notice how the subjects of the questions are related and, through that relationship, the answers to some of the questions may be provided within other questions of the test.
50%of students nationwide answered this question correctly.
View Topics
3203037629
Confidence: Just a guess
Stats
Issue with this question?
12.
A client reports painful and bleeding gingivae, increased saliva with metallic taste, fetid mouth odor, decreased hunger, fever, and general malaise. Upon assessment, the nurse finds eroding necrotic lesions of the interdental papillae and bleeding ulcerations in the mouth. Which microorganism is responsible for this condition?
1
Candida species
Correct2
Spirochetes
3
Herpes simplex viruses
4
Staphylococcus species
Painful, bleeding gingivae, eroding necrotic lesions of interdental papillae, ulcerations that bleed, increased saliva with metallic taste, fetid mouth odor, anorexia, fever, and general malaise are symptoms of Vincent's infection, which is caused by spirochetes. The Candida species cause oral candidiasis manifested as pearly, bluish white membranous lesions on the mucosa of the mouth and larynx. Herpes simplex viruses cause cold sores manifested as lesions on lips and vesicle formation. The Staphylococcus species causes parotitis, which is manifested as pain in the area of the gland and ear, absence of salivation, and purulent exudate from the gland.
67%of students nationwide answered this question correctly.
View Topics
EAQ Home
Class Tools
Progress
Assignments
Grades
Custom Quiz
Elsevier Evolve Credits

...

A primary health care provider prescribes airborne precautions for a client with tuberculosis. After being taught about the details of airborne precautions, the client is seen walking down the hall to get a glass of juice from the kitchen. What is the most effective nursing intervention?
1
Ensure regular visits by staff members.
Correct2
Explore what the precautions mean to the client.
3
Report the situation to the infection control nurse.
Incorrect4
Reteach the concepts of airborne precautions to the client.
Communication facilitates joint solution of the problem; the nurse must first determine the client's understanding and perceptions before solutions to the problem can be attempted. Ensuring regular visits by staff members will not collect data about why the client is leaving the room. Reporting the situation to the infection control nurse abdicates the responsibility of the primary nurse. Reteaching the concepts of airborne precautions to the client may be done, but not until further assessment is performed to determine the reason why the client is leaving the room.

Test-Taking Tip: A psychological technique used to boost your test-taking confidence is to look into a mirror whenever you pass one and say out loud, "I know the material, and I'll do well on the test." Try it; many students have found that it works because it reduces "test anxiety."
40%of students nationwide answered this question correctly.
View Topics
3203026303
Confidence: Just a guess
Stats
Issue with this question?
3.
The nurse has administered an immunosuppressant that acts on T helper cells to prevent production and release of IL-2 and γ-interferon to a client. Which side effects should the nurse monitor for in this client? Select all that apply.
Correct 1
Hirsutism
Correct 2
Gingival hyperplasia
3
Peptic ulcer
Correct 4
Nephrotoxicity
Incorrect5
Bone marrow suppression
Immunosuppressants that belong to calcineurin inhibitors may inhibit T helper cells to prevent production and release of IL-2 and γ-interferon. Hirsutism, gingival hyperplasia, and nephrotoxicity may occur as side effects of calcineurin inhibitors. Peptic ulcer may occur when corticosteroids are injected for immunosuppressant therapy. These drugs do not cause bone marrow suppression or alterations of the normal inflammatoryresponse. Bone marrow suppression may occur when cytotoxic drugs are administered.
0%of students nationwide answered this question correctly.
View Topics
3156442187
Confidence: Pretty sure
Stats
Issue with this question?
5.
What assessment findings indicate that an older client is at risk for developing an infection? Select all that apply.
Correct 1
Thin skin
Correct 2
Weak cough
Correct 3
Sluggish bowel sounds
4
Male-pattern baldness
Correct 5
Indwelling urinary catheter
Thin skin indicates a loss of protection by the integumentary system. A weak cough indicates a loss of protection by the respiratory system. Sluggish bowel sounds indicate a loss of protection by the gastrointestinal system. An indwelling urinary catheter is an invasive device which can introduce microorganisms into the client's system. Male-pattern baldness does not indicate a loss of protection by a body system.
10%of students nationwide answered this question correctly.
View Topics
3203097024
Confidence: Nailed it
Stats
Issue with this question?
8.
Which drugs are used for the treatment of clients with rheumatoid arthritis that inhibit tumor necrosis factor-A? Select all that apply.
Incorrect1
Anakinra
Correct 2
Infliximab
Incorrect3
Abatacept
Correct 4
Etanercept
Correct 5
Golimumab
Biological response modifiers (BRMs) are the substances that modify immune responses by either enhancing an immune response or suppressing it. Infliximab, etanercept, and golimumab are BRMs used in the treatment of rheumatoid arthritis that inhibit tumor necrosis factor (TNF)-A. Anakinra is aninterleukin-1 receptor antagonist used in the treatment of rheumatoid arthritis. Abatacept is a selective T-lymphocyteco-stimulator modulator (T-cell inhibitor) used in the treatment of rheumatoid arthritis.
0%of students nationwide answered this question correctly.
View Topics
3202830473
Confidence: Pretty sure
Stats
Issue with this question?
10.
A registered nurse is evaluating the actions of a nursing student who is injecting an allergen in a client having a severe anaphylactic reaction to insect venom. Which action of the nursing student requires correction?
1
Rotating the sites for each injection
2
Aspirating for blood before giving the injection
Correct3
Injecting in an extremity close to a joint
Incorrect4
Observing the client for 20 minutes after an injection
The allergen extract should always be administered in an extremity away from a joint so that a tourniquet can be applied for a severe reaction. The injection sites should be rotated for each injection to prevent skin damage. Aspirating for blood before giving an injection should always be done to ensure that the allergen extract is not injected into a blood vessel. Systemic reactions are likely to occur immediately. Therefore the client should be observed for 20 minutes after the injection.

Test-Taking Tip: Understand that the nurse is injecting allergen. The nursing intervention should result in the positive outcome towards the health of the client.
50%of students nationwide answered this question correctly.
View Topics
EAQ Home
Class Tools
Progress
Assignments
Grades
Custom Quiz
Elsevier Evolve Credits

...

EAQ TESTING Q & A - Subjecto.com

EAQ TESTING Q & A

Your page rank:

Total word count: 29838
Pages: 109

Calculate the Price

- -
275 words
Looking for Expert Opinion?
Let us have a look at your work and suggest how to improve it!
Get a Consultant

A client is diagnosed with condyloma acuminatum. Which finding in the client supports the diagnosis?

1
Moist, fleshy projections on the penis
2
Pus-filled ulcers on the penis
3
Swollen penis with tight foreskin
4
Macules on the penis

1 Moist, fleshy projections on the penis with single or multiple projections is a clinical manifestation of condyloma acuminatum. Macules on the penis or scrotum are clinical manifestations of penile erythema. Chancroid is manifested by pus-filled ulcers on the penis. A swollen penis with tight foreskin is a clinical manifestation of paraphimosis

A nurse is caring for a client with a diagnosis of cancer of the prostate. The nurse should teach the client that which serum level will be monitored throughout the course of the disease?

1
Albumin
2
Prostate-specific antigen (PSA)
3
Blood urea nitrogen (BUN)
4
Creatinine

.2..The PSA is an indication of cancer of the prostate; the higher the level, the greater the tumor burden. Albumin is a protein that is an indicator of nutritional and fluid status. Increased creatinine or BUN levels may be caused by impaired renal function as a result of blockage by an enlarged prostate but do not indicate that metastasis has occurred.

The nurse is caring for a client with ureteral colic. To prevent the development of renal calculi in the future, which strategy should be included in the client’s plan of care?

1
Excluding milk products from the diet
2
Interventions to decrease the serum creatinine level
3
Instructing the client to drink 8 to 10 glasses of water daily
4
A urinary output goal of 2000 mL per 24 hours

.3..Increasing fluid intake [1] [2] dilutes the urine, and crystals are less likely to coalesce and form calculi. An elevated serum creatinine has no relationship to the formation of renal calculi. Calcium restriction is necessary only if calculi have a calcium phosphate basis. Producing only 2000 mL of urine per 24 hours is inadequate. STUDY TIP: Answer every question. A question without an answer is the same as a wrong answer. Go ahead and guess. You have studied for the test and you know the material well. You are not making a random guess based on no information. You are guessing based on what you have learned and your best assessment of the question.

When assessing a client during peritoneal dialysis, a nurse observes that drainage of the dialysate from the peritoneal cavity has ceased before the required volume has returned. What should the nurse instruct the client to do?

1
Turn from side to side
2
Deep breathe and cough
3
Drink a glass of water
4
Rotate the catheter periodically

1…Turning from side to side will change the position of the catheter, thereby freeing the drainage holes of the tubing, which may be obstructed. Drinking a glass of water and deep breathing and coughing do not influence drainage of dialysate from the peritoneal cavity. The position of the catheter should be changed only by the primary healthcare provider.

A nurse is teaching a birthing/prenatal class about breast-feeding. Which hormone stimulates the production of milk during lactation?

1
Progesterone

2
Prolactin

3
Inhibin

4
Estrogen

2…Prolactin is the hormone that initiates and produces milk during lactation. Inhibin prevents the secretions of follicle stimulating hormone and gonadotropin releasing hormone. Estrogen and progesterone are the sex hormones produced by the ovaries.

A client is diagnosed with condyloma acuminatum. Which finding in the client supports the diagnosis?

1
Moist, fleshy projections on the penis

2
Pus-filled ulcers on the penis

3
Swollen penis with tight foreskin

4
Macules on the penis

1…Moist, fleshy projections on the penis with single or multiple projections is a clinical manifestation of condyloma acuminatum. Macules on the penis or scrotum are clinical manifestations of penile erythema. Chancroid is manifested by pus-filled ulcers on the penis. A swollen penis with tight foreskin is a clinical manifestation of paraphimosis.

Which statement regarding erythropoietin is true?

1
An erythropoietin deficiency is associated with renal failure.

2
Erythropoietin is released only when there is adequate blood flow.

3
An erythropoietin deficiency causes diabetes.

4
Erythropoietin is released by the pancreas.

…1 correct Erythropoietin is produced by the kidneys; its deficiency occurs in renal failure. Erythropoietin is released by the kidneys, not the pancreas. Erythropoietin deficiency causes anemia. Erythropoietin is secreted in response to hypoxia, which results in decreased oxygenated blood flow to the tissues.

Which drug prescribed to a client with a urinary tract infection (UTI) turns urine reddish-orange in color?

1
Nitrofurantoin

2
Ciprofloxacin

3
Phenazopyridine

4
Amoxicillin

…3…Phenazopyridine is a topical anesthetic that is used to treat pain or burning sensation associated with urination. It also imparts a characteristic orange or red color to urine. Amoxicillin is a penicillin form that could cause pseudomembranous colitis as a complication; it is not associated with reddish-orange colored urine. Ciprofloxacin is a quinolone antibiotic used for treating UTIs and can cause serious cardiac dysrhythmias and sunburns. It is not, however, responsible for reddish-orange colored urine. Nitrofurantoin is an antimicrobial medication prescribed for UTIs. This drug may affect the kidneys but is not associated with reddish-orange colored urine.

A client arrives at a health clinic reporting hematuria, frequency, urgency, and pain on urination. Which diagnosis will the nurse most likely observe written in the client’s medical record?

1
Pyelonephritis

2
Nephrotic syndrome

3
Cystitis

4
Chronic glomerulonephritis

…3…Cystitis is an inflammation of the bladder that causes frequency and urgency of urination, pain on micturition, and hematuria. Chronic glomerulonephritis is a disease of the kidney that is associated with manifestations of systemic circulatory overload. Nephrotic syndrome is a condition of increased glomerular permeability characterized by severe proteinuria. Pyelonephritis is a diffuse, pyogenic infection of the pelvis and parenchyma of the kidney that causes flank pain, chills, fever, and weakness.

The nurse providing postoperative care for a client who had kidney surgery reviews the client’s urinalysis results. Which urinary finding should the nurse conclude needs to be reported to the primary healthcare provider?

1
Acidic pH

2
Glucose negative

3
Presence of large proteins

4
Bacteria negative

…3..The glomeruli are not permeable to large proteins such as albumin or red blood cells (RBCs), and it is abnormal if albumin or RBCs are identified in the urine; their presence should be reported. The urine can be acidic; normal pH is 4.0 to 8.0. Glucose and bacteria should be negative; these are normal findings.

What is the function of the structure labeled in the given figure?
3204932528

1
Holds the fetus

2
Secretes ovum

3
Serves as entry to the sperm

4
Massages the ovaries

…4…The structure labeled in the figure represents the fallopian tubes, fingerlike projections that massage the ovaries to facilitate ovum extraction. The ovaries produce ovum. The uterus accommodates the fetus. The cervix serves as an entry to the sperm and is also involved in expulsion of menses.

Tests reveal that a client has phosphatic renal calculi. The nurse teaches the client that the diet may include which food item?

1
Chocolate

2
Apples

3
Cheddar cheese

4
Rye bread

Test-Taking Tip: If you are unable to answer a multiple-choice question immediately, eliminate the alternatives that you know are incorrect and proceed from that point. The same goes for a multiple-response question that requires you to choose two or more of the given alternatives. If a fill-in-the-blank question poses a problem, read the situation and essential information carefully and then formulate your response.

…Apples are low in phosphate; fresh fruit is low in phosphorus. Chocolate contains more phosphate than apples. Rye bread contains more phosphate than apples. Cheese is made with milk, which contains phosphate and should be avoided. Dairy products are high in phosphorus.

A nurse educates the client about the relationship between the kidneys and blood pressure. Which term should the nurse use to describe the part of the kidney that senses changes in blood pressure?

1
Macula densa

2
Calices

3
Glomerulus

4
Juxtaglomerular cells

…1..The macula densa, a part of the distal convoluted tubule, consists of cells that sense changes in the volume and pressure of blood. Calices are cup-like structures, present at the end of each papilla that collect urine. The glomerulus is the initial part of the nephron, which filters blood to make urine. Juxtaglomerular cells secrete renin. Renin is produced when sensing cells in the macula densa sense changes in blood volume and pressure.

The urinalysis report of a client reveals cloudy urine. What does a nurse infer from the client’s report?

1
The client has a biliary obstruction.

2
The client has diabetic ketoacidosis.

3
The client has been on a starvation diet.

4
The client has a urinary infection.

4 The urine becomes cloudy when an infection is present due to the presence of leukocytes. Therefore the nurse concludes that the client has a urinary infection. In cases of biliary obstruction, the urine contains bilirubin. The presence of ketones in the urine indicates diabetic ketoacidosis or prolonged starvation

A client is scheduled to have an indwelling urinary catheter inserted before abdominal surgery. The nurse should insert the catheter in what location in the illustration?
3142352560

1 d
2 c
3 a
4 b

Correct 4 b Option B is the urethral orifice, which anatomically is between the clitoris and the vagina; it is the opening into the urethra, the tubular structure that drains urine from the bladder. Option A is the clitoris, which is situated beneath the anterior commissure, partially hidden between the anterior extremities of the labia minora. Option C is the opening of the vagina; it is the part of the female genitalia that forms a canal from the vaginal orifice through the vestibule to the uterine cervix. Option D is the anus; it is the terminal end of the anal canal that is connected to the rectum; the rectum is a portion of the large intestine that is between the anal canal and the descending sigmoid colon. Test-Taking Tip: Practicing a few relaxation techniques may prove helpful on the day of an examination. Relaxation techniques such as deep breathing, imagery, head rolling, shoulder shrugging, rotating and stretching of the neck, leg lifts, and heel lifts with feet flat on the floor can effectively reduce tension while causing little or no distraction to those around you. It is recommended that you practice one or two of these techniques intermittently to avoid becoming tense. The more anxious and tense you become, the longer it will take you to relax.

A client is admitted to the hospital with severe flank pain, nausea, and hematuria caused by a ureteral calculus. What should be the nurse’s initial intervention?

1
Obtain a urine specimen for culture.

2
Administer the prescribed analgesic.

3
increase oral fluid intake.

4
Strain all urine output.

2…Pain of renal colic may be excruciating; unless relief is obtained, the client will be unable to cooperate with other therapy. Urine can be saved and strained after the client’s priority needs are met. Increasing fluid intake may or may not be helpful. If the stone is large the fluid can build up, leading to hydronephrosis; however, if the stone is small, fluids may help flush the stone. Although a culture generally is prescribed, this is not the priority when a client has severe pain.

Which retrograde procedure involves the examination of the ureters and the renal pelvises?

1
Urethrogram
2
Cystogram
3
Pyelogram
4
Voiding cystourethrogram

3… A pyelogram is a retrograde examination of the ureters and the pelvis of both kidneys. A cystogram is a retrograde examination of the bladder. An urethrogram is a retrograde examination of the urethra. A voiding cystourethrogram is used to determine whether urine is flowing backward into the urethra.

Which parts of the nephron are the sites for the regulation of water balance? Select all that apply.

1
Loop of Henle

2
Descending limb (DL)

3
Proximal convoluted tubule (PCT)

4
Bowman capsule (BC)

5
Glomerulus

1, 2 A loop of Henle is a part of a nephron that continues from the proximal convoluted tubule (PCT). It is permeable to water, sodium chloride, and urea and is a site for the regulation of water balance. The descending limb (DL) continues from the loop of Henle. It is permeable to water, sodium chloride, and urea and is a site for the regulation of water balance. The glomerulus is a site of glomerular filtration. The Bowman capsule (BC) is a site of the collection of glomerular filtrate. The proximal convoluted tubule (PCT) is a site for the reabsorption of sodium, chloride, glucose, water, amino acids, potassium, and calcium.

A nurse evaluates that a client with chronic kidney disease understands an adequate source of high biologic-value (HBV) protein when the client selects which food from the menu?

1
Apple juice

2
Raw carrots

3
Cottage cheese

4
whole wheat bread

3… Cottage cheese contains more protein than the other choices. Apple juice is a source of vitamins A and C, not protein. Raw carrots are a carbohydrate source and contain beta-carotene. Whole wheat bread is a source of carbohydrates and fiber. Test-Taking Tip: Come to your test prep with a positive attitude about yourself, your nursing knowledge, and your test-taking abilities. A positive attitude is achieved through self-confidence gained by effective study. This means (a) answering questions (assessment), (b) organizing study time (planning), (c) reading and further study (implementation), and (d) answering questions (evaluation).

The nurse observes a client with kidney failure has increased rate and depth of breathing. Which laboratory parameter does the nurse suspect is associated with this client’s condition?

1
Potassium 8 mEq/L

2
Phosphorous 7 mg/dL

3
Hemoglobin 10 g/dL

4
Bicarbonate 15 mEq/L

4 An increased rate and depth of breathing is called Kussmaul respiration and occurs due to metabolic acidosis in clients with kidney disease. Serum bicarbonate level decreases in metabolic acidosis. The normal range of serum bicarbonate is 23-30 mEq/L. Therefore the bicarbonate value of 15 mEq/L is associated with Kussmaul respirations in the client. The normal serum potassium is 3.5-5 mEq/L. Therefore a potassium level of 8 mEq/L indicates hyperkalemia and is associated with changes in cardiac rate and rhythm. The normal range of hemoglobin is 12-16 g/dL in females and 14-18 g/dL in males. Therefore a Hgb of 10 g/dL indicates anemia; this is associated with fatigue, pallor, and shortness of breath. The normal range of serum phosphorous is 3-4.5 mg/dL. Therefore a phosphorous value of 7 mg/dL indicates hyperphosphatemia, which is associated with hypocalcemia and demineralization of bone.

Which part of the kidney produces the hormone bradykinin?

1
Juxtaglomerular cells of the arterioles

2
Kidney tissues

3
Kidney parenchyma

4
Renin-producing granular cells

1 The juxtaglomerular cells of the arterioles produce the hormone bradykinin, which increases blood flow and vascular permeability. The kidney tissues produce prostaglandins that regulate internal blood flow by vasodilation or vasoconstriction. The kidney parenchyma produces erythropoietin that stimulates the bone marrow to make red blood cells. The renin-producing granular cells produce the renin hormone that raises blood pressure as a result of angiotensin and aldosterone secretion.

The nurse is performing a physical examination of a client by placing the left hand on the back and supporting the client’s right side between the rib cage and the iliac crest. Which physical assessment maneuver is the nurse performing on this client?

1
Palpation

2
Percussion

3
Auscultation

4
Inspection

1 The physical assessment involves inspection, palpation, percussion, and auscultation. During palpation of the right kidney, the nurse places the left hand behind and supports the client’s right side between the ribcage and the iliac crest. During an inspection, the nurse assesses the client for changes in skin, abdomen, weight, face, and extremities. During percussion, the nurse strikes the fist of one hand against the dorsal surface of the other hand, which is placed flat along the post costovertebral angle (CVA) margin. While performing auscultation, the nurse uses the bell of the stethoscope over both CVAs and in the upper abdominal quadrants.

A client who has been told she needs a hysterectomy for cervical cancer is upset about being unable to have a third child. Which action should the nurse take next?

1
Emphasize that she does have two children already.

2
Encourage her to focus on her own recovery.

3
Ensure that other treatment options for her will be explored.

4
Evaluate her willingness to pursue adoption.

3 Although a hysterectomy may be performed, conservative management may include cervical conization [1] [2] [3] and laser treatment that do not preclude future pregnancies; clients have a right to be informed by their primary healthcare provider of all treatment options. Willingness to pursue adoption currently is not the issue for this client. Encouraging her to focus on her own recovery and emphasizing that she does have two children already negate the client’s feelings.

A client is admitted to the hospital from the emergency department with a diagnosis of urolithiasis. The nurse reviews the client’s clinical record and performs an admission assessment. Which is the priority nursing action?

1
Collect a urine specimen for culture and sensitivity.

2
Administer the prescribed morphine.

3
Strain the client’s urine.

4
Place the client in the high-Fowler position.

2 Pain relief is the priority. Clients report that ureteral colic is excruciatingly painful. Once pain is under control and the client is comfortable, other medical and nursing interventions can be implemented. Although straining all urine is required, pain relief is the priority. Once the client is medicated for pain, the urine that was set aside can be strained. The high-Fowler position is not necessary. The client can be assisted to assume a position of comfort. The urine was sent for a culture and sensitivity in the emergency department.

A client is taught how to change the dressing and how to care for a recently inserted nephrostomy tube. On the day of discharge the client states, "I hope I can handle all this at home; it’s a lot to remember." Which is the best response by the nurse?

1
"Oh, a family member can do it for you."

2
"Perhaps you can stay in the hospital another day."

3
"I’m sure you can do it."

4
"You seem to be nervous about going home."

4 4 The response "You seem to be nervous about going home" is the best reply. Reflection conveys acceptance and encourages further communication. The response "I’m sure you can do it" is false reassurance that does not help to reduce anxiety. The response "Oh, a family member can do it for you" provides false reassurance and removes the focus from the client’s needs. The response "Perhaps you can stay in the hospital another day" is unrealistic, and it is too late to suggest this

A client with ascites has a paracentesis, and 1500 mL of fluid is removed. For which immediate response is it most important for the nurse to monitor?

1
Respiratory congestion

2
Increase in temperature

3
Rapid, thready pulse

4
Decreased peristalsis

…3 Fluid shifts from the intravascular compartment into the abdominal cavity, causing hypovolemia. A rapid, thready pulse [1] [2], which is indicative of shock, is a compensatory response to this shift. Decreased peristalsis is not likely to occur in the immediate period. After a paracentesis, intravascular fluid shifts into the abdominal cavity, not into the lungs. Increase in temperature is not the priority; body temperature usually is not affected immediately; an infection will take several days

The nurse is performing an assessment of a client’s reproductive system. Which action should the nurse take?

1
Maintain friendly demeanor with the client during assessment

2
Ask about sexual practices at the beginning of assessment

3
Ask about menstrual history at the beginning of assessment

4
Maintain gender-specific terms while questioning during assessment

…3 It is necessary to gather health information as part of an assessment of the reproductive system. The nurse should always start the questioning with minimally sensitive information such as menstrual history. This will help the client adjust gradually. The nurse should maintain a professional demeanor while assessing or taking a reproductive health history. Sensitive information, such as client’s sexual practices, should be asked after basic and less-sensitive topics. The nurse should make use of gender-neutral terms while questioning the client regarding their sexual partners.

Which hormone is crucial for ovulation and complete maturation of a client’s ovarian follicles?

Incorrect 1

Gonadotropin releasing hormone

Correct 2

Luteinizing hormone

3

Follicle stimulating hormone

4

Human chorionic gonadotropin hormone

Ovulation and complete maturation of ovarian follicles can only take place in the presence of luteinizing hormone. However, follicle stimulating hormone initiates maturation of the follicles. Gonadotropin releasing hormone stimulates the pituitary gland to release follicle stimulating hormone and luteinizing hormone. Human chorionic gonadotropin hormone is released after implantation and is responsible for secretion of progesterone and estrogen during pregnancy.

Test-Taking Tip: You have at least a 25% chance of selecting the correct response in multiple-choice items. If you are uncertain about a question, eliminate the choices that you believe are wrong and then call on your knowledge, skills, and abilities to choose from the remaining responses.

A female client has a history of recurrent urinary tract infections. What should the nurse include in the teaching plan when educating the client about health practices that may help decrease future urinary tract infections?

1

"Void at least every 6 hours."

2

"Wipe from back to front after toileting."

Correct 3

"Wear cotton underpants."

4

"Increase foods containing alkaline ash in the diet."

Cotton allows air to circulate and does not retain moisture the way synthetic fabrics do; microorganisms multiply in warm, moist environments. Voiding frequently helps to flush ascending microorganisms from the bladder, thereby reducing the risk for urinary tract infections; holding urine for 6 hours can lead to urinary tract infections. Foods high in acid, not alkaline, ash help to acidify urine; this urine is less likely to support bacterial growth. Alkaline urine promotes bacterial growth. Wiping from back to front after toileting may transfer bacteria from the perianal area toward the urinary meatus, which will increase the risk for urinary tract infection.

A nurse is providing client teaching to a woman who has recurrent urinary tract infections. Which information should the nurse include concerning the reason why women are more susceptible to urinary tract infections than men?

Correct 1

The length of the urethra

2

The continuity of mucous membranes

3

Inadequate fluid intake

4

Poor hygienic practices

The length of the urethra is shorter in women than in men; therefore, microorganisms have a shorter distance to travel to reach the bladder. The proximity of the meatus to the anus in women also increases the incidence of urinary tract infections. Fluid intake may or may not be adequate in both men and women and does not account for the difference. Hygienic practices can be inadequate in men or women. Mucous membranes are continuous in both men and women

The nurse provides discharge instructions to a male client who had an ureterolithotomy. The client has a history of recurrent urinary tract infections (UTIs). For which indicators of a UTI should the nurse instruct the client?

1

Pain radiating to the external genitalia

Correct 2

Urgency or frequency of urination

3

The inability to maintain an erection

4

An increase of ketones in the urine

Urgency or frequency of urination occur with a urinary tract infection [1] [2] because of bladder irritability; burning on urination and fever are additional signs of a UTI. Increase of ketones is associated with diabetes mellitus, starvation, or dehydration. The inability to maintain an erection is not related to a UTI. Pain radiating to the external genitalia is a symptom of a urinary calculus, not infection.

Radium inserted in the vagina of a client now is being removed. Which safety precaution should the nurse employ when assisting with the radium removal?

1

Document how long the radium was in place and when it was removed.

2

Clean the radium in ether or alcohol.

Correct 3

Ensure that long forceps are available for removing the radium.

4

Wear foil-lined rubber gloves while handling the radium.

Radium must be handled with long forceps because distance helps limit exposure. A nurse does not clean radium implants. Foil-lined rubber gloves do not provide adequate shielding from the gamma rays emitted by radium. The amount and duration of exposure are important in assessing the effect on the client; however, documentation will not affect safety during removal.

Which part of the nephron secretes creatinine required for elimination?

Correct 1

Proximal tubule

2

Glomerulus

Incorrect 3

Loop of Henle

4

Collecting duct

The proximal tubule of the nephron secretes creatinine and hydrogen ions. It also reabsorbs water and electrolytes. The glomerulus filters the blood selectively. The ascending loop of Henle reabsorbs sodium and chloride, whereas the descending loop of Henle concentrates the filtrate. The collecting duct reabsorbs water.

Which phase of the woman’s sexual response is characterized by elevation of the uterus?

1

Orgasmic phase

Correct 2

Plateau phase

3

Excitation phase

4

Resolution phase

The plateau phase occurs after the excitation phase, and excitation is maintained through the plateau phase, wherein the vagina expands and the uterus is elevated. Therefore elevation of the uterus is a characteristic of the plateau phase of a woman’s sexual response. The orgasmic phase is characterized by uterine and vaginal contractions. In the excitation phase, the clitoris is congested and vaginal lubrication increases. The resolution phase is characterized by returning to the preexisting state.

Which structures are included in the external genitalia in males? Select all that apply.

Correct 1

Scrotum

2

Urethra

Correct 3

Penis

4

Seminal vesicles

Incorrect 5

Testes

The male reproductive system is divided into primary reproductive organs and secondary reproductive organs. Secondary reproductive organs include ducts, sex glands, and external genitalia. The external genitalia consists of the penis and the scrotum. Testes are the primary reproductive organs. The urethra is the duct, and the seminal vesicles are sex glands.

A nurse writes a goal of preventing renal calculi in a care plan for a client with paraplegia. Which information most likely caused the nurse to write this goal?

Correct 1

Accelerated bone demineralization

Incorrect 2

High fluid intake

3

Inadequate kidney function

4

Increased intake of calcium

Calcium that has left the bones as a response to prolonged inactivity enters the blood and may precipitate in the kidneys, forming calculi. Increased fluid intake is helpful in preventing this condition by preventing urinary stasis. Calcium intake usually is limited to prevent the increased risk for calculi. Calculi may develop despite adequate kidney function; kidney function may be impaired by the presence of calculi and urinary tract infections associated with urinary stasis or repeated catheterizations.

A pathology report states that a client’s urinary calculus is composed of uric acid. Which food item should the nurse instruct the client to avoid?

Correct 1

Liver

2

Vegetables

3

Milk

4

Cheese

Uric acid stones are controlled by a low-purine diet. Foods high in purine, such as organ meats and extracts, should be avoided. Milk should be avoided with calcium, not uric acid, stones. Cheese or animal protein should be avoided with cystine, not uric acid, stones. Vegetables do not have to be avoided.

Test-Taking Tip: Avoid spending excessive time on any one question. Most questions can be answered in 1 to 2 minutes.

A client with chronic renal failure has been on hemodialysis for 2 years. The client communicates with the nurse in the dialysis unit in an angry, critical manner and is frequently noncompliant with medications and diet. The nurse can best intervene by first considering that the client’s behavior is most likely for which reason?

1

A constructive method of accepting reality

Correct 2

A defense against underlying depression and fear

3

An attempt to punish the nursing staff

4

An effort to maintain life and to live it as fully as possible

Both hostility and noncompliance are forms of anger that are associated with grieving. The client’s behavior is not a conscious attempt to hurt others but a way to relieve and reduce anxiety within the self. The client’s behavior is a self-destructive method of coping, which can result in death. The client’s behavior is an effort to maintain control over a situation that is really controlling the client; it is an unconscious method of coping, and noncompliance may be a form of denial.

Which statement indicates the nurse has a correct understanding about kidney ultrasonography?

1
Kidney ultrasonography primarily makes use of iodinated contrast dye.

2
Kidney ultrasonography gives three-dimensional information regarding kidneys.

3
Kidney ultrasonography makes use of sound waves and has minimal risk.

4
Kidney ultrasonography is performed on the client with an empty bladder.

3 Kidney ultrasonography is a minimal risk diagnostic procedure. Ultrasonography makes use of sound waves which, when reflected from internal organs of varying density, will produce the images of the kidneys, bladder, and associated structures on the display screen. While a dye can be used in computed tomography (CT), it is not the primary method. Generally kidney ultrasonography is performed on the client with full bladder. A CT gives three-dimensional information about the kidney and associated structures.

A student nurse is caring for a client with chronic kidney failure who is to be treated with continuous ambulatory peritoneal dialysis (CAPD). Which statement by the student nurse indicates to the primary nurse that the student nurse understands the purpose of this therapy?

Incorrect 1

"It decreases the need for immobility because it clears toxins in short and intermittent periods."

2

"It provides continuous contact of dialyzer and blood to clear toxins by ultrafiltration."

3

"It exchanges and cleanses blood by correction of electrolytes and excretion of creatinine."

Correct 4

"It uses the peritoneum as a semipermeable membrane to clear toxins by osmosis and diffusion."

Diffusion [1] [2] moves particles from an area of greater concentration to an area of lesser concentration; osmosis moves fluid from an area of lesser to an area of greater concentration of particles, thereby removing waste products into the dialysate, which is then drained from the abdomen. The principle of ultrafiltration involves a pressure gradient, which is associated with hemodialysis, not peritoneal dialysis. Peritoneal dialysis uses the peritoneal membrane to indirectly cleanse the blood. Dialysate does not clear toxins in a short time; exchanges may occur four or five times a day.

Which statement is true regarding the functions of kidney hormones?

1

Prostaglandin increases blood flow and vascular permeability.

2

Erythropoietin promotes the absorption of calcium in the gastrointestinal tract (GI) tract.

3

Bradykinin regulates intrarenal blood flow via vasodilation or vasoconstriction.

Correct 4

Renin raises blood pressure because of angiotensin and aldosterone secretion.

Renin is a kidney hormone that raises blood pressure as a result of angiotensin and aldosterone secretion. Prostaglandin is a kidney hormone that regulates intrarenal blood flow via vasodilation or vasoconstriction. Bradykinin is a kidney hormone that increases blood flow and vascular permeability. Erythropoietin is a kidney hormone that stimulates the bone marrow to make red blood cells.

After prostate surgery a client’s indwelling catheter and continuous bladder irrigation (CBI) are to be removed. The nurse discusses with the client the procedure and what to expect after the removal. Which statement by the client indicates teaching by the nurse is understood?

Correct 1

"I probably will experience some burning on urination."

2

"I probably will be unable to urinate."

3

"I probably will produce dark red urine."

4

"I probably will have dilute urine."

Because of the trauma to the mucous membranes of the urinary tract, burning on urination is an expected response that should subside gradually. The urine should no longer be dilute after the continuous bladder irrigation is discontinued and removed. However, the urine may have a slight pink tinge because of the trauma from the surgery and the presence of the catheter. An inability to urinate should not occur unless the indwelling catheter is removed too soon and there is still edema of the urethra. Production of dark red urine is a sign of hemorrhage, which should not occur.

A nurse is evaluating a client’s understanding of peritoneal dialysis. Which information in the client’s response indicates an understanding of the purpose of the procedure?

1
Providing fluid for intracellular spaces

2
Cleaning the peritoneal membrane

3
Reestablishing kidney function

4
Removing toxins in addition to other metabolic wastes

4 Peritoneal dialysis uses the peritoneum as a selectively permeable membrane for diffusion of toxins and wastes from the blood into the dialyzing solution. Peritoneal dialysis acts as a substitute for kidney function; it does not reestablish kidney function. The dialysate does not clean the peritoneal membrane; the semipermeable membrane allows toxins and wastes to pass into the dialysate within the abdominal cavity. Fluid in the abdominal cavity does not enter the intracellular compartment

A client who had a transurethral resection of the prostate is transferred to the postanesthesia care unit with an intravenous (IV) line and a urinary retention catheter. For which major complication is it most important for the nurse to assess during the immediate postoperative period?

Correct 1

Hemorrhage

2

Phlebitis

3

Leakage around the IV catheter

4

Sepsis

After transurethral surgery [1] [2], hemorrhage is common because of venous oozing and bleeding from many small arteries in the area. Sepsis is unusual, and if it occurs it will manifest later in the postoperative course. Phlebitis is assessed for, but it is not the most important complication. Hemorrhage is more important than phlebitis. Leaking around the IV catheter is not a major complication.

Test-Taking Tip: Work with a study group to create and take practice tests. Think of the kinds of questions you would ask if you were composing the test. Consider what would be a good question, what would be the right answer, and what would be other answers that would appear right but would in fact be incorrect.

Which hormone is crucial in maintaining the implanted egg at its site?

1

Testosterone

Correct 2

Progesterone

3

Estrogen

4

Inhibin

Progesterone is necessary to maintain an implanted egg. Inhibin regulates the release of follicle-stimulating hormone (FSH) and gonadotropin-releasing hormone (GnRH). Estrogen plays a vital role in the development and maintenance of secondary sexual characteristics. Testosterone is important for bone strength and development of muscle mass.

Which urinalysis finding indicates a urinary tract infection?

1
Presence of leukoesterase

2
Presence of crystals

3
Presence of ketones

4
Presence of bilirubin

1 Leukoesterases are released by white blood cells as a response to an infection or inflammation. Therefore, the presence of this chemical in urine indicates a urinary tract infection. The presence of crystals in the urine indicates that the specimen had been allowed to stand. Presence of bilirubin in the urine indicates anorexia nervosa, diabetic ketoacidosis, and prolonged fasting. The presence of ketones indicates diabetic ketoacidosis.

To help prevent a cycle of recurring urinary tract infections in a female client, which instruction should the nurse share?

1

"Take bubble baths regularly."

Correct 2

"Urinate as soon as possible after intercourse."

3

"Douche regularly with alkaline agents."

4

"Increase your daily intake of citrus juice."

Intercourse may cause urethral inflammation, increasing the risk of infection; voiding clears the urinary meatus and urethra of microorganisms. Most fruit juices, with the exception of cranberry juice, cause alkaline urine, which promotes bacterial growth. Douching is no longer recommended because it alters the vaginal flora. Bubble baths can promote urinary tract infections.

STUDY TIP: The old standbys of enough sleep and adequate nutritional intake also help keep excessive stress at bay. Although nursing students learn about the body’s energy needs in anatomy and physiology classes, somehow they tend to forget that glucose is necessary for brain cells to work. Skipping breakfast or lunch or surviving on junk food puts the brain at a disadvantage.

A nurse is counseling a woman who had recurrent urinary tract infections. Which factor should the nurse explain is the reason why women are at a greater risk than men for contracting a urinary tract infection?

1

Altered urinary pH

Correct 2

Proximity of the urethra to the anus

3

Hormonal secretions

4

Juxtaposition of the bladder

Because a woman’s urethra is closer to the anus than a man’s, it is at greater risk for becoming contaminated. Urinary pH is within the same range in both men and women. Hormonal secretions have no effect on the development of bladder infections. The position of the bladder is the same in men and women.

A client in a nursing home is diagnosed with urethritis. What should the nurse plan to do before initiating antibiotic therapy prescribed by the primary healthcare provider?

Correct 1

Obtain a urine specimen for culture and sensitivity.

Incorrect 2

Teach the client how to perform perineal care.

3

Prepare for urinary catheterization.

4

Start a 24-hour urine collection.

The causative organism should be isolated before starting antibiotic therapy; a culture and sensitivity [1] [2] should be obtained before starting the antibiotic. A 24-hour urine test will not determine the infective organism causing the problem. Catheterization is not a routine intervention for urethritis. Although client teaching is important, it is not the priority at this time.

STUDY TIP: Determine whether you are a "lark" or an "owl." Larks, day people, do best getting up early and studying during daylight hours. Owls, night people, are more alert after dark and can remain up late at night studying, catching up on needed sleep during daylight hours. It is better to work with natural biorhythms than to try to conform to an arbitrary schedule. You will absorb material more quickly and retain it better if you use your most alert periods of each day for study. Of course, it is necessary to work around class and clinical schedules. Owls should attempt to register in afternoon or evening lectures and clinical sections; larks do better with morning lectures and day clinical sections.

Which hormone influences kidney function?

Incorrect 1

Erythropoietin

Correct 2

Aldosterone

3

Renin

4

Bradykinin

Released from the adrenal cortex, aldosterone influences kidney function. Renin, bradykinin, and erythropoietin are kidney hormones.

Which client’s urine specific gravity level is abnormal?

1

1.028

2

1.006

Correct 3

1.041

4

1.012

The normal specific gravity of urine lies between 1.005 and 1.030. A specific gravity value of 1.041 is higher than the normal range; therefore, it’s abnormal. The specific gravity values of urine such as 1.006, 1.012, and 1.028 lie in the normal range.

Which diagnostic tests are used to measure the kidney size of a client with kidney dysfunction? Select all that apply.

Correct 1

Computed tomography (CT)

Incorrect 2

Cystography

Correct 3

Radiography

Incorrect 4

Cystoscopy

Incorrect 5

Cystourethrography

A radiography and a computed tomography (CT) are diagnostic tests used to measure kidney size in clients with kidney dysfunction. A cystoscopy is used to identify abnormalities of the bladder wall in clients with kidney dysfunction. A cystography and a cystourethrography are used to examine the structure of the urethra and to detect backward flow of urine

A client who is recovering from deep partial-thickness burns develops chills, fever, flank pain, and malaise. The primary healthcare provider makes a tentative diagnosis of urinary tract infection. Which diagnostic tests should the nurse expect the primary healthcare provider to prescribe to confirm this diagnosis?

1

Creatinine clearance and albumin/globulin (A/G) ratio

Correct 2

Urinalysis and urine culture and sensitivity

3

Cystoscopy and bilirubin level

4

Specific gravity and pH of the urine

The client’s manifestations may indicate a urinary tract infection; a culture of the urine will identify the microorganism, and sensitivity will identify the most appropriate antibiotic. A cystoscopy is too invasive as a screening procedure; altered bilirubin results indicate liver or biliary problems, not urinary signs and symptoms. Creatinine clearance reflects renal function; A/G ratio reflects liver function. Although an increased urine specific gravity may indicate red blood cells (RBCs), white blood cells (WBCs), or casts in the urine, which are associated with urinary tract infection, it will not identify the causative organism.

A male client has discharge from the penis. Gonorrhea is suspected. To obtain a specimen for a culture, what should the nurse do?

1

Swab the discharge when it appears on the prepuce.

2

Instruct the client to provide a semen specimen.

3

Instruct the client how to obtain a clean catch specimen of urine.

Correct 4

Swab the drainage directly from the urethra to obtain a specimen.

Swabbing the drainage directly from the urethra obtains a specimen uncontaminated by environmental organisms. Instructing the client to provide a semen specimen is not as accurate as obtaining the purulent discharge from the site of origin. Swabbing the discharge when it appears on the prepuce will contaminate the specimen with organisms external to the body. Teaching the client how to obtain a clean catch specimen of urine will dilute and possibly contaminate the specimen.

What instruction regarding sample collection should the nurse give a client who is ordered a clean-catch urine specimen?

1

Collect the last urine sample voided in the night

2

Keep the urine sample in dry warm area if delay is anticipated

Correct 3

Urinate small amount, stop flow, fill half of cup

4

Send the urine sample to the laboratory within 6 hours of collection

The nurse instructs the client to always collect the midstream urine to send as a test specimen. The client should be instructed to cleanse the perineum with the wipe provided, urinate a small amount, and then stop the flow. The client should then position the specimen cup a few inches from the urethra and resume urination, filling the cup at least half way. The client is asked to collect the first sample voided in the morning because the urine is highly concentrated in the morning. Keeping the urine sample in the refrigerator helps reduce bacterial growth due to alkaline environment. The cells in the urine sample begin to break down in alkalinity, and therefore the client is instructed to send the sample to the laboratory as soon as collected.

The primary healthcare provider suspects pituitary gland dysfunction in a female client. Which diagnostic test would the primary healthcare provider suggest to the client?

Incorrect 1

Estradiol test

2

Sims-Huhner test

Correct 3

Prolactin test

4

Papanicolaou (Pap) test

A prolactin test is used to detect pituitary gland dysfunction that causes amenorrhea. Therefore the primary healthcare provider would suggest that the client have a prolactin test to determine if the client does or does not have any pituitary gland dysfunction. Estradiol is tested to determine functioning of the ovaries. In men, the estradiol test is used to detect testicular tumors. The Sims-Huhner test is used to evaluate the hostility of the cervix for passage of sperm from the vagina into the uterus. The Papanicolaou (Pap) test detects malignancies, particularly cervical cancer.

The nurse is educating student nurses about the anatomy and physiology of the kidneys. What term does the nurse explain is used for the tip of the pyramid of a kidney?

Incorrect 1

Calyx

Correct 2

Papilla

3

Renal column

4

Renal pelvis

Pyramids are components of renal medulla, and the tip of each pyramid is called a papilla. A calyx is a structure that collects the urine at the end of each pyramid. The renal calices join together to form the renal pelvis. A renal column is a cortical tissue that separates the pyramids.

Which part of the female reproductive system produces testosterone in females?

Correct 1

Ovary

2

Ovarian follicle

3

Uterus

4

Fallopian tube

Testosterone is an androgen, and in females, androgens are produced by the ovaries and adrenal glands. The uterus holds the fetus during pregnancy. Fallopian tubes facilitate fertilization of oocyte and sperm. An ovarian follicle is a collection of oocytes in the ovary.

Which electrolyte deficiency triggers the secretion of renin?

1

Calcium

2

Potassium

Correct 3

Sodium

4

Chloride

Low sodium ion concentration causes decreased blood volume, thereby resulting in decreased perfusion. Decreased blood volume triggers the release of renin from the juxtaglomerular cells. Deficiencies of calcium, chloride, and potassium do not stimulate the secretion of renin.

A client is admitted to the hospital with a diagnosis of cancer of the liver with ascites and is scheduled for a paracentesis. Which nursing intervention is appropriate to include in the client’s plan of care?

1
Marking the anesthetic insertion site

2
Cleansing the intestinal tract

3
Discussing the operating room set-up

4
Having the client void before the procedure

4 Because the trocar is inserted below the umbilicus, having the client void decreases the danger of puncturing the bladder. Cleansing the intestinal tract is not necessary because the gastrointestinal tract is not involved in a paracentesis. The primary healthcare provider, not the nurse, uses a local anesthetic to block pain during the insertion of the aspirating needle; marking the site usually is not done. A paracentesis usually is performed in a treatment room or at the client’s bedside, not in the operating room.

A nurse is reviewing the clinical record of a client with a diagnosis of benign prostatic hyperplasia (BPH). Which test result will the nurse check to confirm the diagnosis?

Correct 1

Biopsy of prostatic tissue

2

Rectal examination

3

Serum phosphatase level

4

Massage of prostatic fluid

A definitive diagnosis of the cellular changes associated with benign prostatic hyperplasia [1] [2] (BPH) is made by biopsy, with subsequent microscopic evaluation. Palpation of the prostate gland through rectal examination is not a definitive diagnosis; this only reveals size and configuration of the prostate. The serum phosphatase level will provide information for prostatic cancer; a definitive diagnosis cannot be made with this test for BPH. A sample of prostatic fluid helps to diagnosis prostatitis.

A nurse is assessing the urine of a client with a urinary tract infection. For which characteristic should the nurse assess each specimen of urine?

1
Viscosity

2
Clarity

3
Specific gravity

4
Glucose level

Cloudy urine usually indicates drainage associated with infection. Viscosity is a characteristic that is not measurable in urine. Urinary glucose levels are not affected by urinary tract infections. Specific gravity yields information related to fluid balance.

2 Cloudy urine usually indicates drainage associated with infection. Viscosity is a characteristic that is not measurable in urine. Urinary glucose levels are not affected by urinary tract infections. Specific gravity yields information related to fluid balance.

What does the presence of ketones in the urine of a client with renal dysfunction indicate?

Correct 1

Anorexia nervosa

2

Cystitis

3

Heart failure

Incorrect 4

Urinary calculi

The body of a client with anorexia nervosa produces ketones as an alternate source of fuel for muscles and organs. Increased red blood cells (RBCs) in the urine indicate cystitis. Increased specific gravity of the urine indicates heart failure. The presence of casts in the urine indicates urinary calculi.

Which urodynamic study provides information on bladder capacity, bladder pressure, and voiding reflexes?

Correct 1

Cystometrography (CMG)

2

Renal arteriography

3

Radiography

Incorrect 4

Electromyography (EMG)

Cystometrography (CMG) is an urodynamic study that provides information on bladder capacity, bladder pressure, and voiding reflexes. Radiography is a diagnostic test for clients with disorders of kidney and urinary system to screen for the presence of two kidneys, to measure kidney size, and to detect gross obstruction in kidneys or urinary tract. Renal arteriography is a diagnostic study used to determine renal blood vessel size and abnormalities. Electromyography (EMG) is an urodynamic study used to test the strength of perineal muscles in voiding.

The nurse is educating new parents about circumcision. Which structure of the penis would this nurse tell the parents is removed during circumcision?

Correct 1

Prepuce

2

Vas deferens

3

Glans

4

Epididymis

Circumcision is a procedure that involves removal of the prepuce, a skin fold over the glans. The glans is the tip of the penis. The epididymis is the internal structure that promotes transportation of the sperm. The vas deferens carries the sperm from the epididymis to the ejaculatory duct.

A 55-year-old client reports cessation of menstrual periods. Which term best describes the client’s condition?

1

Menarche

Correct 2

Menopause

3

Amenorrhea

4

Dyspareunia

Cessation of menstruation is called menopause; this is an aging process and occurs due to functional decline of the ovaries. The first episode of menstrual bleeding is called menarche. Dyspareunia refers to painful sexual intercourse. The absence of menstruation is called amenorrhea.

A client has a kidney transplant. The nurse should monitor for which assessment findings associated with rejection of the transplant? Select all that apply.

Correct 1

Oliguria

2

Polydipsia

Correct 3

Fever

Correct 4

Weight gain

Incorrect 5

Jaundice

Fever is a characteristic of the systemic inflammatory response to the antigen (transplanted kidney). Oliguria or anuria occurs when the transplanted kidney is rejected and fails to function. Weight gain can occur from fluid retention when the transplanted kidney fails to function or as a result of steroid therapy; this response must be assessed further. Jaundice is unrelated to rejection. Polydipsia is associated with diabetes mellitus; it is not a clinical manifestation of rejection.

What is the cup-like structure that collects a client’s urine and is located at the end of each papilla?

Correct 1

Calyx

2

Capsule

3

Renal cortex

4

Renal columns

The calyx is a cup-like structure that collects urine and is located at the end of each papilla. The outer surface of the kidney consists of fibrous tissue and is called the capsule. The renal cortex is the outer tissue layer. The renal columns are the cortical tissue that dip down into the interior of the kidney and separate the pyramids.

Which is a primary glomerular disease?

Correct 1

Chronic glomerulonephritis

2

Diabetic glomerulopathy

3

Systemic lupus erythematosus (SLE)

4

Hemolytic-uremic syndrome

Chronic glomerulonephritis is a primary glomerular disease. Diabetic glomerulopathy, hemolytic-uremic syndrome, and systemic lupus erythematosus (SLE) are secondary glomerular diseases.

The nurse is caring for a client with ureteral colic. To prevent the development of renal calculi in the future, which strategy should be included in the client’s plan of care?
1
Excluding milk products from the diet
2
Interventions to decrease the serum creatinine level
Correct 3
Instructing the client to drink 8 to 10 glasses of water daily
4
A urinary output goal of 2000 mL per 24 hours

Correct 3 Increasing fluid intake [1] [2] dilutes the urine, and crystals are less likely to coalesce and form calculi. An elevated serum creatinine has no relationship to the formation of renal calculi. Calcium restriction is necessary only if calculi have a calcium phosphate basis. Producing only 2000 mL of urine per 24 hours is inadequate. STUDY TIP: Answer every question. A question without an answer is the same as a wrong answer. Go ahead and guess. You have studied for the test and you know the material well. You are not making a random guess based on no information. You are guessing based on what you have learned and your best assessment of the question

Which drug prescribed to a client with a urinary tract infection (UTI) turns urine reddish-orange in color?

1
Nitrofurantoin
2
Ciprofloxacin
3
Phenazopyridine
4
Amoxicillin

correct:3 Phenazopyridine is a topical anesthetic that is used to treat pain or burning sensation associated with urination. It also imparts a characteristic orange or red color to urine. Amoxicillin is a penicillin form that could cause pseudomembranous colitis as a complication; it is not associated with reddish-orange colored urine. Ciprofloxacin is a quinolone antibiotic used for treating UTIs and can cause serious cardiac dysrhythmias and sunburns. It is not, however, responsible for reddish-orange colored urine. Nitrofurantoin is an antimicrobial medication prescribed for UTIs. This drug may affect the kidneys but is not associated with reddish-orange colored urine.

When assessing a client during peritoneal dialysis, a nurse observes that drainage of the dialysate from the peritoneal cavity has ceased before the required volume has returned. What should the nurse instruct the client to do?
1
Turn from side to side
2
Deep breathe and cough
3
Drink a glass of water
4
Rotate the catheter periodically

CORRECT 1 Turning from side to side will change the position of the catheter, thereby freeing the drainage holes of the tubing, which may be obstructed. Drinking a glass of water and deep breathing and coughing do not influence drainage of dialysate from the peritoneal cavity. The position of the catheter should be changed only by the primary healthcare provider

A client arrives at a health clinic reporting hematuria, frequency, urgency, and pain on urination. Which diagnosis will the nurse most likely observe written in the client’s medical record?
1
Pyelonephritis
2
Nephrotic syndrome
3
Cystitis
4
Chronic glomerulonephritis

3 CORRECT Cystitis is an inflammation of the bladder that causes frequency and urgency of urination, pain on micturition, and hematuria. Chronic glomerulonephritis is a disease of the kidney that is associated with manifestations of systemic circulatory overload. Nephrotic syndrome is a condition of increased glomerular permeability characterized by severe proteinuria. Pyelonephritis is a diffuse, pyogenic infection of the pelvis and parenchyma of the kidney that causes flank pain, chills, fever, and weakness

The nurse providing postoperative care for a client who had kidney surgery reviews the client’s urinalysis results. Which urinary finding should the nurse conclude needs to be reported to the primary healthcare provider?
1
Acidic pH
2
Glucose negative
3
Presence of large proteins
4
Bacteria negative

3 The glomeruli are not permeable to large proteins such as albumin or red blood cells (RBCs), and it is abnormal if albumin or RBCs are identified in the urine; their presence should be reported. The urine can be acidic; normal pH is 4.0 to 8.0. Glucose and bacteria should be negative; these are normal findings.

A nurse is teaching a birthing/prenatal class about breast-feeding. Which hormone stimulates the production of milk during lactation?
1
Progesterone
2
Prolactin
3
Inhibin
4
Estrogen

answer 2 Prolactin is the hormone that initiates and produces milk during lactation. Inhibin prevents the secretions of follicle stimulating hormone and gonadotropin releasing hormone. Estrogen and progesterone are the sex hormones produced by the ovaries.

What is the function of the structure labeled in the given figure?

picture wont copy onto here….

1
Holds the fetus
2
Secretes ovum
3
Serves as entry to the sperm
4
Massages the ovaries

correct 4 The structure labeled in the figure represents the fallopian tubes, fingerlike projections that massage the ovaries to facilitate ovum extraction. The ovaries produce ovum. The uterus accommodates the fetus. The cervix serves as an entry to the sperm and is also involved in expulsion of menses.

Tests reveal that a client has phosphatic renal calculi. The nurse teaches the client that the diet may include which food item?
1
Chocolate
2
Apples
3
Cheddar cheese
4
Rye bread

answer: 2 Apples are low in phosphate; fresh fruit is low in phosphorus. Chocolate contains more phosphate than apples. Rye bread contains more phosphate than apples. Cheese is made with milk, which contains phosphate and should be avoided. Dairy products are high in phosphorus. Test-Taking Tip: If you are unable to answer a multiple-choice question immediately, eliminate the alternatives that you know are incorrect and proceed from that point. The same goes for a multiple-response question that requires you to choose two or more of the given alternatives. If a fill-in-the-blank question poses a problem, read the situation and essential information carefully and then formulate your response.

A nurse educates the client about the relationship between the kidneys and blood pressure. Which term should the nurse use to describe the part of the kidney that senses changes in blood pressure?

1
Macula densa
2
Calices
3
Glomerulus
4
Juxtaglomerular cells

correct 1 The macula densa, a part of the distal convoluted tubule, consists of cells that sense changes in the volume and pressure of blood. Calices are cup-like structures, present at the end of each papilla that collect urine. The glomerulus is the initial part of the nephron, which filters blood to make urine. Juxtaglomerular cells secrete renin. Renin is produced when sensing cells in the macula densa sense changes in blood volume and pressure.

The nurse is caring for a client in the postanesthesia care unit. The client had a suprapubic prostatectomy for cancer of the prostate and has a continuous bladder irrigation (CBI) in place. Which primary goal is the nurse trying to achieve with the CBI?
1
Provide continuous pressure on the prostatic fossa.
2
Stimulate continuous formation of urine.
3
Facilitate the measurement of urinary output.
4
Prevent the development of clots in the bladder.

4 A continuous flushing of the bladder dilutes the bloody urine and empties the bladder, preventing clots. Fluid instilled into the bladder does not affect kidney function. Urinary output can be measured regardless of the amount of fluid instilled. The urinary retention catheter is not designed to exert pressure on the prostatic fossa.

A client is diagnosed as having invasive cancer of the bladder, and brachytherapy is scheduled. What should the nurse expect the client to demonstrate that indicates success of this therapy?
1
Shrinkage of the tumor on scanning
2
Increase in pulse strength
3
Increase in the quantity of white blood cells (WBCs)
4
Decrease in urine output

1 Brachytherapy, in which isotope seeds are implanted in the tumor, interferes with cell multiplication, which should control the growth and metastasis of cancerous tumors. Radiation affects healthy as well as abnormal cells; urinary output will increase with successful therapy. With brachytherapy of the bladder, increase in pulse strength is not a sign of success. Bone marrow sites may be affected by radiation, resulting in a reduction of WBCs.

What is the concentration of estradiol in the blood during the follicular phase of the menstrual cycle?
1
159 pg/mL
2
165 pg/mL
3
171 pg/mL
4
130 pg/mL

4 In the follicular phase of the menstrual cycle, 20-150 pg/mL of estradiol is released. Therefore 130 pg/mL of estradiol would be its concentration during the follicular phase of the menstrual cycle. Concentrations of 159, 165, and 171 pg/mL are greater than the reference range.

A client with an indwelling catheter is prescribed a urinalysis test. Arrange the steps involved in the collection of the urine sample in correct order.

1.Clamp drainage tubing

2.Attach a sterile syringe

3.Aspirate the urine

4. Remove the clamp

In a client with an indwelling catheter, urine sample is collected by first applying a clamp, distal to the injection port, on to the drainage tubing. Then the injection port cap of the catheter drainage tubing is cleaned with alcohol. The next step is to attach a 5-mL sterile syringe into the port and aspirate the urine sample required. Finally the clamp is removed so that the drainage is resumed.

A nurse is notified that the latest potassium level for a client in acute kidney injury is 6.2 mEq (6.2 mmol/L). Which action should the nurse take first?
1
Call the laboratory to repeat the test.
2
Take vital signs and notify the primary healthcare provider.
3
Obtain an electrocardiogram (ECG) strip and obtain an antiarrhythmic medication.
4
Alert the cardiac arrest team.

2 Vital signs monitor the cardiopulmonary status; the primary healthcare provider must treat this hyperkalemia [1] [2] to prevent cardiac dysrhythmias. The cardiac arrest team responds to a cardiac arrest; there is no sign of arrest in this client. A repeat laboratory test will take time and probably reaffirm the original results; the client needs medical attention. Although obtaining an ECG strip is appropriate, obtaining an antiarrhythmic is premature; vital signs and medical attention is needed first.

A client is transferred to the postanesthesia care unit after undergoing a pyelolithotomy. The client’s urinary output is 50 mL/hr. What should the nurse do?
1
Record the output as an expected finding.
2
Milk the client’s nephrostomy tube.
3
Encourage the client to drink oral fluids.
4
Notify the primary healthcare provider.

1 An output of 50 mL/hr is adequate; when urine output drops below 20 to 30 mL/hr, it may indicate renal failure, and the primary healthcare provider should be notified. Encouraging the client to drink oral fluids is contraindicated; the client probably still will be under the influence of anesthesia, and the gag reflex may be depressed. Milking the client’s nephrostomy tube is unnecessary because the output is adequate.

How long will a client’s ovum stay viable after its release to get fertilized?
1
72 hours
2
78 hours
3
76 hours
4
74 hours

Ovum can be fertilized up to 72 hours after its release. The ovum disintegrates after 72 hours, and menstruation begins soon after. Therefore the ovum cannot be viable for 74, 76, or 78 hours, and fertilization will not occur.

Which hormone is released in response to low serum levels of calcium?
1
Atrial natriuretic peptide
2
Renin
3
Parathyroid hormone
4
Erythropoietin

3 If serum calcium levels decline, the parathyroid gland releases parathyroid hormone to maintain calcium homeostasis. Renin is a hormone released in response to decreased renal perfusion; this hormone is responsible for regulating blood pressure. Erythropoietin is released by the kidneys in response to poor blood flow to the kidneys; it stimulates the production of red blood cells. Atrial natriuretic peptide is produced by the right atrium of the heart in response to increased blood volume. This hormone then acts on the kidneys to promote sodium excretion, which decreases the blood volume.

Which test helps to identify fibroids, tumors, and fistulas while performing a reproductive tract examination?
1
Mammography
2
Computed tomography
3
Ultrasonography
4
Hysterosalpingography

4 A hysterosalpingogram is an X-ray used to evaluate tubal anatomy and patency and used to identify uterine problems such as fibroids, tumors, and fistulas. A mammography is an X-ray of the soft tissue of the breast. An ultrasonography (US) is a technique used to assess fibroids, cysts, and masses. Computer tomography is used to detect and evaluate masses and identify lymphatic enlargement from metastasis. Test-Taking Tip: Identifying content and what is being asked about that content is critical to your choosing the correct response. Be alert for words in the stem of the item that are the same or similar in nature to those in one or two of the options.

The nurse finds that a client with a urinary disorder has very pale-yellow-colored urine. What is the significance of this abnormal finding?
1
It indicates dilute urine.
2
It indicates concentrated urine.
3
It indicates the presence of myoglobin.
4
It indicates blood in the urine.

Dilute urine tends to appear very pale-yellow in color. Dark-red or brown color urine indicates the presence of blood in the urine. Dark-amber color urine indicates concentrated urine. Red color urine may indicate the presence of myoglobin.

What are the functions of antidiuretic hormone (ADH)? Select all that apply.
1
Promoting the reabsorption of sodium in the distal convoluted tubule (DCT)
2
Increasing tubular permeability to water
3
Increasing arteriole constriction
4
Controlling calcium balance
5
Stimulating the bone marrow to make red blood cells

2, 3 Antidiuretic hormone (ADH), also known as vasopressin, is a hormone released from the posterior pituitary gland. ADH increases arteriole constriction and tubular permeability to water. Calcium balance is controlled by blood levels of calcitonin and the parathyroid hormone (PTH). Erythropoietin stimulates the bone marrow to make red blood cells. Aldosterone promotes the reabsorption of sodium in the distal convoluted tubule (DCT).

The nurse is aware that the Cowper gland is also often referred to by which other term?
1
Skene gland
2
Prostate gland
3
Bartholin gland
4
Bulbourethral gland

4 Cowper glands are accessory glands of the male reproductive system; they are also referred to as the bulbourethral glands. Skene glands are a part of the female reproductive system. The prostate gland is also a gland of the male reproductive system. Bartholin glands are part of the female reproductive system.

The nurse is teaching a client receiving peritoneal dialysis about the reason dialysis solution is warmed before it is instilled into the peritoneal cavity. Which information will the nurse share with the client?
1
Because it adds extra warmth to the body because metabolic processes are disturbed
2
Because it helps prevent cardiac dysrhythmias by speeding up removal of excess potassium
3
Because it encourages removal of serum urea by preventing constriction of peritoneal blood vessels
4
Because it forces potassium back into the cells, thereby decreasing serum levels

3 Encouraging the removal of serum urea by preventing constriction of peritoneal blood vessels promotes vasodilation so that urea, a large-molecular substance, is shifted from the body into the dialyzing solution. Heat does not affect the shift of potassium into the cells. The removal of metabolic wastes is affected in kidney failure, not the metabolic processes. Heating dialysis solution does not affect cardiac dysrhythmias.

Which diagnostic procedure helps in the detection of uropathologic features in a client who has a urinary pouch or ileal conduit?
1
Loopogram
2
Cystogram
3
Computed tomography urogram
4
Urethrogram

1 Loopogram helps in the detection of uropathologic features in a client who has a urinary pouch or ileal conduit. Cystogram helps to visualize the bladder and evaluates vesicoureteral reflux. A computed tomography (CT) urogram provides excellent visualization of kidneys and kidney size can be evaluated. When urethral trauma is suspected, an urethrogram is done before catheterization.

What are the general manifestations associated with clients who have urinary system disorders? Select all that apply.
1
Nausea and vomiting
2
Facial edema
3
Excessive thirst
4
Elevated blood pressure
5
Stress incontinence

1,3,4 The general manifestations associated with urinary system disorders include excessive thirst, nausea and vomiting, and elevated blood pressure. The specific manifestations associated with urinary system disorders include facial edema and stress incontinence.

Which component of the client’s nephron acts as a receptor site for the antidiuretic hormone and regulates water balance?
1
Proximal convoluted tubule
2
Distal convoluted tubule
3
Bowman’s capsule
4
Collecting ducts

4 The collecting ducts regulate water balance and act as a receptor site for antidiuretic hormone. The Bowman’s capsule collects glomerular filtrate and funnels it into the tubule. The distal convoluted tubule acts as a site for additional water and electrolyte reabsorption. The proximal convoluted tubule is the site for reabsorption of sodium, chloride, water, and urea.

Which urinary diagnostic test does not require any dietary or activity restrictions for the client before or after the test?
1
Renal scan
2
Concentration test
3
Renal arteriogram
4
Renal biopsy

1 A renal scan does not require any dietary or activity restrictions. A renal biopsy requires bed rest for 24 hours after the procedure. A renal arteriogram requires the client to maintain bed rest with affected leg straight. A concentration test requires the client to fast after a given time in the evening.

A client reports to a health clinic because a sexual partner recently was diagnosed as having gonorrhea. The health history reveals that the client has engaged in receptive anal intercourse. What should the nurse assess for in this client?
1
Anal itching
2
Ribbon-shaped stools
3
Melena
4
Constipation

1 Anal itching and irritation can occur from having anal intercourse with a person infected with gonorrhea. Frank rectal bleeding, not upper gastrointestinal bleeding (melena), occurs. Painful defecation, not constipation, occurs. The shape of formed stool does not change; however, defection can be painful.

Which is an abnormal finding of the urinary system?
1
Pain in the flank region upon hitting
2
Presence of bowel sounds
3
Nonpalpable urinary bladder
4
Nonpalpable left kidney

1 Normally, a blow in the flank region should not elicit pain. Pain in the flank region upon hitting indicates kidney infection or polycystic kidney disease. But the client experiences pain when his/her flank area is hit; therefore, this is an abnormal finding. The left kidney is covered by the spleen and is not palpable, which is a normal finding. The client has bowel sounds. However, no alteration of bowel sounds is seen. Therefore it is a normal finding. The urinary bladder is not normally palpable, unless it is distended with urine.

A nurse is caring for a client who had a nephrectomy because of cancer of the kidney. Which factor will influence the client’s ability to deep breathe and cough postoperatively?
1
Pulmonary congestion from preoperative medications
2
Location of the surgical incision
3
Inflammatory process associated with surgery
4
Increased anxiety about the prognosis

2 The location of the surgical site in relation to the diaphragm increases incisional pain when deep breathing or coughing. Anxiety about the prognosis should not interfere with the ability to deep breathe and cough, especially when encouraged by the nurse. Inflammatory changes will cause discomfort in the area of any incision but are not necessarily the prime factor preventing deep breathing after a nephrectomy. The client will need to cough and deep breathe if there is congestion in the lungs.

A client scheduled for a hemicolectomy because of ulcerative colitis asks if having a hemicolectomy means wearing a pouch and having bowel movements in an abnormal way. Which is the best response by the nurse?
1
"Yes, but it will be temporary until the colitis is cured."
2
"No, that is necessary when a tumor is blocking the rectum."
3
"Yes, hemicolectomy is the same as a colostomy."
4
"No, only part of the colon is removed and the rest reattached."

4 Hemicolectomy [1] [2] is removal of part of the colon with an anastomosis between the ileum and transverse colon; a colostomy is not necessary. With a colostomy the intestine opens on the abdomen, whereas in a hemicolectomy a portion of the intestine is resected and the ends reconnected. "Yes, but it will be temporary until the colitis is cured" is the description of a temporary colostomy; a cure occurs only when the entire colon is removed. A colostomy is done for a variety of reasons other than a tumor; a colectomy with a colostomy is only one intervention that may be used to treat a tumor.

A nurse is caring for an older bedridden male client who is incontinent of urine. Which action should the nurse take first?
1
Offer the urinal regularly.
2
Insert an indwelling urinary catheter.
3
Apply incontinence pants.
4
Restrict fluid intake.

1 Offering the urinal is the first step. Retraining the bladder includes a routine pattern of attempts to void, which may increase bladder muscle tone and produce a conditioned response. Restricting fluid intake can result in dehydration and a urinary tract infection in an older client. Applying incontinence pants does not address the cause of the incontinence; also it promotes skin breakdown and can lower self-esteem. Inserting an indwelling urinary catheter increases the risk of a urinary tract infection. Also, it requires a primary healthcare provider’s prescription.

The nurse is performing bedside sonography for a female client who underwent a hysterectomy. Which nursing intervention needs correction?
1
Pointing the scan head so the ultrasound is projected towards the client’s coccyx
2
Placing the midline of the probe over the abdomen about 1.5 inches (3.8 cm) above the pubic bone
3
Placing an ultrasound gel pad right above the pubic bone
4
Using the female icon on the bladder scanner

4 Before performing a bedside sonography, the male or female icon on the scanner should be selected. The male icon should be selected for men and for women who have undergone a hysterectomy. An ultrasound gel pad should be placed right above the pubic bone. The scan head should be pointed in such a way that the ultrasound is projected towards the client’s coccyx. The midline of the probe should be placed over the abdomen about 1.5 inches (3.8 cm) above the pubic bone.

A client with acute kidney injury is to receive peritoneal dialysis and asks why the procedure is necessary. Which is the nurse’s best response?
1
"It speeds recovery because the kidneys are not responding to regulating hormones."
2
"It removes toxic chemicals from the body so you will not get worse."
3
"It prevents the development of serious heart problems."
4
"It helps perform some of the work usually done by the kidneys."

4 Dialysis removes chemicals, wastes, and fluids usually removed from the body by the kidneys. The mention of heart problems is a threatening response and may cause increased fear or anxiety. Stating that peritoneal dialysis "removes toxic chemicals from the body so you will not get worse" is threatening and can cause an increase in anxiety. Dialysis helps maintain fluid and electrolytes; the nephrons are damaged in acute kidney injury, so it may or may not speed recovery.

Which part of the reproductive system secretes androgens in female clients?
1
Ovarian follicle
2
Uterus
3
Ovaries
4
Fallopian tube

3 The ovaries and adrenal glands produce androgens in women. The fetus develops in the uterus during pregnancy. Fallopian tubes facilitate fertilization of oocyte and sperm. Ovarian follicle is a collection of oocytes in the ovary.

A male client reports dysuria, nocturia, and difficulty starting the urinary stream. A cystoscopy and biopsy of the prostate gland have been scheduled. After the procedure the client reports an inability to void. Which action should the nurse take?
1
Insert a urinary retention catheter.
2
Palpate above the pubic symphysis.
3
Assure the client that this is expected.
4
Limit oral fluids until the client voids.

2 A full bladder is palpable with urinary retention and distention, which are common problems after a cystoscopy because of urethral edema. More conservative nursing methods, such as running water or placing a warm cloth over the perineum, should be attempted to precipitate voiding; catheterization carries a risk of infection and is used as the last resort. Fluids dilute the urine and reduce the chance of infection after cystoscopy and should not be limited. Although urinary retention can occur, it is not expected; the nurse must assess the extent of bladder distention and discomfort.

A nurse is performing peritoneal dialysis for a client. Which action should the nurse take?
1
Withhold the routine medications until after the procedure.
2
Infuse the dialysate solution slowly over several hours.
3
Warm the dialysate solution slightly before instillation.
4
Place the client in a side-lying position.

3 The infusion should be warmed to body temperature to decrease abdominal discomfort and promote dilation of peritoneal vessels. The side-lying position may restrict fluid inflow and prevent maximum urea clearance; the client should be placed in the semi-Fowler position. The infusion of dialysate solution should take approximately 10 to 20 minutes. Routine medications should not interfere with the infusion of dialysate solution.

The nurse is reviewing the urinalysis reports of four clients with renal disorders.
Which client’s finding signifies the presence of excessive bilirubin?
1
Client 1
2
Client 2
3
Client 3
4
Client 4

yellow brown to olive green Client 3’s urinalysis reports findings of the presence of yellow-brown to olive-green-colored urine which signifies excessive bilirubin. Client 1’s urinalysis report findings of the presence of amber-yellow-colored urine signifies a normal finding. Client 2’s urinalysis report findings of the presence of dark, smoky-colored urine signifies hematuria. Client 4’s urinalysis report findings of orange-red or orange-brown-colored urine indicates the presence of phenazopyridine in the urine.

Which condition should be reported immediately to the primary healthcare provider?
1
Rectal bleeding for 2 days after prostate biopsy
2
Pelvic pain immediately after colposcopy
3
Light vaginal bleeding for 1 to 2 days following a hysterosalpingogram
4
Body temperature of 102° F with vaginal discharge 48 hours after cervical biopsy

4 The client with cervical biopsy should immediately report to the primary healthcare provider if experiencing a body temperature of 102° F with vaginal discharge. This is because fever and vaginal discharge that develops 48 hours after cervical biopsy may be the signs of infection related to the procedure. The client should take pain relievers for pelvic pain after colposcopy. Light vaginal bleeding for 1 to 2 days following hysterosalpingogram is common. If the amount of bleeding increases or extends beyond 2 days, the healthcare provider should be notified. Light rectal bleeding for a few days is common after prostate biopsy.

To prevent bleeding after a suprapubic prostatectomy, the client should be instructed to avoid straining on defecation. Which foods should the nurse encourage the client to eat to help prevent constipation during the recovery period? Select all that apply.
1
Scrambled eggs
2
Green peas
3
Milk
4
Apples
5
Oatmeal

2, 4, 5 Apples, oatmeal, and green peas are high in fiber, which helps prevent constipation. Milk and milk products can be constipating; they do not contain bulk. Scrambled eggs contain little dietary fiber and do not prevent constipation.

The nurse is preparing a client who is on metformin therapy and is scheduled to undergo renal computed tomography with contrast dye. What does the nurse anticipate the primary healthcare provider to inform the client regarding the procedure?
1
"Discontinue metformin a half-day prior to procedure."
2
"Discontinue metformin 7 days following the procedure."
3
"Discontinue metformin 1 day prior to procedure."
4
"Discontinue metformin 3 days following the procedure."

3 Metformin can react with the iodinated contrast dye that is given for a renal computed tomography (CT) and cause lactic acidosis. Therefore the nurse anticipates an instruction that the client should discontinue the metformin 1 day before the procedure. Stopping the metformin a half-day before the renal CT may not reduce the risk of lactic acidosis. The client is advised to discontinue the metformin for at least 48 hours after the procedure. It is not necessary to discontinue metformin for 3 to 7 days after a renal CT with contrast media.

A client is diagnosed with calcium oxalate renal calculi. Which foods should the nurse teach the client to avoid? Select all that apply.
1
Liver
2
Spinach
3
Rhubarb
4
Milk
5
Tea

2, 3, 5 Tea, rhubarb, and spinach are high in calcium oxalate. Limiting oxalate-rich foods limits oxalate absorption and the formation of calcium oxalate calculi. Milk is an acceptable calcium-rich protein and is avoided in calcium stones but not with oxalate stones. Liver is a purine-rich food that may be eaten. Test-Taking Tip: When using this program, be sure to note if you guess at an answer. This will permit you to identify areas that need further review. Also it will help you to see how correct your guessing can be.

A client has glomerulonephritis. To prevent future attacks of glomerulonephritis, the nurse planning discharge teaching includes which instruction?
1
"Avoid situations that involve physical activity."
2
"Restrict fluid intake."
3
"Seek early treatment for respiratory infections."
4
"Take showers instead of bubble baths."

3 A common cause of glomerulonephritis is a streptococcal infection. This infection initiates an antibody formation that damages the glomeruli. Any fluid restriction is moderated as the client improves; fluid is allowed to prevent urinary stasis. The alkalinity of bubble baths is linked to urinary tract infections, not glomerulonephritis. Moderate activity is helpful in preventing urinary stasis, which can precipitate urinary infection.

A client who has been on hemodialysis for several weeks asks the nurse what substances are being removed by the dialysis. Which substance removal should the nurse share with the client?
1
Sodium
2
Bacteria
3
Glucose
4
Blood

1 Sodium is an electrolyte that passes through the semipermeable membrane during hemodialysis. Red blood cells do not pass through the semipermeable membrane during hemodialysis. Glucose does not pass through the semipermeable membrane during hemodialysis. Bacteria do not pass through the semipermeable membrane during hemodialysis.

After a transurethral prostatectomy, a client returns to the postanesthesia care unit with a three-way indwelling catheter with continuous bladder irrigation. Which nursing action is the priority?
1
Observing the suprapubic dressing for drainage
2
Maintaining the client in the semi-Fowler position
3
Monitoring for bright red blood in the drainage bag
4
Encouraging fluids by mouth as soon as the gag reflex returns

3 Blood clots are normal 24 to 36 hours after surgery, but bright red blood can indicate hemorrhage. The surgery is performed through the urinary meatus and urethra; there is no suprapubic incision. It is unnecessary to keep the client in the semi-Fowler position. The client is initially allowed nothing by mouth and then advanced to a regular diet as tolerated. Continuous irrigation supplies enough fluid to flush the bladder.

A client with a history of chronic kidney disease is hospitalized. Which assessment findings will alert the nurse to kidney insufficiency?
1
Edema and pruritus
2
Facial flushing
3
Diminished force and caliber of stream
4
Dribbling after voiding and dysuria

1 The accumulation of metabolic wastes in the blood (uremia) can cause pruritus; edema results from fluid overload caused by impaired urine production. Pallor, not flushing, occurs with chronic kidney disease as a result of anemia. Dribbling after voiding is a urinary pattern that is not caused by chronic kidney disease; this may occur with prostate problems. Diminished force and caliber of stream occur with an enlarged prostate, not kidney disease.

The registered nurse is preparing to assess a client’s renal system. Which statement by the nurse indicates effective technique?
1
"I must first palpate the client if a tumor is suspected."
2
"I must first auscultate the client and then proceed to percussion and palpation."
3
"I must first listen for normal pulse at the client’s wrist region."
4
"I must first examine tender abdominal areas and then proceed to nontender areas."

2 Palpation and percussion can cause an increase in normal bowel sounds and hide abdominal vascular sounds. Therefore it is wise to perform auscultation prior to percussion and palpation during clinical assessment of the renal system. Palpation should be avoided if a client is suspected of having a tumor because it could harm the client. It is more important as part of clinical assessment of the renal system to listen for bruit by auscultating over the renal artery. Bruit indicates renal artery stenosis. The nontender areas should be examined prior to tender areas to avoid confusion regarding radiating pain from the tender area being percussed.

A nurse is assessing a client with a diagnosis of kidney failure for clinical indicators of metabolic acidosis. What should the nurse conclude is the reason metabolic acidosis develops with kidney failure?
1
Inability of the renal tubules to reabsorb water to dilute the acid contents of blood
2
Impaired glomerular filtration, causing retention of sodium and metabolic waste products
3
Depressed respiratory rate due to metabolic wastes, causing carbon dioxide retention
4
Inability of the renal tubules to secrete hydrogen ions and conserve bicarbonate

4 Bicarbonate buffering is limited, hydrogen ions accumulate, and acidosis results. The rate of respirations increases in metabolic acidosis to compensate for a low pH. The fluid balance does not significantly alter the pH. The retention of sodium ions is related to fluid retention and edema rather than to acidosis.

A nurse is caring for a client who is experiencing urinary incontinence. The client has an involuntary loss of small amounts (25 to 35 mL) of urine from an overdistended bladder. How should this be documented in the client’s medical record?
1
Overflow incontinence
2
Stress incontinence
3
Functional incontinence
4
Urge incontinence

1 Overflow incontinence [1] [2] [3] describes what is happening with this client; overflow incontinence occurs when the pressure in the bladder overcomes sphincter control. Urge incontinence describes a strong need to void that leads to involuntary urination regardless of the amount in the bladder. Stress incontinence occurs when a small amount of urine is expelled because of an increase in intraabdominal pressure that occurs with coughing, lifting, or sneezing. Functional incontinence occurs from other issues rather than the bladder, such as cognitive (dementia) or environmental (no toileting facilities). Test-Taking Tip: Do not panic while taking an exam! Panic will only increase your anxiety. Stop for a moment, close your eyes, take a few deep breaths, and resume review of the question.

Which instruction would be most beneficial for an aging African-American client with hypertension?
1
"Record blood pressure weekly."
2
"Check the pulse daily."
3
"Visit an ophthalmologist monthly."
4
"Have an annual urinalysis."

4 African-American clients have 20% less blood flow to the kidneys because of high sodium consumption. This causes anatomical changes in the blood vessels, thereby increasing the risk of kidney failure. Therefore instructing the client with hypertension to have an annual urine examination would be beneficial. If the client has protein in the urine, this is a sign of high blood pressure and can signify kidney damage. Checking the pulse daily poses no harm to the individual, but does not determine if the client has hypertension. Recording the blood pressure weekly is not a good indicator of an aging African-American client with hypertension. The client’s blood pressure should be taken at least daily to determine if the client has problems. If the client has an eye-related problem, visiting an ophthalmologist should be suggested.

The nurse is assisting the primary healthcare provider during a renal ultrasonography. Arrange the steps involved in the procedure in correct sequence.
Incorrect
2
Apply gel over skin
Incorrect
4
Wipe cotton pad over gel
Correct
3.
Move transducer across skin
Incorrect
1
Place client in prone position

The client undergoing renal ultrasonography should first be placed in the prone position. Then the sonographic gel should be applied on the client’s skin over the back and flank regions. Then the transducer is moved across the client’s skin to measure the echoes. The images are visualized on the display screen. At the end of the procedure the gel is removed from the client’s skin by using a piece of wet cotton or cloth over the gel.

A client has a permanent colostomy. During the first 24 hours there is no drainage from the colostomy. How should the nurse interpret this finding?
1
Effective functioning of the nasogastric tube is causing this.
2
Absence of intestinal peristalsis is causing this.
3
Edema after the surgery is causing this.
4
Decrease in fluid intake before surgery is causing this.

2 Absence of peristalsis is caused by manipulation of abdominal contents and the depressant effects of anesthetics and analgesics. Edema will not interfere with peristalsis; edema may cause peristalsis to be less effective, but some output will result. An absence of fiber has a greater effect on decreasing peristalsis than does decreasing fluids. A nasogastric tube decompresses the stomach; it does not cause cessation of peristalsis.

A client scheduled for a transurethral prostatectomy expresses concern about the effect the surgery will have on sexual ability. Which information should the nurse share with the client?
1
Will have prolonged erections
2
Will be impotent
3
May have a diminished sex drive
4
May experience retrograde ejaculations

4 Ejection of semen into the bladder instead of the urethra is common after a transurethral prostatectomy. The surgery should not interfere with the libido and will not cause prolonged erections. Impotence is not typical with this approach; it may occur with the retroperitoneal approach.

After reviewing the urinalysis reports of a client with kidney dysfunction, the nurse suspects the presence of myoglobin. Which finding in the test reports supports the nurse’s suspicion?
1
Very pale yellow colored urine
2
Dark amber colored urine
3
Red-colored urine
4
Brown-colored urine

3 Red-colored urine in clients with kidney dysfunction indicates the presence of myoglobin. Brown-colored urine indicates increased bilirubin levels. Dark amber urine indicates concentrated urine. Very pale yellow urine indicates dilute urine.

A nurse is caring for a client with a ureteral calculus. Which are the most important nursing actions? Select all that apply.
1
Limiting fluid intake at night
2
Administering the prescribed analgesic
3
Monitoring intake and output
4
Recording the client’s blood pressure
5
Straining the urine at each voiding

2, 3, 5 A urinary calculus may obstruct urine flow, which will be reflected in a decreased output; obstruction may result in hydronephrosis [1] [2]. Urine is strained to determine whether any calculi or calcium gravel is passed. Reduction of pain is a priority. A calculus obstructing a ureter causes flank pain that extends toward the abdomen, scrotum and testes, or vulva; the pain begins suddenly and is severe (renal colic). Fluids should be encouraged to promote dilute urine and facilitate passage of the calculi. Recording the blood pressure is not critical.

Which vascular component of the client’s nephron delivers arterial blood from the glomerulus into the peritubular capillaries or the vasa recta?
1
Afferent arteriole
2
Interlobular artery
3
Efferent arteriole
4
Arcuate artery

3 The efferent arteriole is the vascular component of the nephron that delivers arterial blood from the glomerulus into the peritubular capillaries or the vasa recta. The arcuate artery is a curved artery of the renal system that surrounds the renal pyramids. The afferent arteriole is the vascular component of the nephron that delivers arterial blood from the branches of the renal artery into the glomerulus. The interlobular artery feeds the lobes of the kidney.

The nurse is preparing a blood transfusion for a client with renal failure. Why does anemia often complicate renal failure?
1
Increase in blood pressure
2
Decrease in serum sodium concentration
3
Decrease in erythropoietin
4
Increase in serum phosphate levels

3 The hormone erythropoietin, produced by the kidneys, stimulates the bone marrow to produce red blood cells. In renal failure there is a deficiency of erythropoietin that often results in the client developing anemia. Therefore the nurse is instructed to administer blood. In renal failure, increased blood pressure is due to impairment of renal vasodilator factors and is not treated by administration of blood. Phosphate is retained in the body during renal failure, causing binding of calcium leading to done demineralization, not anemia. Increase in urinary sodium concentration and decrease in serum sodium concentration trigger the release of renin from the juxtaglomerular cells.

The nurse is providing education to a client with calculi in the calyces of the right kidney. The client is scheduled to have the calculi removed. Which information should the nurse include in the teaching?
1
The surgery will be performed transurethrally.
2
After surgery, a suprapubic catheter will be in place.
3
During the surgery, the right ureter will be removed.
4
After surgery, there will be a small incision in the right flank area.

4 If the calculi are in the renal pelvis, a percutaneous pyelolithotomy is performed; the calculi are removed via a small flank incision. Removal of the right ureter is not necessary. Performing surgery transurethrally is used for calculi in the ureters and renal pelvis. Placement of a suprapubic catheter usually is unnecessary.

What age-related changes are associated with the female genitalia? Select all that apply.
1
Firm breasts
2
Erected nipples
3
Graying of pubic hair
4
Dry vagina
5
Decreased size of the labia majora

3,4,5 An elderly female client may have dry, smooth, and thin vaginal walls due to atrophy of the vaginal tissue and secretory glands on the vaginal walls. Graying and thinning pubic hair and decreased size of the labia majora and clitoris are also normal signs of aging. The client may have increased flabbiness and fibrosis of the breasts, which hang lower on the chest wall, along with decreased erection of the nipples.

The nurse prepares a client for a Papanicolaou test (Pap test). What should the nurse instruct the client before conducting the test?
1
Douche the vagina with soap
2
Empty the bladder
3
Avoid sexual intercourse for at least 24 hours before the test
4
Avoid scheduling a Pap test to be performed during menses

3 The Papanicolaou test is a cytological study used to detect precancerous and cancerous cells within the cervix. The nurse should advise the client to avoid sexual intercourse at least 24 hours before the test to prevent test misinterpretations. A client undergoing a pelvic examination should empty her bladder immediately before the test. Douching the vagina with soap or applying deodorants may lead to false test results. The Papanicolaou test should be scheduled between the client’s menstrual periods so that the menstrual flow does not interfere with laboratory analysis.

A client with a renal disorder is scheduled for an intravenous pyelogram (IVP). Which interventions should the nurse undertake prior to the procedure? Select all that apply.
1
Instruct the client to lie still during the procedure
2
Have the client remove all metal objects
3
Ensure that the consent form is signed
4
Administer an enema or cathartic to the client
5
Assess the client for iodine sensitivity

3,4,5 The presence, position, shape, and size of kidneys, ureters, and bladder can be evaluated using an intravenous pyelogram (IVP). The contrast medium used in the procedure may cause hypersensitivity reactions. Therefore, the nurse should assess the client for sensitivity to iodine prior to the procedure. The nurse should use a cathartic or enema to empty the colon of feces and gas. An IVP does need a consent form since the procedure is invasive. The nurse has the client remove all metal objects before performing a magnetic resonance imaging (MRI) procedure. The nurse instructs the client to lie still during a computed tomographic (CT) scan procedure; during an IVP the client may be asked to turn certain ways.

The nurse is reviewing blood screening tests of the immune system of a client with acquired immunodeficiency syndrome (AIDS). What does the nurse expect to find?
1
A decrease in the serum level of glucose-6-phosphate dehydrogenase
Incorrect2
An increase in immunoglobulin E
3
An increase in thymic hormones
Correct4
A decrease in CD4 T cells
The human immunodeficiency virus (HIV) infects helper T-cell lymphocytes; therefore 300 or fewer CD4 T cells per cubic millimeter of blood or CD4 cells accounting for less than 20% of lymphocytes is suggestive of AIDS. The thymic hormones necessary for T-cell growth are decreased. An increase in immunoglobulin E is associated with allergies and parasitic infections. A decrease in the serum level of glucose-6-phosphate dehydrogenase is associated with drug-induced hemolytic anemia and hemolytic disease of the newborn.

Test-Taking Tip: As you answer each question, write a few words about why you think that answer is correct; in other words, justify why you selected that answer. If an answer you provide is a guess, mark the question to identify it. This will permit you to recognize areas that need further review. It will also help you to see how correct your "guessing" can be. Remember: on the licensure examination you must answer each question before moving on to the next question.
Topics

Client Needs – Reduction of Risk Potential
Mastery Level 0
1 2 3

Why is a Neisseria gonorrhoeae infection particularly troublesome for a female client?
Correct1
Symptoms are often overlooked.
2
Treatment has many adverse effects.
3
It is difficult to treat with antibiotics.
4
The medication is expensive.
Many female clients who contract gonorrhea are asymptomatic or have minor symptoms that are often overlooked, making it possible for them to remain a source of infection. There is no evidence to support that the medication to treat the infection is expensive. The infection can be treated with one intramuscular injection of ceftriaxone. There is no evidence to support that the medication to treat this infection has many adverse effects.

Which type of cytokine is used to treat anemia related to chronic kidney disease?
Correct1
Erythropoietin
2
α-Interferon
3
Interleukin-2
4
Interleukin-11
Erythropoietin is used to treat anemia related to chronic kidney disease. α-Interferon is used to treat hairy cell leukemia or malignant melanoma. Interleukin-2 is used to treat metastatic renal carcinoma. Interleukin-11 is used to prevent thrombocytopenia after chemotherapy.

Which cytokine is used to treat multiple sclerosis?
1
Erythropoietin
2
Colony-stimulating factor
3
Interleukin-2
Correct4
β-Interferon
β-Interferon is a cytokine used to treat multiple sclerosis. Interleukin-2 is used to treat metastatic melanoma. Erythropoietin is a cytokine used to treat anemia related to chemotherapy. Colony-stimulating factor is a cytokine used to treat chemotherapy-induced neutropenia.

Which sexually transmitted disease is caused by the human papilloma virus?
1
Genital herpes
2
Chlamydia infection
Correct3
Condylomata acuminata
4
Gonorrhea
Condylomata acuminata is a sexually transmitted disease caused by the human papilloma virus. Gonorrhea is caused by Neisseria gonorrhoeae. Genital herpes is causes by the herpes simplex virus. Chlamydia infection is caused by Chlamydia trachomatis.

Test-Taking Tip: Identify option components as correct or incorrect. This may help you identify a wrong answer.

The echoviruses can cause which diseases in clients? Select all that apply.
1
Burkitt’s lymphoma
Incorrect 2
Mononucleosis
Correct 3
Gastroenteritis
4
Parotitis
Correct 5
Aseptic meningitis
Echoviruses cause gastroenteritis and aseptic meningitis. Parotitis is caused by mumps. Burkitt’s lymphoma and mononucleosis are caused by the Epstein-Barr virus.

Which parameter should the nurse consider while assessing the psychologic status of a client with acquired immune deficiency syndrome (AIDS)?
Correct1
Presence of anxiety
2
Sleep pattern
3
Severity of pain
4
Cognitive changes
Presence of anxiety should be considered while assessing the psychologic status of a client with AIDS. Sleep patterns and severity of pain are related to the assessment of activity and rest, a physical status. Cognitive changes are related to the assessment of neurologic status.

A client with localized redness and swelling due to a bee sting reports intense local pain, a burning sensation, and itching. What would be the most appropriate nursing action?
1
Monitoring for neurological and cardiac symptoms
2
Advising the client to launder all clothes with bleach
3
Ensuring the client keeps the skin clean and dry
Correct4
Applying cold compresses to the affected area
A client with a bee sting may have localized redness, swelling, pain, and itching due to an allergic reaction. The nurse should apply cold compresses to the affected area to reduce the pain in the client. A client with Candida albicans infection should keep his or her skin clean and dry to prevent further fungal infections. A client with a Borrelia burgdorferi infection may suffer from cardiac, arthritic, and neurologic manifestations. Therefore the nurse has to monitor for these symptoms. Direct contact may transmit a Sarcoptes scabiei infection; the nurse should make sure that the client’s clothes are bleached to prevent the transmission of the infection.

Test-Taking Tip: Read the question carefully before looking at the answers: (1) Determine what the question is really asking; look for key words; (2) Read each answer thoroughly and see if it completely covers the material asked by the question; (3) Narrow the choices by immediately eliminating answers you know are incorrect.

A client scheduled for surgery has a history of methicillin-resistant Staphylococcus aureus (MRSA) since developing an infection in a surgical site 9 months ago. The site is healed, and the client reports having received antibiotics for the infection. What should the nurse do to determine if the infecting organism is still present?
1
Call the surgeon for an infectious disease consultation.
Incorrect2
Inform the operating room of the MRSA.
3
Notify the infection control officer.
Correct4
Obtain an order to culture the client’s blood.
Obtaining cultures is the most reliable method of determining the presence of an infecting microorganism. Although notifying the infection control officer should be done, the presence of an infecting microorganism should be identified first. Informing the operating room personnel of the MRSA is usual when an infecting microorganism is present; however, it is not yet confirmed that an infecting microorganism is present. Although calling the surgeon for an infectious disease consultation may be done, the presence of an infecting agent should be identified first.

Test-Taking Tip: Pace yourself while taking a quiz or exam. Read the entire question and all answer choices before answering the question. Do not assume that you know what the question is asking without reading it entirely.

Which is the first medication approved to reduce the risk of human immunodeficiency virus (HIV) infection in unaffected individuals?
1
Cromolyn
2
Methdilazine
Incorrect3
Abacavir
Correct4
Truvada
Truvada is the first medication approved to reduce the risk of HIV infection in unaffected individuals who are at a high risk of HIV infection. Abacavir is administered to treat HIV infection and is a reverse transcriptase inhibitor. Cromolyn is administered in the management of allergic

Which sexually transmitted disease is caused by a virus?
1
Syphilis
2
Chlamydial infection
Correct3
Genital warts
4
Gonorrhea
Genital warts are caused by a sexually transmitted virus. Bacteria cause syphilis, gonorrhea, and chlamydial infections.

Which is the first antibody formed after exposure to an antigen?
1
IgG
2
IgA
Correct3
IgM
4
IgE
IgM (immunoglobulin M) is the first antibody formed by a newly sensitized B-lymphocyte plasma cell. IgA has very low circulating levels and is responsible for preventing infection in the upper and lower respiratory tracts, and the gastrointestinal and genitourinary tracts. IgE has variable concentrations in the blood and is associated with antibody-mediated hypersensitivity reactions. IgG is heavily expressed on second and subsequent exposures to antigens to provide sustained, long-term immunity against invading microorganisms.

What are the mediators of injury in IgE-mediated hypersensitivity reactions? Select all that apply.
1
Neutrophils
Correct 2
Leukotrienes
Correct 3
Mast cells
Correct 4
Histamines
5
Cytokines
Mast cells, histamines, and leukotrienes are the mediators of injury in IgE-mediated hypersensitivity reactions. Cytokines are the mediators of injury in the delayed type of hypersensitivity reaction. Neutrophils are the mediators of injury in the immune-complex type of hypersensitivity reaction.

Which immunoglobulin crosses the placenta?
1
IgA
2
IgE
3
IgM
Correct4
IgG
IgG is the only immunoglobulin that crosses the placenta. IgE is found in the plasma and interstitial fluids. IgA lines the mucous membranes and protects body surfaces. IgM is found in plasma; this immunoglobulin activates due to the invasion of ABO blood antigens.

Which type of hypersensitivity reaction is associated with rheumatoid arthritis?
Incorrect1
Cytotoxic
2
Delayed
3
IgE-mediated
Correct4
Immune-complex
Rheumatoid arthritis is an autoimmune disorder associated with an immune-complex type of hypersensitivity reaction. Contact dermatitis caused by poison ivy is associated with a delayed type of hypersensitivity reaction. Goodpasture’s syndrome is associated with a cytotoxic type of hypersensitivity reaction. Asthma is associated with an IgE-mediated type of hypersensitivity reaction.

Which disease is caused by Coronaviruses?
Correct1
Severe acute respiratory syndrome
2
Inhalation anthrax
3
Coccidioidomycosis
4
Pertussis
Severe acute respiratory syndrome is a respiratory infection caused by Coronaviruses. Pertussis is caused by the bacterium Bordetella pertussis. Inhalation anthrax is caused by Bacillus anthracis. Coccidioidomycosis is caused by Coccidioides.

Which disease is caused by Escherichia coli?
Incorrect1
Food poisoning
2
Diphtheria
Correct3
Peritonitis
4
Tetanus
Escherichia coli causes peritonitis. Clostridium tetani causes tetanus. Corynebacterium diphtheria causes diphtheria. Clostridium botulinum causes food poisoning.

A mother with the diagnosis of acquired immunodeficiency syndrome (AIDS) states that she has been caring for her baby even though she has not been feeling well. What important information should the nurse determine?
1
When the baby last received antibiotics
2
How long she has been caring for the baby
Correct3
If the baby is breast-feeding
4
If she has kissed the baby
Epidemiologic evidence has identified breast milk as a source of human immunodeficiency virus (HIV) transmission. Kissing is not believed to transmit HIV. When the baby last received antibiotics is unrelated to transmission of HIV. HIV transmission does not occur from contact associated with caring for a newborn.

What is the mechanism of action of penicillin?
Correct1
Inhibits cell wall synthesis of the pathogen
2
Prevents reproduction of the pathogen
3
Injures the cytoplasmic membrane of the pathogen
4
Inhibits nucleic acid synthesis of the pathogen
Penicillin is an antimicrobial medication that inhibits cell wall synthesis of the susceptible pathogen. Gentamicin is an antimicrobial medication that prevents the reproduction of the susceptible pathogen. Actinomycin is an antimicrobial medication that inhibits nucleic acid synthesis of the susceptible pathogen. Antifungal agents injure the cytoplasmic membrane of the susceptible pathogen.

Which drug treats hay fever by preventing leukotriene synthesis?
Incorrect1
Chlorpheniramine
2
Diphenhydramine
3
Cromolyn sodium
Correct4
Zileuton
Zileuton [1] [2] is a leukotriene antagonist drug; this substance prevents the synthesis of leukotrienes and helps in managing and preventing hay fever. Cromolyn sodium stabilizes mast cells and prevents the opening of mast cell membranes in response to allergens binding to immunoglobulin E.. Chlorpheniramine and diphenhydramine are antihistamines and prevent the binding of histamine to receptor cells and decrease allergic manifestations.

Which sexually transmitted infection (STI) is caused by Treponema pallidum?
1
Gonorrhea
2
Vulvovaginitis
3
Genital warts
Correct4
Syphilis
Syphilis is an STI caused by Treponema pallidum. Neisseria gonorrhoeae causes gonorrhea. Haemophilus ducreyi and Klebsiella granulomatis cause genital warts. Herpes simplex virus, Trichomonas vaginalis, and Candida albicans may cause vulvovaginitis.

Which drug treats hay fever by preventing leukotriene synthesis?
Incorrect1
Chlorpheniramine
2
Diphenhydramine
3
Cromolyn sodium
Correct4
Zileuton
Zileuton [1] [2] is a leukotriene antagonist drug; this substance prevents the synthesis of leukotrienes and helps in managing and preventing hay fever. Cromolyn sodium stabilizes mast cells and prevents the opening of mast cell membranes in response to allergens binding to immunoglobulin E.. Chlorpheniramine and diphenhydramine are antihistamines and prevent the binding of histamine to receptor cells and decrease allergic manifestations.

Which client organ is protected by microglial cells?
Correct1
Brain
2
Kidney
3
Lung
4
Liver
Microglial cells are macrophages present in the brain. The lungs are protected by alveolar macrophages. The liver is protected by Kupffer cells. Mesangial cells are present in the kidneys.

A nurse is teaching a health class about human immunodeficiency virus (HIV). Which basic methods are used to reduce the incidence of HIV transmission? Select all that apply.
1
Using separate toilets
2
Sterilizing the household utensils
Correct 3
Practicing sexual abstinence
4
Preventing direct casual contacts
Correct 5
Using condoms
HIV is found in body fluids such as blood, semen, vaginal secretions, breast milk, amniotic fluid, urine, feces, saliva, tears, and cerebrospinal fluid. Therefore a client should use condoms to prevent contact between the vaginal mucus membranes and semen. Practicing sexual abstinence is the best method to prevent transmission of the virus. The HIV virus is not transmitted by sharing the same toilet facilities, casual contacts such as shaking hands and kissing, or by sharing the same household utensils.

Which immunomodulatory is beneficial for the treatment of clients with multiple sclerosis?
1
Interleukin 2
2
Interleukin 11
3
Alpha interferon
Correct4
Beta interferon
Beta interferon is an immunomodulator that is administered in the treatment of multiple sclerosis. Interleukin 11 is used in the prevention of thrombocytopenia after chemotherapy. Interleukin 2 is used for the treatment of metastatic renal cell carcinoma and metastatic melanoma. Alpha interferon is administered for the treatment of hairy cell leukemia, chronic myelogenous leukemia, and malignant melanoma.

Which cytokine stimulates the liver to produce fibrinogen and protein C?
Correct1
Interleukin-6
Incorrect2
Interleukin-1
3
Tumor necrosis factor
4
Thrombopoietin
Interleukin-6 stimulates the liver to produce fibrinogen and protein C. Interleukin-1 stimulates the production of prostaglandins. Thrombopoietin increases the growth and differentiation of platelets. Tumor necrosis factor stimulates delayed hypersensitivity reactions and allergies.

Test-Taking Tip: Identify option components as correct or incorrect. This may help you identify a wrong answer.

Which cytokine medication is administered to treat chemotherapy-induced neutropenia?
1
Aldesleukin
2
Oprelvekin
3
Darbepoetin alfa
Correct4
Filgrastim
Colony-stimulating factors such as filgrastim are administered to treat chemotherapy-induced neutropenia. Oprelvekin is used to prevent thrombocytopenia. Aldesleukin is used to treat metastatic renal cell carcinoma. Darbepoetin alfa is administered to treat anemia related to chronic cancer and anemia related to chronic kidney disease.

A client’s sputum smears for acid-fast bacilli (AFB) are positive, and transmission-based airborne precautions are prescribed. What should the nurse teach visitors to do?
Correct1
Wear a particulate respirator mask.
2
Limit contact with the client’s nonexposed family members.
Incorrect3
Put on a gown and gloves.
4
Avoid touching objects in the client’s room.
Tubercle bacilli are transmitted through air currents; therefore personal protective equipment, such as a particulate respirator that filters out organisms as small as 1 µm, is necessary. Gowns and gloves are not necessary. Tuberculosis is spread by airborne microorganisms; gloves are necessary only when touching articles contaminated with respiratory secretions. It is only necessary to avoid contact with objects in the client’s room that are contaminated with respiratory secretions. Limiting contact with the client’s nonexposed family members is unnecessary.

A client’s laboratory report reveals a CD4+ T-cell count of 520 cells/mm3. According to the Centers for Disease Control and Prevention (CDC), which stage of human immunodeficiency virus (HIV) disease is present in the client?
Incorrect1
Stage 3
2
Stage 4
Correct3
Stage 1
4
Stage 2
According to the CDC, HIV disease is divided into four stages. A client with a CD4+ T-cell count of greater than 500 cells/mm3 is in the first stage of HIV disease. A client with a CD4+ T-cell count between 200 and 499 cells/mm3 is in the second stage of HIV disease. A client with a CD4+ T-cell count of less than 200 cells/mm3 is in the third stage of HIV disease. The fourth stage of HIV disease indicates a confirmed HIV infection with no information regarding the CD4+ T-cell counts.

What are the clinical manifestations of inhalation anthrax? Select all that apply.
Correct 1
Fatigue
Correct 2
Fever
Correct 3
Dry cough
4
Sore throat
5
Rhinitis
Inhalation anthrax is a bacterial infection caused by Bacillus anthracis. Clinical manifestations include fever, fatigue, and a dry cough. Rhinitis and a sore throat are upper respiratory manifestations not associated with inhalation anthrax.

Which is a leukotriene antagonist used to manage and prevent allergic rhinitis?
Correct1
Zileuton
2
Ephedrine
Incorrect3
Cromolyn sodium
4
Scopolamine
Zileuton is a leukotriene antagonist used to manage and prevent allergic rhinitis. Ephedrine is an ingredient in decongestants used to treat allergic rhinitis. Scopolamine is an anticholinergic used to reduce secretions. Cromolyn sodium is a mast cell stabilizing drug used to prevent mast cell membranes from opening when an allergen binds to IgE.

Test-Taking Tip: You have at least a 25% chance of selecting the correct response in multiple-choice items. If you are uncertain about a question, eliminate the choices that you believe are wrong and then call on your knowledge, skills, and abilities to choose from the remaining responses.

Which age-related effects on the immune system are seen in the older client?
1
Increased expression of IL-2 receptors
2
Increased delayed hypersensitivity reaction
Correct3
Increased autoantibodies
4
Increased primary and secondary antibody responses
The effects of aging on the immune system include increased autoantibodies. Expression of IL-2 receptors, delayed hypersensitivity reaction, and primary and secondary antibody responses decrease in older adults because of the effects of aging on the immune system.

A child is diagnosed with hepatitis A. The client’s parent expresses concern that the other members of the family may get hepatitis because they all share the same bathroom. What is the nurse’s best reply?
1
"I suggest that you buy a commode exclusively for your child’s use."
Correct2
"All family members, including your child, need to wash their hands after using the bathroom."
3
"Your child may use the bathroom, but you need to use disposable toilet covers."
4
"You will need to clean the bathroom from top to bottom every time a family member uses it."
Hepatitis A is spread via the fecal-oral route; transmission is prevented by proper hand washing. Buying a commode exclusively for the child’s use is unnecessary; cleansing the toilet and washing the hands should control the transmission of microorganisms. It is not feasible to clean "from top to bottom" each time the bathroom is used. The use of disposable toilet covers is inadequate to prevent the spread of microorganisms if the bathroom used by the child also is used by others. Hand washing by all family members must be part of the plan to prevent the spread of hepatitis to other family members.
Topics

Which organism is responsible for causing Lyme disease in clients?
Correct1
Borrelia burgdorferi
2
Sarcoptes scabiei
3
Pediculushumanus var. corporis
4
Phthirus pubis
Lyme disease is a bacterial infection caused by Borrelia burgdorferi, which is transmitted by ticks. Phthirus pubis causes pediculosis. Scabies is caused by Sarcoptes scabiei. Pediculushumanus var. corporis also causes pediculosis.

Which type of hypersensitivity reaction will occur when the client’s T cytotoxic cells are involved as the mediators of injury?
Correct1
Type IV
2
Type I
Incorrect3
Type II
4
Type III
Type IV hypersensitivity reaction will occur when the T cytotoxic cells are involved as the mediators of injury. Type I IgE-mediated reaction will occur when histamine is involved as the mediators of injury. Type II cytotoxic reaction will occur when complement lysis is the mediator of injury. Type III immune complex reaction will occur when neutrophils are involved as the mediators of injury.

What finding in the client is a sign of allergic rhinitis?
Correct1
Reduced transillumination on the skin over the sinuses
2
Presence of pinkish nasal discharge
3
Presence of high-grade fever
Incorrect4
Reduced breathing through the mouth
Reduced transillumination on the skin overlying the sinuses indicates allergic rhinitis. This effect is caused by the sinuses becoming inflamed and blocked with thick mucoid secretions. Generally, fever does not accompany allergic rhinitis unless the client develops a secondary infection. In allergic rhinitis, the client is unable to breathe through the nose because it gets stuffy and blocked. Instead the client will resort to mouth breathing. Clients with allergic rhinitis will have clear or white nasal discharge.

What causes medications used to treat AIDS to become ineffective?
1
Taking medications from different classifications
2
Taking the medications 90% of the time
Correct3
Missing doses of the prescribed medications
Incorrect4
Developing immune reconstitution inflammatory syndrome (IRIS)
The most important reason for the development of drug resistance in the treatment of AIDS is missing doses of drugs. When doses are missed, the blood drug concentrations become lower than what is needed to inhibit viral replication. The virus replicates and produces new particles that are resistant to the drugs. Taking the medications 90% of the time prevents medications from becoming ineffective. Taking medications from different classes prevents the drugs from becoming ineffective. Immune reconstitution inflammatory syndrome (IRIS) occurs when T-cells rebound with medication therapy and become aware of opportunistic infections.

Which diseases may occur due to rickettsial infections? Select all that apply.
1
West Nile fever
2
Leprosy
Correct 3
Rocky Mountain spotted fever
4
Lyme disease
Correct 5
Typhoid fever
Typhoid fever and Rocky Mountain spotted fever are caused by rickettsial infections. Spirochetes and Mycobacterium leprae cause leprosy. Borrelia burgdorferi cause Lyme disease. The West Nile virus causes West Nile fever.

What should be used to clean needles and syringes between intravenous drug users (IDUs)?
1
Hot water
Correct2
Bleach
3
Ammonia
Incorrect4
Rubbing alcohol
Intravenous drug users (IDUs) should be instructed to fill syringes with household bleach and shake the syringe for 30 to 60 seconds. Hot water, ammonia, or rubbing alcohol are not used to disinfect used syringes.

What are the clinical manifestations during the fulminant stage in a client with inhalation anthrax? Select all that apply.
Correct 1
Pleural effusion
2
Harsh cough
3
Mild chest pain
Correct 4
Body temperature of 104 °F
Correct 5
Septic shock
Inhalation anthrax is a bacterial infection caused by Bacillus anthracis. Manifestations such as septic shock, pleural effusion, and body temperature above 103°F indicate the fulminant stage of inhalation anthrax. The prodromal stage is the early stage of inhalation anthrax; clinical manifestations include a harsh cough and mild chest pain.

A nurse discusses the potential for cross-contamination with the nursing assistants on a surgical unit. What does the nurse explain that standard precautions are designed to do?
1
Ensure that hygiene practices by clients are performed in a universal way
2
Be used when clients are suspected of having a communicable disease
3
Create categories in which certain additional precautions must be followed
Correct4
Decrease the risk of transmitting unidentified pathogens
Standard precautions are used for all clients in all settings, regardless of their diagnosis or presumed infectiousness. Practices associated with standard precautions require healthcare providers, not a client, to use hand washing and personal protective equipment to protect themselves and others from body fluids. Transmission-based precautions, known as airborne, droplet, and contact precautions, are based on a client’s diagnosed infection.

Which autoantigens are responsible for the development of Crohn’s disease?
Correct1
Crypt epithelial cells
2
Basement membranes of the lungs
3
Thyroid cell surface
4
Basement membranes of the glomeruli
Crypt epithelial cells are considered to be the autoantigens responsible for Crohn’s disease. Thyroid cell surfaces are autoantigens responsible for Hashimoto’s thyroiditis. The pulmonary and glomerular basement membranes act as autoantigens responsible for Goodpasture syndrome.

Why would a client with acquired immunodeficiency syndrome (AIDS) be administered pregabalin?
Correct1
To reduce muscle and joint pain
2
To reduce cognitive difficulty
Incorrect3
To reduce neuropathic pain
4
To reduce swallowing difficulty
Pregabalin is used to manage muscle and joint pain in clients with AIDS. Neuropathic pain can be managed with amitriptyline, which is a tricyclic antidepressant. Clients with AIDS generally exhibit emotional and behavioral changes, which can be managed with appropriate antidepressants and anxiolytics. AIDS clients who experience difficulty swallowing may have candidal esophagitis; this condition can be managed with antifungal mediations such as fluconazole or amphotericin B.

A nurse is caring for a client with pruritic lesions from an IgE-mediated hypersensitivity reaction. Which mediator of injury is involved?
1
Cytokine
Correct2
Histamine
3
Neutrophil
4
Macrophage
Histamine is one of the mediators of injury involving IgE-mediated injury that may cause pruritus. Cytokines are the mediators of injury in delayed hypersensitivity reaction. Neutrophils are involved in immune complex-mediated hypersensitivity reactions. Macrophages in tissues are involved in cytotoxic reactions.

The nurse suspects that a client is in the chronic persistent stage of Lyme disease. Which symptoms support the nurse’s suspicion? Select all that apply.
1
Erythema migrans
2
Dyspnea
3
Dizziness
Correct 4
Chronic fatigue
Correct 5
Arthritis
Lyme disease is a systemic infectious disease caused by the spirochete Borrelia burgdorferi. The symptoms of the chronic persistent stage are arthritis and chronic fatigue. Dyspnea and dizziness are the symptoms of the early disseminated stage. Erythema migrans is observed in the localized stage.

Test-Taking Tip: Be alert for details about what you are being asked to do. In this Question Type, you are asked to select all options that apply to a given situation or client. All options likely relate to the situation, but only some of the options may relate directly to the situation.

A client who abused intravenous drugs was diagnosed with the human immunodeficiency virus (HIV) several years ago. What does the nurse explain to the client regarding the diagnostic criterion for acquired immunodeficiency syndrome (AIDS)?
Incorrect1
Is capable of transmitting the virus to others
2
Contracts HIV-specific antibodies
3
Develops an acute retroviral syndrome
Correct4
Has a CD4+T-cell lymphocyte level of less than 200 cells/µL (60%)
AIDS is diagnosed when an individual with human immunodeficiency virus (HIV) develops one of the following: a CD4+T-cell lymphocyte level of less than 200 cells/µL (60%), wasting syndrome, dementia, one of the listed opportunistic cancers (e.g., Kaposi sarcoma [KS], Burkitt lymphoma), or one of the listed opportunistic infections (e.g., Pneumocystis jiroveci pneumonia, Mycobacterium tuberculosis). The development of HIV-specific antibodies (seroconversion), accompanied by acute retroviral syndrome (flulike syndrome with fever, swollen lymph glands, headache, malaise, nausea, diarrhea, diffuse rash, joint and muscle pain) 1 to 3 weeks after exposure to HIV reflects acquisition of the virus, not the development of AIDS. A client who is HIV positive is capable of transmitting the virus with or without the diagnosis of AIDS.

Which client is most likely to develop IgE antibodies?
1
A client undergoing a blood transfusion
Incorrect2
A client with bacterial infection
3
A client undergoing a poison ivy reaction
Correct4
A client with pollen allergy
A client with a pollen allergy develops IgE antibodies that may result in an anaphylactic reaction. A client with poison ivy develops delayed hypersensitivity, which is mediated by T lymphocytes. A client with a bacterial infection develops IgG and IgM antibodies. A client undergoing blood transfusion may develop IgG and IgM type II hypersensitivity reactions.

What is the function of a client’s natural killer cells?
Correct1
Attack non-selectively on non-self cells, especially mutated and malignant cells
Incorrect2
Enhance immune activity through secretion of various factors, cytokines, and lymphokines
3
Heighten selectively and destroy non-self cells, including virally infected cells
4
Secrete immunoglobulins in response to the presence of a specific antigen
Natural killer cells attack non-selectively on non-self cells, especially body cells that have undergone mutation and become malignant. Plasma cells secrete immunoglobulins in response to the presence of a specific antigen. Cytotoxic T-cells attack selectively and destroy non-self cells, including virally infected cells. Helper T-cells enhance immune activity through secretion of various factors, cytokines, and lymphokines.

Which malnutrition condition may predispose a client to secondary immunodeficiency?
1
Hodgkin’s lymphoma
2
Cirrhosis
Correct3
Cachexia
4
Diabetes mellitus
Cachexia is a nutrition disorder that may occur due to wasting of muscle mass and weight, resulting in secondary immunodeficiency disorder. Cirrhosis, diabetes mellitus, and Hodgkin’s lymphoma also lead to secondary immunodeficiency disorder, but these are not malnutrition disorders.

Which autoantigens are responsible for the development of Crohn’s disease?
1
Basement membranes of the glomeruli
2
Thyroid cell surface
Correct3
Crypt epithelial cells
4
Basement membranes of the lungs
Crypt epithelial cells are considered to be the autoantigens responsible for Crohn’s disease. Thyroid cell surfaces are autoantigens responsible for Hashimoto’s thyroiditis. The pulmonary and glomerular basement membranes act as autoantigens responsible for Goodpasture syndrome.

Which type of immunity is acquired through the transfer of colostrum from the mother to the child?
1
Artificial active immunity
2
Artificial passive immunity
Correct3
Natural passive immunity
4
Natural active immunity
Natural passive immunity is acquired through the transfer of colostrum from the mother to the child. Natural active immunity is acquired when there is a natural contact with an antigen through a clinical infection. Artificial active immunity is acquired through immunization with an antigen. Artificial passive immunity is acquired by injecting serum from an immune human.

Which autoimmune disease is directly related to the client’s central nervous system?
Incorrect1
Rheumatic fever
2
Goodpasture syndrome
Correct3
Multiple sclerosis
4
Myasthenia gravis
Multiple sclerosis is a central nervous system-specific autoimmune disease. Rheumatic fever is related to the heart. Myasthenia gravis is a muscle-related autoimmune disease. Goodpasture syndrome is a kidney-related autoimmune disease.

Which drug can be administered via the intramuscular route to treat anaphylaxis?
1
Phenylephrine
2
Mycophenolate mofetil
Correct3
Epinephrine
4
Methdilazine
Epinephrine is administered through the intramuscular route to treat anaphylaxis. Methdilazine is administered to treat allergic reactions and pruritus. Phenylephrine is administered orally, not intramuscularly, to treat anaphylaxis. Mycophenolate mofetil is administered intravenously as an immunosuppressant agent.

client is treated with methyldopa for hypertension. For which side effect should the nurse monitor the client?
Correct1
Hemolytic anemia
2
Lupus-like syndrome
Incorrect3
Xerostomia
4
Thrombocytopenia
Methyldopa is used in the treatment of hypertension. It can be a precipitating factor in an autoimmune disease such as hemolytic anemia. Scopolamine transdermal, an anticholinergic, may cause dry mouth or xerostomia. Chemotherapy drugs, such as mycophenolate mofetil and azathiprine, can cause thrombocytopenia. Procainamide is an anti-arrhythmic agent that can induce the formation of antinuclear antibodies and cause a lupus-like syndrome.

A client’s laboratory report shows severe neutropenia and thrombocytopenia. Which medication may have caused this condition?
Incorrect1
Cyclosporine
Correct2
Mycophenolate mofetil
3
Methylprednisolone
4
Daclizumab
Mycophenolate mofetil is a cytotoxic drug that may cause neutropenia and thrombocytopenia. Daclizumab may cause hypersensitivity reaction and anaphylaxis. Cyclosporine may cause neurotoxicity, nephrotoxicity, and hypertension. Methylprednisolone may cause peptic ulcers, osteoporosis, and hyperglycemia.

A client presents with red, inflamed skin covered with papules, vesicles, and bullae from a type IV hypersensitivity reaction. Which condition/disease will the nurse most likely observe written in the client’s electronic medical record?
1
Systemic lupus erythematosus (SLE)
Correct2
Contact dermatitis
Incorrect3
Allergic rhinitis
4
Goodpasture syndrome
Contact dermatitis is a type of type IV delayed hypersensitivity reaction. Type IV hypersensitivity involves a cell-mediated response that may result in tissue damage and skin lesions. The skin lesions with redness of skin are characterized by the presence of papules, vesicles, and bullae. This indicates the presence of erythematosus resulting in contact dermatitis. Sneezing, lacrimation, swelling with airway obstruction, and pruritus around the eyes, nose, throat, and mouth are symptoms that occur due to hypersensitivity reaction resulting in allergic rhinitis, a type 1 hypersensitivity reaction mediated by immunoglobulin E. Goodpasture syndrome, a type II hypersensitivity reaction, is a disorder involving the lungs and kidneys that causes deposits of immunoglobulin G to form along the basement membranes of the lungs or kidneys. Systemic lupus erythematosus (SLE) is an autoimmune disorder that is characterized by damage to multiple organs such as kidneys, joints, and the brain; SLE is a type III hypersensitivity reaction.

The primary healthcare provider has prescribed fluticasone nasal spray for a client with sinusitis. Which of these instructions regarding the use of the spray are appropriate for the client? Select all that apply.
Correct 1
Use the drug on a regular basis, not PRN
2
Remember that driving may be dangerous because of the drug’s sedative effect
3
Begin taking the drug 2 weeks before pollen season starts and use it throughout the season
Correct 4
Discontinue use of the drug if nasal infection develops
Correct 5
Clear the nasal passages before using the drug
Fluticasone is a corticosteroid that is prescribed as a nasal spray in cases of sinusitis and rhinitis. The nurse should instruct the client to use the drug on a regular basis, not as needed, to clear the nasal passages before using the drug, and to discontinue use of the drug if nasal infection develops. Warning the client that driving may be dangerous because of the sedative effect would be beneficial for a client prescribed an antihistamine such as brompheniramine. The nurse should tell a client prescribed a mast cell stabilizer such as cromolyn spray to begin the drug 2 weeks before pollen season starts and to use it throughout the pollen season.

A client with bubonic plague has a body temperature of 103° F associated with chills, swollen glands, headache, and weakness. Which microorganism is most likely responsible for the client’s condition?
1
Bordetella pertusis
2
Mycobacterium tuberculosis
3
Corynebacterium diphtheria
Correct4
Yersinia pestis
Bubonic plague is a reemerging infection caused by Yersinia pestis, which is associated with elevated body temperature, chills, swollen glands, headache, and weakness. Bordetella pertusis causes pertussis, which is associated with acute, highly contagious respiratory disease characterized by loud whooping cough. Mycobacterium tuberculosis causes tuberculosis, which is transmitted by inhalation of infected droplets. Corynebacterium diphtheria causes a localized infection of mucous membranes or skin such as diphtheria.

After assessing an older client’s medical report, the nurse finds that the client is at an increased risk for bacterial and fungal infections. Which change in immune function may have occurred?
Correct1
Decrease in circulating T-lymphocytes
2
Reduction of colony-forming B-lymphocytes
3
Decline in natural bodies
4
Reduction of neutrophil function
A decrease in circulating T-lymphocytes occurs with cell-mediated immunity, resulting in an increased risk of bacterial and fungal infections. A client would need booster shots for old vaccinations and immunizations when there is a decline in natural antibodies. A reduced neutrophil function may be an implication when neutrophil function is decreased. The older adult should receive immunizations, such as flu shots, when the number of colony-forming B-lymphocytes is diminished.

A nurse is developing a teaching plan for a client with scleroderma. What should the nurse include about skin care?
1
Use calamine lotion for pruritus.
Correct2
Keep skin lubricated with lotion.
3
Apply warm soaks to inflamed areas.
4
Take frequent baths to remove scaly lesions.
With scleroderma, the skin becomes dry because of interference with the underlying sweat glands. Pruritus, inflamed areas, and skin lesions are not associated with scleroderma.

The nurse suspects the Jarisch-Herxheimer reaction in a client with syphilis who is on antibiotic therapy. Which symptoms in the client support the nurse’s suspicion? Select all that apply.
Correct 1
Generalized ache
Correct 2
Fever
3
Vasoconstriction
Correct 4
Pain at the injection site
5
Hypertension
Fever, generalized ache, and pain at the injection site are signs of the Jarisch-Herxheimer reaction in a client with syphilis receiving antibiotic therapy. This reaction is caused by the rapid release of products from the disruption of cells of the organism. Hypotension because of vasodilatation and declining peripheral resistance, not hypertension and vasoconstriction, are additional signs of Jarisch-Herxheimer reaction.

A client in the emergency department states, "I was bitten by a raccoon while I was fixing a water pipe in the crawl space of my basement." Which is the most effective first-aid treatment for the nurse to use for this client?
1
Applying a tourniquet proximal to the wound
Incorrect2
Administering an antivenin
Correct3
Cleansing the wound with soap and water
4
Maintaining a pressure dressing
Infection is caused by viral contact with the dermal layer of skin; cleansing the wound with soap and water helps remove superficial contaminants. Antivenins are not effective against microbiologic stresses. A pressure dressing will not prevent infection. Application of a tourniquet may impair circulation and will not prevent infection.

A client presents with cutaneous lesions with swelling in the face, eyelids, and lips from dilation and engorgement of the capillaries. No welts or vesicles are observed. Which condition most likely has occurred in the client?
Correct1
Angioedema
2
Atopic dermatitis
3
Urticaria
4
Systemic lupus erythematosus (SLE)
Angioedemais a localized cutaneous lesion similar to urticaria but involving deeper layers of the skin and the submucosa. The principal areas of involvement include the eyelids, lips, tongue, and face. It occurs due to the dilation and engorgement of the capillaries. Urticaria is a cutaneous reaction against systemic allergens occurring in atopic people. It is characterized by transient wheals that may vary in size and shape and occur all over the body. Atopic dermatitis is a chronic, inherited skin disorder that is characterized by exacerbations and remissions. The skin lesions are more generalized and involve vasodilation of blood vessels, resulting in interstitial edema with vesicle formation. SLE is an autoimmune disease that is characterized by damage to multiple organs (kidneys, skin, joints, lungs).

A client with P. jiroveci pneumonia (PCP) infection arrives at the hospital for a follow-up visit. The laboratory report shows the CD4+ T-cell count as 150. Which medication will be most beneficial for this client?
Correct1
Pentamidine isethionate
2
Piperacillin/tazobactam
3
Dapsone
Incorrect4
Atovaquone
Aerosolized pentamidine isethionate is used as prophylaxis for clients with CD4+ T-cell counts below 200 as well as for those who already have PCP. Dapsone and atovaquone are used as alternative therapies to trimethoprim/sulfamethoxazole for existing PCP or as prophylaxis. Piperacillin/tazobactam is used to treat certain pneumonias but not P. jiroveci.

A nurse in a public health clinic is teaching clients how to prevent toxoplasmosis. What should the nurse instruct the clients to avoid?
Incorrect1
Ingestion of freshwater fish
2
Exposure to heavy metals
Correct3
Contact with cat feces
4
Excessive radiation exposure
Toxoplasma gondii, a protozoan, can be transmitted by exposure to infected cat feces or by ingestion of undercooked, contaminated meat. Toxoplasmosis is not related to heavy metals. T. gondii is a parasite of warm-blooded animals; fish are not considered the source of contamination. Toxoplasmosis is not related to radiation.

Which immunodeficiency disorders affect polymorphonuclear leukocytes and monocytes? Select all that apply.
Correct 1
Chronic granulomatous disease
Correct 2
Job syndrome
3
Graft-versus-host disease
4
DiGeorge syndrome
5
Ataxia-telangiectasia
Job syndrome and chronic granulomatous disease are immunodeficiency disorders that affect polymorphonuclear leukocytes and monocytes. DiGeorge syndrome affects T-cells. Ataxia-telangiectasia affects B- and T-cells. Graft-versus-host disease affects B- and T-lymphocytes.

Which test result should a nurse review to determine if the antibiotic prescribed for the client will be effective?
Incorrect1
Sedimentation rate
2
Serologic test
3
Serum osmolality
Correct4
Sensitivity test
Infected body fluids are tested to determine the antibiotics to which the organism is particularly sensitive or resistant (sensitivity). The serologic test checks for antibody content. The serum osmolality test provides data about fluid and electrolyte balance. The erythrocyte sedimentation rate (ESR) is a nonspecific test for the presence of inflammation.

The T lymphocytes in a healthy older adult client are 120 cells/mm3 (120 cells/uL). The chest x-ray reveals shrinkage of the thymus gland. What might have led to the client’s condition?
Incorrect1
Immunosuppression
2
Immunodeficiency
3
Immunocompetence
Correct4
Immunosenescence
Immunosenescence is the primary cause of thymic involution (shrinking) and decrease in number of T cells to 120 cells/mm3 (120 cells/uL) in an older client (normal range in healthy clients: 500 cells/mm3 [500 cells/uL] to 1,200 cells/mm3 [1,200 cells/uL]). Immunodeficiency is characterized by lack of or diminished reaction to an antigen or a group of antigens. Immunosuppression is a reduction of the activation or efficacy of the immune system, which occurs due to adverse reactions to treatment of other conditions. Immunocompetence exists when the body’s immune system can identify and inactivate or destroy foreign substances.

A client with acquired immunodeficiency syndrome (AIDS) is suspected to be infected with Cryptococcus neoforman. Which symptoms in the client support the nurse’s suspicion? Select all that apply.
Incorrect 1
Diarrhea
Correct 2
Seizures
Correct 3
Confusion
Incorrect 4
Persistent dry cough
Correct 5
Fever
Cryptococcosis is caused by Cryptococcus neoformans and is a debilitating meningitis. It can be a widely spread infection in a client with AIDS. The symptoms of cryptococcosis are fever, seizures, and confusion because the disease affects the brain. Cryptosporidium infection causes diarrhea. Pneumocystis jiroveci pneumonia causes persistent dry cough.

Which leukocyte values should be assessed to determine the adequacy of a client’s response to inflammation? Select all that apply.
1
Plasma cells
Correct 2
Monocytes
3
T-helper cells
Correct 4
Macrophages
Correct 5
Neutrophils
In response to inflammation, monocytes destroy bacteria and cellular debris; neutrophils ingest and phagocytize microorganisms and foreign protein; and macrophages destroy bacteria and cellular debris. Plasma cells are a part of antibody-mediated immunity and secrete immunoglobulins in response to the presence of a specific antigen. T-helper cells are a part of cell-mediated immunity and enhance immune activity through the secretion of various factors, cytokines, and lymphokines.

Arrange the sequence of steps required to stimulate antibody-mediated immunity in its correct sequence.

Antibody-mediated immunity reactions start when new antigens invade the human body. After exposure, the antigen is recognized by the macrophages. Next virgin B lymphocytes are sensitized to the new antigen. After this action, B lymphocytes produce antibodies. These antibodies then bind to the antigen and an immune complex is formed. The binding of antibodies causes cellular events and results in the elimination of the antigen.

A client with Lyme disease presents with dyspnea, dizziness, and facial paralysis. Which medication may be included in the prescription?
Correct1
Ceftriaxone
Incorrect2
Amoxicillin
3
Doxycycline
4
Erythromycin
Lyme disease is a systemic infectious disease caused by the spirochete Borrelia burgdorferi and results from the bite of an infected deer tick, also known as the blacklegged tick. Stage II of Lyme disease is characterized by dizziness, dyspnea, and facial paralysis, and may be treated with ceftriaxone. Amoxicillin, doxycycline, and erythromycin are prescribed to treat localized stage I of Lyme disease.

A client reports a cold and a whooping sound with the cough. Which organism is responsible for this condition?
1
Mycobacterium tuberculosis
2
Corynebacterium diphtheria
3
Yersinia pestis
Correct4
Bordetella pertussis
Pertussis is an acute highly contagious respiratory disease characterized by loud whooping inspirations (whooping cough). Pertussis is caused by Bordetella pertussis. Yersinia pestis causes plague. Corynebacterium diphtheria causes diphtheria. Mycobacterium tuberculosis causes tuberculosis.

Which pathogen is responsible for the condition illustrated in the image?
3202828073
1
Candida albicans
2
Human herpes virus-8
3
Coccidioides immitis

Correct 4
Epstein-Barr virus

Oral hairy leukoplakia is an infection in which painless, white, raised lesions on the lateral aspect of the tongue are present. Epstein-Barr virus causes this condition. Candida albicans causes oral thrush. Coccidioides immitis causes pneumonia and fever. Human herpes virus-8 causes Kaposi sarcoma.

What should the nurse explain as the reason a client prescribed corticosteroid therapy for a chronic health problem develops frequent infections?

1

"They enhance the inflammatory process."

2

"The medication makes the white blood cells work harder."

Correct 3

"They affect antigen-antibody immunity."

Incorrect 4

"They increase the number of inflammatory chemicals in the blood."

Corticosteroids reduce the number of circulating T-cells and suppress cell-mediated immunity. They also interfere with immunoglobulin G (IgG) production and reduce antibody-antigen binding. Corticosteroids suppress the inflammatory process, block the movement of white blood cells, and disrupt the synthesis of a variety of inflammatory

A pregnant woman presents with a body temperature of 103 °F, cough, headache, muscle aches, chest pain, severe joint pain and night sweats and is diagnosed with coccidioidomycosis. Which medication will the nurse most likely observe prescribed on the medication administration record?

Correct 1

Amphotericin B

Incorrect 2

Ciprofloxacin

3

Pyrazinamide

4

Doxycycline

An elevated body temperature of 103 °F, cough, headache, muscle aches, chest pain, severe joint pain and night sweats are symptoms of coccidioidomycosis, a fungal infection. Pregnant women can safely take amphotericin B because the drug will not affect the fetus. Doxycycline is a tetracycline that may lead to discoloration of the teeth in the newborn. Ciprofloxacin is a broad-spectrum antibiotic used to treat various bacterial infections and is ineffective with a fungal infection. Pyrazinamide is one of the first-line treatments for tuberculosis.

A client has been admitted to the hospital with a diagnosis of methicillin-resistant Staphylococcus aureus (MRSA) in the urine. The client has a urinary catheter in place. No private rooms are available. Which room assignment would be most appropriate for this client?

1

Roommate is bedridden and uses a bedpan for urination.

Correct 2

Roommate has MRSA in the urine. The roommate is ambulatory, but confused.

3

Roommate has a urinary catheter.

Incorrect 4

Roommate is alert and oriented with a diagnosis of pneumonia but practices good hygiene when using the bathroom.

If a private room is not available, the client may be placed in a room with another client who has the same active infection with the same microorganism if no other infection is present. The roommate with a urinary catheter, who is bedridden, or who has pneumonia does not indicate MRSA in the urine.

A client with tuberculosis asks the nurse about the communicability of the disease. Which is the best response by the nurse?

Correct 1

"Untreated active tuberculosis is communicable."

2

"With the newer long-term therapies, tuberculosis is not communicable."

3

"Tuberculosis is communicable during the primary stage."

4

"Tuberculosis is not communicable at this time."

The statement that untreated active tuberculosis is communicable is an accurate statement; treatment is necessary to stop communicability. The statement that tuberculosis is not communicable at this time is false reassurance; untreated active tuberculosis is communicable. Tuberculosis is not communicable when there is no active infection; the primary complex refers to the presence of a primary (Ghon) tubercle and enlarged lymph nodes and is the initial response to exposure; active disease may or may not occur. Tuberculosis is a communicable disease; close contacts should be screened via a skin test.

A nurse is assessing clients who are to be given the smallpox vaccination. Which client should the nurse remove from the immunization line for medical counseling?

1

75-year-old man with Parkinson disease

Correct 2

45-year-old woman with breast cancer

3

20-year-old healthy woman

4

50-year-old man with diabetes mellitus

The smallpox vaccine should not be given to individuals who may be immunocompromised as a result of therapy for cancer. There is no contraindication to giving the smallpox vaccination to a healthy woman, a client with diabetes mellitus, or a client with Parkinson disease.

In clients with Goodpasture syndrome, what is the pathophysiology of this syndrome?

1

Antigen-specific IgE binds to the surface of basophils and mast cells

Correct 2

Auto-antibodies attack the glomerular basement membrane and neutrophils

3

Rapid and systemic reactions of the organs within seconds after exposure to an allergen

4

Autoimmune destruction of the lacrimal and salivary mucus-producing glands

Goodpasture syndrome is an autoimmune disorder in which auto-antibodies attack the glomerular basement membrane and neutrophils, especially in the lungs and kidneys. A client may have allergies when an antigen-specific IgE binds to the surface of basophils and mast cells, which stimulates the production of vasoactive amines. Rapid and systemic reactions of the organs within seconds after exposure to an allergen may lead to anaphylactic shock. A client with Sjögren’s syndrome may have the autoimmune destruction of the lacrimal and salivary mucus/moisture-producing glands.

Which medications act by binding with integrase enzyme and prevent human immunodeficiency virus (HIV) from incorporating its genetic material into the client’s cell? Select all that apply.

I

Ritonavir

2

Nelfinavir

3

Tenofovir

Correct 4

Raltegravir

Correct 5

Elvitegravir

Raltegravir and elvitegravir are integrase inhibitors. They act by binding with integrase enzyme and prevent HIV from incorporating its genetic material into the client’s cell. Ritonavir and nelfinavir are protease inhibitors. They act by preventing the protease enzyme from cutting HIV proteins into the proper lengths needed to allow viable virions to assemble and bud out from the cell membrane. Tenofovir is a nucleotide reverse transcriptase inhibitor. It acts by combining with reverse transcriptase enzyme to block the process needed to convert HIV ribose nucleic acid into HIV deoxyribose nucleic acid.

A client has been prescribed tacrolimus for immunosuppressant therapy. Which drug safety alert should the nurse mention?

Correct 1

Do not consume grapefruit or grapefruit juice.

2

Take acetaminophen before taking tacrolimus.

3

Once-a-day doses should be taken at bedtime (at least initially).

4

The client should make sure to report any vivid or bizarre dreams.

Tacrolimus is a calcineurin inhibitor prescribed in immunosuppressant therapy. The client should not consume grapefruit or grapefruit juice because a substance in the grapefruit prevents the metabolism of these medications and may increase their toxicity. Monoclonal antibodies may cause fever, chills, headache, and chest pain among the adverse effects; acetaminophen should be taken before this drug is administered. When a client is prescribed efavirenz, a once-a-day dose should be taken at bedtime to help the client cope with the side effects, which may include dizziness and confusion. The nurse should also suggest that the client report experiencing vivid and sometimes bizarre dreams.

Scabies

2

Pediculosis

3

Herpes zoster

Correct 4

Cutaneous anthrax

Cutaneous anthrax is an infection caused by the spores of Bacillus anthracis. The primary clinical manifestation is the presence of an ulcer with a vesicular ring, induration, and erythema on the skin. The clinical manifestation of scabies is the presence of visible horizontal white skin ridges. The clinical manifestations of pediculosis are visible white flecks attached to the hair shaft near the scalp. The clinical manifestations of herpes zoster are the presence of fluid-filled blisters on the skin (shingles).

Which medications prescribed to a client after a kidney transplant surgery may require the client to visit a dentist? Select all that apply.

1

Sirolimus

Correct 2

Everolimus

3

Prednisone

Correct 4

Cyclosporine

5

Prednisolone

Everolimus is an antiproliferative drug that inhibits cell division in activated lymphocytes. Everolimus could cause gingival hyperplasia as a side effect. Cyclosporine may also cause gingival hyperplasia. Therefore the client may have to visit a dentist because of these medications. Sirolimus may cause leukopenia and thrombocytopenia. Prednisone and prednisolone may cause hypertension and hyperglycemia.

Which type of hypersensitivity reaction is present in a client with a body temperature of 102 °F, severe joint pain, rashes on the extremities, and enlarged lymph nodes from serum sickness?

1

Delayed reaction

2

Cytotoxic reaction

3

Immediate reaction

4

Immune complex-mediated reaction

4 Serum sickness is a type III immune complex-mediated reaction. A delayed reaction is a type IV hypersensitivity reaction that may include poison ivy skin rashes, graft rejection, and sarcoidosis. A cytotoxic reaction is a type II hypersensitivity reaction that includes autoimmune hemolytic anemia, Goodpasture syndrome, and myasthenia gravis. An immediate reaction is a type I hypersensitive reaction that includes allergic asthma, hay fever, and anaphylaxis. Test-Taking Tip: Serum sickness is manifested by elevated body temperature, severe joint pain, rashes on limbs, and enlarged lymph nodes. Recall the type of hypersensitivity reaction to which serum sickness belongs.

A client with tuberculosis is prescribed isoniazid. What statements should the nurse tell the client? Select all that apply.

Correct 1

"Take the drug on an empty stomach."

Incorrect 2

"Report any changes in vision to your primary healthcare physician."

Correct 3

"Take daily multiple vitamins that contain B-complex."

4

"Wear protective clothing when going outdoors during the day."

Correct 5

"Report darkening of the urine or a yellowish skin discoloration."

Isoniazid should be taken on an empty stomach because food prevents absorption of the drug. Multiple vitamins that contain the vitamin B-complex should be taken along with isoniazid because the drug depletes vitamin B. A client on isoniazid should report darkening of the urine and yellowish skin discoloration because these conditions are signs of liver toxicity. A client on ethambutol should be taught to report changes in vision. A client on pyrazinamide is instructed to wear protective clothing if he or she will be exposed to sunlight.

A client with acquired immunodeficiency syndrome (AIDS) reports speech, gait, and vision difficulty. The nurse observes the client is confused and lethargic. Which microorganism is most likely responsible for this condition?

Incorrect 1

Candida albicans

Correct 2

Toxoplasma gondii

3

Pneumocystis jiroveci

4

Mycobacterium tuberculosis

Toxoplasmosis encephalitis is an opportunistic infection caused by Toxoplasma gondii. The symptoms of toxoplasmosis encephalitis are speech, gait, and vision difficulty along with confusion and lethargy. An overgrowth of Candida albicans causes mouth pain and difficulty swallowing. Pneumocystis jiroveci causes pneumonia in a client with acquired immunodeficiency syndrome (AIDS). Mycobacterium tuberculosis causes tuberculosis in a client with AIDS

Why would a primary healthcare provider recommend that a client with acquired immunodeficiency syndrome (AIDS) and Kaposi’s sarcoma (KS) wear hats and long sleeves?

Correct 1

To maintain a normal appearance

2

To reduce pain

3

To promote healing

4

To prevent infection

Clients with KS lesions may be advised to wear hats, makeup, or long sleeves to maintain a normal appearance. Pain associated with KS lesions is treated with analgesics and comfort measures. Modified burrow’s solution soaks may promote healing in some clients with KS. The cleaning and dressing of KS lesions will prevent infections.

When taking the blood pressure of a client who has acquired immunodeficiency syndrome (AIDS), what must the nurse do?

1

Don clean gloves.

2

Use barrier techniques.

3

Put on a mask and gown.

Correct 4

Wash hands thoroughly.

Because this procedure does not involve contact with blood or secretions, additional protection other than washing the hands thoroughly is not indicated. Donning clean gloves and using barrier techniques are necessary only when there is risk of contact with blood or body fluid. A mask and gown are indicated only if there is a danger of secretions or blood splattering on the nurse (for example, during suctioning).

A client has undergone scratch testing but the causative allergen is yet to be identified. What would be the next step to confirm a strongly suspected allergen?

1

Administer allergy shots

Correct 2

Begin intradermal testing

3

Request that a client take an oral food challenge

4

Begin radioallergosorbent testing

An intradermal test is administered when a strongly suspected allergen tests negative with the scratch test. A testing dose of sera is injected intradermally into the arm and any signs of allergy are observed to confirm the allergen. Allergy shots are a therapeutic method of reducing sensitivity to a known allergen when exposure cannot be avoided. An oral food challenge is done to confirm an ingested allergen if skin testing completely fails. Radioallergosorbent testing is useful to measure immunoglobulin E levels to ascertain the presence of an allergic reaction.

While assessing a client with acquired immunodeficiency syndrome (AIDS), the nurse suspects that the client has developed cryptococcosis. Which clinical manifestations support the nurse’s suspicion of a cryptococcosis infection? Select all that apply.

Correct 1

Seizures

Incorrect 2

Dyspnea

Correct 3

Blurred vision

Correct 4

Neurologic deficits

5

Enlarged lymph nodes

Seizures, neurologic problems/deficits, and blurred vision are the manifestations of cryptococcosis. Cryptococcosis is a debilitating meningitis and can be a widely spread infection in clients who have AIDS. It is caused by Cryptococcus neoformans. Histoplasmosis is a respiratory infection caused by Histoplasma capsulatum, which progresses to widespread infection in a client with AIDS. The symptoms of histoplasmosis are dyspnea and enlarged lymph nodes

Which throat manifestations are the key features for a client with acute viral pharyngitis? Select all that apply.

Incorrect 1

Petechiae on the soft palate

Correct 2

Scant or no tonsillar exudate

Correct 3

Slight erythema of the pharynx and tonsils

4

Severe hyperemia of the pharyngeal mucosa

5

Erythema of the tonsils with yellow exudates

The throat manifestations of acute viral pharyngitis are scant or no tonsillar exudate and slight erythema of the pharynx and tonsils. Petechiae on the soft palate, severe hyperemia of the pharyngeal mucosa, and erythema of the tonsils with yellow exudates are the manifestations of acute bacterial pharyngitis.

Arrange in order the steps involved in skin testing for an allergen causing a type I allergic reaction.

The first step is to discontinue corticosteroid medications 2 weeks prior to the test to prevent a suppressed immune response. Then, on the day of testing, the test site (the inner side of the forearm) should be cleaned with soap and water to remove surface contaminants. Then drops of sera containing a known allergen should be placed on the skin. The skin is then scratched through the drop with the use of a skin testing needle. Control drops are also applied to compare reactions. Then positive signs for allergic reactions, such as erythema or wheal formation, should be observed. Finally the client should be discharged after a topical steroid is applied on the skin.

Test-Taking Tip: In this Question Type, you are asked to prioritize (put in order) the options presented. For example, you might be asked the steps of performing an action or skill such as those involved in medication administration.

The skin of a client with syphilis is illustrated in the image. Which stage of syphilis is present in the client?
3202827598

Correct 1

Late

2

Latent

Incorrect 3

Primary

4

Secondary

Syphilis is a sexually transmitted disease caused by Treponema pallidum. In the late stage of the disease, chronic destructive lesions (gummas) are present. Signs and symptoms are absent in the latent stage of syphilis. In the primary stage of syphilis, indurated lesions are present on the skin. In the secondary stage of syphilis, a mucous patch is present in the mouth.

Which antitubercular medications may increase a client’s risk for gout? Select all that apply.

Incorrect 1

Rifampin

2

Isoniazid

3

Bedaquiline

Correct 4

Ethambutol

Correct 5

Pyrazinamide

Pyrazinamide and ethambutol increase uric acid formation and may increase the risk of gout. Rifampin and isoniazid may lead to liver toxicity. Bedaquiline does not increase the risk of gout; it is used for multidrug-resistant tuberculosis

A client undergoing tuberculosis therapy reports eye pain that worsens when moving the eyes with decreased color vision. Which medication most likely is responsible for the client’s condition?

1

Rifampin

2

Isoniazid

3

Ethambutol

4

Pyrazinamide

Ethambutol Eye pain that is worsened when the eyes are moved with decreased color vision may be indicative of optic neuritis. Ethambutol, especially at high dosages, can cause optic neuritis, a condition that can result in blindness. Rifampin reduces the effectiveness of oral contraceptives, increasing the risk of an unplanned pregnancy, and can change bodily fluid orange. Isoniazid can deplete the body of the B-complex vitamins. Pyrazinamide increases uric acid formation and worsens gout.

A tuberculin skin test with purified protein derivative (PPD) tuberculin is performed as part of a routine physical examination. When does the nurse instruct the client to make an appointment so the test can be read?

1

1 week

2

12 hours

3

24 to 48 hours

4

48 to 72 hours

It takes 48 to 72 hours for antibodies to respond to the antigen and form an indurated area. The results of tuberculosis skin tests that are not read within this timeframe will not be accurate.

Which medication class helps to prevent human immunodeficiency virus (HIV) incorporating its genetic material into the client’s cell?

1

Entry inhibitors

2

Protease inhibitors

3

Integrase inhibitors

4

Reverse transcriptase inhibitors

Integrase inhibitors such as raltegravir and dolutegravir bind with integrase enzymes and prevent HIV from incorporating its genetic material into the host (client’s) cell. Entry inhibitors prevent the binding of HIV. Protease inhibitors prevent the protease enzyme from cutting HIV proteins into the proper lengths needed to allow viable virions to assemble. Reverse transcriptase inhibitors inhibit the action of reverse transcriptase

A client presents with a localized bacterial infection of mucous membranes. Which organism is most likely responsible for this condition in the client?

1

Giardia

2

Aspergillus fumigatus

3

Corynebacterium diphtheria

4

Mycobacterium tuberculosis

Corynebacterium diphtheria Diphtheria is a re-emerging infection that can be characterized by localized infection of mucous membranes or skin. It is caused by Corynebacterium diphtheria. Giardia, a parasite, causes giardiasis, a diarrheal illness known as traveler’s diarrhea. Aspergillosis is a lung disease caused by Aspergillus fumigatus, a fungus. Mycobacterium tuberculosis causes tuberculosis.

Human herpes virus-8

The image illustrates lesions in Kaposi sarcoma, a condition caused by Human herpes virus-8. Kaposi sarcoma develops as small, purplish-brown, raised lesions that are usually not painful or itchy. The skin and mucous membrane lesions can occur anywhere on the body. Candida albicans causes oral thrush that affects the hard and soft palates. Epstein-Barr virus causes oral hairy leukoplakia, which is a painless, white, raised lesion on the lips and tongue. Toxoplasma gondii causes toxoplasmosis encephalitis, which results in changes in mental status.

A nurse is concerned about the public health implications of gonorrhea diagnosed in a 16-year-old adolescent. Which should be of most concern to the nurse?

1

Interviewing the client’s parents

2

Finding the client’s sexual contacts

3

Instructing the client about birth control measures

4

Determining the reasons for the client’s promiscuity

2 Gonorrhea is a highly contagious disease transmitted through sexual intercourse. The incubation period varies, but symptoms usually occur 2 to 10 days after contact. Early effective treatment prevents complications such as sterility. The parents may be unaware that their child has gonorrhea. Most birth control measures do not protect against the transmission of sexually transmitted infections. Contracting venereal infection is not necessarily indicative of promiscuity.

While assessing a client with acquired immunodeficiency syndrome (AIDS), the nurse suspects that the client has developed cryptococcosis. Which clinical manifestations support the nurse’s suspicion of a cryptococcosis infection? Select all that apply.

Correct 1

Seizures

Incorrect 2

Dyspnea

Correct 3

Blurred vision

Correct 4

Neurologic deficits

5

Enlarged lymph nodes

Seizures, neurologic problems/deficits, and blurred vision are the manifestations of cryptococcosis. Cryptococcosis is a debilitating meningitis and can be a widely spread infection in clients who have AIDS. It is caused by Cryptococcus neoformans. Histoplasmosis is a respiratory infection caused by Histoplasma capsulatum, which progresses to widespread infection in a client with AIDS. The symptoms of histoplasmosis are dyspnea and enlarged lymph nodes.

14%
of students nationwide answered this question correctly.

View Topics

ID: 3203094436

Confidence: Nailed it

Stats

Issue with this question?

3.

Which throat manifestations are the key features for a client with acute viral pharyngitis? Select all that apply.

Incorrect 1

Petechiae on the soft palate

Correct 2

Scant or no tonsillar exudate

Correct 3

Slight erythema of the pharynx and tonsils

4

Severe hyperemia of the pharyngeal mucosa

5

Erythema of the tonsils with yellow exudates

The throat manifestations of acute viral pharyngitis are scant or no tonsillar exudate and slight erythema of the pharynx and tonsils. Petechiae on the soft palate, severe hyperemia of the pharyngeal mucosa, and erythema of the tonsils with yellow exudates are the manifestations of acute bacterial pharyngitis.

14%
of students nationwide answered this question correctly.

View Topics

ID: 3202802806

Confidence: Just a guess

Stats

Issue with this question?

4.

Arrange in order the steps involved in skin testing for an allergen causing a type I allergic reaction.

Incorrect

1.

Use a skin testing needle to scratch the surface of the skin.

Incorrect

2.

Apply topical steroid on the skin.

Incorrect

3.

Observe for a wheal formation on the skin.

Incorrect

4.

Discontinue corticosteroid medications.

Incorrect

5.

Clean the inner side of the forearm with soap and water.

Incorrect

6.

Place drops of sera containing the known allergen on skin.

The first step is to discontinue corticosteroid medications 2 weeks prior to the test to prevent a suppressed immune response. Then, on the day of testing, the test site (the inner side of the forearm) should be cleaned with soap and water to remove surface contaminants. Then drops of sera containing a known allergen should be placed on the skin. The skin is then scratched through the drop with the use of a skin testing needle. Control drops are also applied to compare reactions. Then positive signs for allergic reactions, such as erythema or wheal formation, should be observed. Finally the client should be discharged after a topical steroid is applied on the skin.

Test-Taking Tip: In this Question Type, you are asked to prioritize (put in order) the options presented. For example, you might be asked the steps of performing an action or skill such as those involved in medication administration.

11%
of students nationwide answered this question correctly.

View Topics

ID: 3202827536

Confidence: Nailed it

Stats

Issue with this question?

5.

The skin of a client with syphilis is illustrated in the image. Which stage of syphilis is present in the client?
3202827598

Correct 1

Late

2

Latent

Incorrect 3

Primary

4

Secondary

Syphilis is a sexually transmitted disease caused by Treponema pallidum. In the late stage of the disease, chronic destructive lesions (gummas) are present. Signs and symptoms are absent in the latent stage of syphilis. In the primary stage of syphilis, indurated lesions are present on the skin. In the secondary stage of syphilis, a mucous patch is present in the mouth.

22%
of students nationwide answered this question correctly.

View Topics

ID: 3202809469

Confidence: Just a guess

Stats

Issue with this question?

6.

Which antitubercular medications may increase a client’s risk for gout? Select all that apply.

Incorrect 1

Rifampin

2

Isoniazid

3

Bedaquiline

Correct 4

Ethambutol

Correct 5

Pyrazinamide

Pyrazinamide and ethambutol increase uric acid formation and may increase the risk of gout. Rifampin and isoniazid may lead to liver toxicity. Bedaquiline does not increase the risk of gout; it is used for multidrug-resistant tuberculosis.

17%
of students nationwide answered this question correctly.

View Topics

ID: 3203094061

Confidence: Just a guess

Stats

Issue with this question?

7.

A client undergoing tuberculosis therapy reports eye pain that worsens when moving the eyes with decreased color vision. Which medication most likely is responsible for the client’s condition?

Incorrect 1

Rifampin

2

Isoniazid

Correct 3

Ethambutol

4

Pyrazinamide

Eye pain that is worsened when the eyes are moved with decreased color vision may be indicative of optic neuritis. Ethambutol, especially at high dosages, can cause optic neuritis, a condition that can result in blindness. Rifampin reduces the effectiveness of oral contraceptives, increasing the risk of an unplanned pregnancy, and can change bodily fluid orange. Isoniazid can deplete the body of the B-complex vitamins. Pyrazinamide increases uric acid formation and worsens gout.

Test-Taking Tip: Be alert for details. Details provided in the stem of the item, such as behavioral changes or clinical changes (or both) within a certain time period, can provide a clue to the most appropriate response or, in some cases, responses.

46%
of students nationwide answered this question correctly.

View Topics

ID: 3028952565

Confidence: Just a guess

Stats

Issue with this question?

8.

A tuberculin skin test with purified protein derivative (PPD) tuberculin is performed as part of a routine physical examination. When does the nurse instruct the client to make an appointment so the test can be read?

Incorrect 1

1 week

2

12 hours

3

24 to 48 hours

Correct 4

48 to 72 hours

It takes 48 to 72 hours for antibodies to respond to the antigen and form an indurated area. The results of tuberculosis skin tests that are not read within this timeframe will not be accurate.

Test-Taking Tip: As you answer each question, write a few words about why you think that answer is correct; in other words, justify why you selected that answer. If an answer you provide is a guess, mark the question to identify it. This will permit you to recognize areas that need further review. It will also help you to see how correct your "guessing" can be. Remember: on the licensure examination you must answer each question before moving on to the next question.

75%
of students nationwide answered this question correctly.

View Topics

ID: 3203024783

Confidence: Just a guess

Stats

Issue with this question?

9.

Which medication class helps to prevent human immunodeficiency virus (HIV) incorporating its genetic material into the client’s cell?

1

Entry inhibitors

2

Protease inhibitors

Correct 3

Integrase inhibitors

Incorrect 4

Reverse transcriptase inhibitors

Integrase inhibitors such as raltegravir and dolutegravir bind with integrase enzymes and prevent HIV from incorporating its genetic material into the host (client’s) cell. Entry inhibitors prevent the binding of HIV. Protease inhibitors prevent the protease enzyme from cutting HIV proteins into the proper lengths needed to allow viable virions to assemble. Reverse transcriptase inhibitors inhibit the action of reverse transcriptase.

33%
of students nationwide answered this question correctly.

View Topics

ID: 3202921073

Confidence: Just a guess

Stats

Issue with this question?

10.

A client presents with a localized bacterial infection of mucous membranes. Which organism is most likely responsible for this condition in the client?

1

Giardia

Incorrect 2

Aspergillus fumigatus

Correct 3

Corynebacterium diphtheria

4

Mycobacterium tuberculosis

Diphtheria is a re-emerging infection that can be characterized by localized infection of mucous membranes or skin. It is caused by Corynebacterium diphtheria. Giardia, a parasite, causes giardiasis, a diarrheal illness known as traveler’s diarrhea. Aspergillosis is a lung disease caused by Aspergillus fumigatus, a fungus. Mycobacterium tuberculosis causes tuberculosis.

19%
of students nationwide answered this question correctly.

View Topics

ID: 3156449189

Confidence: Nailed it

Stats

Issue with this question?

11.

Which organism is responsible for the condition illustrated in the image?
3156449191

1

Candida albicans

Incorrect 2

Epstein-Barr virus

3

Toxoplasma gondii

Correct 4

Human herpes virus-8

The image illustrates lesions in Kaposi sarcoma, a condition caused by Human herpes virus-8. Kaposi sarcoma develops as small, purplish-brown, raised lesions that are usually not painful or itchy. The skin and mucous membrane lesions can occur anywhere on the body. Candida albicans causes oral thrush that affects the hard and soft palates. Epstein-Barr virus causes oral hairy leukoplakia, which is a painless, white, raised lesion on the lips and tongue. Toxoplasma gondii causes toxoplasmosis encephalitis, which results in changes in mental status.

46%
of students nationwide answered this question correctly.

View Topics

«
1
2
»

EAQ Home

Class Tools

Progress

Assignments

Grades

Custom Quiz

Elsevier Evolve

Credits

The nurse is preparing a teaching plan for clients receiving antitubercular medications. Which teaching plan needs correction?

1

Clients taking ethambutol should drink plenty of fluids.

Correct 2

Clients taking Isoniazid should take the drug with food.

3

Clients taking pyrazinamide should wear a hat while going out in the sun.

Incorrect 4

Clients taking rifampin should use other contraceptive methods even after stopping the medication.

Taking isoniazid with food should be corrected. The presence of food may slow or even prevent the absorption of isoniazid from the stomach. Therefore the client should take the medication on an empty stomach, either 1 hour before or 2 hours after eating. Ethambutol may increase uric acid formation. The client should drink plenty of water to reduce uric acid precipitation and kidney problems. Pyrazinamide is a photosensitive medication that may increase the risk of sunburn. Therefore the client should wear a hat and protect himself or herself from sun exposure. Rifampin may decrease the efficiency of oral contraceptives. Therefore the nurse should instruct the client to use an additional method of contraception even after stopping the medication.

The nurse teaches a group of clients that nutritional support of natural defense mechanisms indicates the need for a diet high in what nutrient or nutrients?

1

Essential fatty acids

2

Dietary cellulose and fiber

3

Tryptophan, an amino acid

Correct 4

Vitamins A, C, E, and selenium

Vitamins A, C, E, and selenium stimulate the immune system. The role of fatty acids in natural defense mechanisms is uncertain. Dietary cellulose and fiber have no known effect on natural defense mechanisms. Tryptophan has no known effect on natural defense mechanisms.

The image illustrates the toe of a client with a sexually transmitted infection (STI). Which other symptom should a nurse assess for in the client with this disease?
3203066447

1

Gummas on the skin

Incorrect 2

Loss of vision and perception

Correct 3

Mucopurulent cervical discharge

4

Unilateral scrotal pain and swelling

Gonorrhea is an STI caused by Neisseria gonorrhoeae. The image depicts skin lesions with disseminated gonococcal infection, which is also manifested by mucopurulent cervical discharge. Gummas on the skin accompanied with a loss of vision and perception are symptoms of tertiary syphilis. Unilateral scrotal pain and swelling are symptoms of a chlamydial infection.

A client receiving chemotherapy takes a steroid daily. The client has a white blood cell count of 12,000/mm3 (12 X 109/L) and a red blood cell count of 4.5 million/mm3 (4.5 X 1012/L). What is the priority instruction that the nurse should teach the client?

1

Omit the daily dose of prednisone.

Correct 2

Avoid large crowds and persons with infections.

Incorrect 3

Shave with an electric shaver rather than a safety razor.

4

Increase the intake of high-protein foods and red meats.

Moderate leukopenia increases the risk of infection; the client should be taught protective measures. Leukopenia is a side effect of cyclophosphamide, not prednisone. The platelet count has not been provided, so bleeding precautions are not indicated. Increasing the intake of high-protein foods and red meat are measures to correct anemia; protection from infection takes priority.

While assessing a client with presence of neurotoxicity, lymphoma, and hypertension, the medical history reveals that the client is on immunosuppressant drug therapy. Which drug class might have caused these conditions?

1

Corticosteroids

Incorrect 2

Cytotoxic drugs

3

Monoclonal antibodies

Correct 4

Calcineurin inhibitors

Calcineurin inhibitors such as cyclosporine act on T helper cells to prevent production and release of IL-2 and γ-interferon. This class of medications can cause adverse effects such as nephrotoxicity, lymphoma, hypertension, gingival hyperplasia, and hirsutism. Corticosteroids may cause peptic ulcer, osteoporosis, and hyperglycemia. Cytotoxic drugs may cause bone marrow suppression, hypertension, diarrhea, and nausea. Monoclonal antibodies may cause pulmonary edema, hypersensitivity reactions, fever/chills, and chest pain.

A client is admitted with full-blown anaphylactic shock that developed due to a type 1 latex allergic reaction. Which findings will the nurse observe upon assessment? Select all that apply.

Correct 1

Stridor

2

Fissuring

Correct 3

Hypotension

Correct 4

Dyspnea

5

Cracking of the skin

Full-blown anaphylactic shock produces stridor, hypotension, and dyspnea. Fissuring and cracking of the skin occurs in individuals with a type IV contact dermatitis.

The nurse educates a client on decreasing the risk of developing antibiotic-resistant infections. Which statement made by the nurse will be most significant?

Incorrect 1

"Wash your hands frequently."

Correct 2

"Do not skip any dose of your antibiotics."

3

"Save the unfinished antibiotics for later use."

4

"Stop taking the antibiotics when you feel better."

Antibiotic-resistant infection develops when the hardiest bacteria survive and multiply. This may happen when a client stops taking an entire course of antibiotics, which leads to infections that are resistant to many antibiotics. Therefore a client should not skip any dose of an antibiotic. Hand washing is required to prevent infections; it is not related to antibiotic-resistant infections. Antibiotics should not be stopped even if the client has started feeling better; the full course of treatment should be taken. Non-compliance in taking the full course of prescribed antibiotics can lead to an antibiotic-resistant infection. It is dangerous to take the unfinished antibiotics at a later time; it may prove fatal if the antibiotics are outdated.

Which organism causes Hansen’s disease?

Incorrect 1

Clostridium tetani

2

Haemophilus pertusis

Correct 3

Mycobacterium leprae

4

Legionella pneumophila

Mycobacterium leprae causes Hansen’s disease (leprosy). Clostridium tetani causes tetanus (lockjaw). Haemophilus pertusis causes pertussis (whooping cough) and Legionella pneumophila causes pneumonia (Legionnaires’ disease).

A nurse is preparing to obtain a blood specimen for culture and sensitivity from a client with an elevated temperature for the last 2 days. Place in order of priority the nursing actions that should be taken.

First, the procedure should be explained to the client. Then the specimen should be collected and sent to the laboratory. The antibiotic should be administered last. If it were administered before the specimen was collected, the test results might not indicate the cause of the fever.

A client who works manufacturing latex gloves presents with dryness, pruritus, fissuring, and cracking of the skin followed by redness and inflammation about 24 hours after contact. The nurse identifies it as an allergic reaction. Which condition most likely has occurred?

Incorrect 1

Type I allergic reaction

Correct 2

Type IV contact dermatitis reaction

3

Immune complex reaction

4

Cytotoxic hypersensitivity reaction

Type IV contact dermatitis is caused by the chemicals used in the manufacturing process of latex gloves. It is characterized by dryness, pruritus, fissuring, and cracking of the skin and occurs within 6 to 48 hours of contact. Type I allergic reaction is a response to the natural rubber latex proteins and occurs within minutes of contact with the proteins; skin redness, urticaria, rhinitis, and conjunctivitis are the clinical manifestations. Immune-complex reaction is a type III hypersensitivity. The kidneys, skin, joints, blood vessels and lungs are common sites for deposit. Cytotoxic hypersensitivity reactions involvethe direct binding of IgG or IgM antibodies to an antigen on the cell surface and is a type II hypersensitivity.

Test-Taking Tip: Consider the types of latex allergies and recollect the symptoms associated with each condition to choose the correct answer.

A client with jaundice reports fatigue, abdominal pain, loss of appetite, dark urine, nausea, and vomiting. Which pathogen is most likely responsible for the client’s condition?

Correct 1

Hepatitis C virus

2

Candida albicans

3

Varicella zoster virus

4

Cryptosporidium muris

A client with jaundice, fatigue, abdominal pain, loss of appetite, dark urine, nausea, and vomiting may have hepatitis, caused by hepatitis C virus. Candida albicans is a fungus that causes oral thrush, esophagitis, and vaginitis. Varicella zoster virus causes shingles, an erythematous maculopapular rash along dermatomal planes, pain, pruritis, and progressive outer retinal necrosis. Cryptosporidium muris causes gastroenteritis, watery diarrhea, abdominal pain, and weight loss.

A client with scleroderma reports having difficulty chewing and swallowing. What should the nurse recommend to safely facilitate eating?

Incorrect 1

Liquefy food in a blender.

Correct 2

Eat a mechanical soft diet.

3

Take frequent sips of water with meals.

4

Use a local anesthetic mouthwash before eating.

Scleroderma causes chronic hardening and shrinking of the connective tissues of any organ of the body, including the esophagus and face; a mechanical soft diet includes foods that limit the need to chew and are easier to swallow. Liquefied foods are difficult to swallow; esophageal peristalsis is decreased, and liquids are aspirated easily. Taking frequent sips of water with meals will not help; it is equally difficult to swallow solids and liquids, and aspiration may result. Using a local anesthetic mouthwash before eating is not necessary; oral pain is not associated with scleroderma.

STUDY TIP: Laughter is a great stress reliever. Watching a short program that makes you laugh, reading something funny, or sharing humor with friends helps decrease stress.

A client with mild diarrhea is diagnosed with a Clostridium difficile infection. Which is the first-line drug that would be used to treat this condition?

1

Rifaximin

2

Fidaxomicin

Incorrect 3

Vancomycin

Correct 4

Metronidazole

Metronidazole is the first line of treatment prescribed to clients with a Clostridium difficile infection. Rifaximin is used to treat traveler’s diarrhea caused by Escherichia coli. Fidaxomicin is reserved for clients who are at risk for the relapse of or have recurrent Clostridium difficile infections. Vancomycin is preferred for serious Clostridium difficile infections.

While assessing the mouth of a client with acquired immunodeficiency syndrome (AIDS), the nurse finds the condition illustrated in the image. Which pathogen is responsible for the client’s condition?
3202921058

1

Cryptosporidium

2

Candida albicans

3

Toxoplasma gondii

4

Histoplasma capsulatum

The nurse is caring for a client on antiretroviral therapy who has Pneumocystis jiroveci pneumonia. Which action is priority?

1

Assisting the client in eating and drinking

2

Maintaining fluid balance in the client

3

Providing adequate oxygenation for the client

4

Encouraging the client to perform breathing exercise

3

A client with radiation therapy for neck cancer reports, "I feel a lump while swallowing and foods get stuck." What does the nurse document in the client’s medical history?

Incorrect 1

Dysgeusia

Correct 2

Dysphagia

3

Xerostomia

4

Odynophagia

Dysphagia is having difficulty while swallowing. This characterizes pharyngeal and esophageal involvement, which further impedes eating. In this condition, the client may report a feeling of having a "lump" when swallowing and feeling that "foods get stuck." Dysgeusia is the loss of taste; clients will report that all food has lost its flavor. While xerostomia may contribute to difficulty swallowing, it is not the term used; xerostomia is used to indicate dry mouth. Odynophagia is painful swallowing; clients will report severe pain while swallowing.

A client has colorectal cancer and is receiving cetuximab. Which process does cetuximab inhibit?

Incorrect 1

Proteasome activity

2

BCR-ABL tyrosine kinase (TK)

3

Anaplastic lymphoma kinase

Correct 4

Epidermal growth factor receptors (EGFRs)

Cetuximab is an EGFR-tyrosine TK inhibitor that acts by inhibiting EGFRs in clients with colorectal cancer. Bortezomib inhibits proteasome activity in clients with multiple myeloma. Dasatinib acts by inhibiting BCR-ABL TK in clients with chronic myeloid leukemia. Crizotinib acts by inhibiting anaplastic lymphoma kinase (ALK) in clients with locally advanced or metastatic non-small cell lung cancer that is ALK positive.

Place the pathophysiologic process of tuberculosis infection in its correct order.

tuberculosis is a highly communicable disease caused by Mycobacterium tuberculosis. The process of infection in tuberculosis starts by formation of granulomatous inflammation by tuberculosis bacillus in lungs. This granulomatous inflammation then becomes surrounded by collagen, fibroblasts, and lymphocytes. The necrotic tissue then turns into a granular mass, called a caseation necrosis, which occurs in the center of the lesion. Then the areas of caseation undergo resorption, degeneration, and fibrosis. Finally, the necrotic areas undergo calcification or liquefaction.

The nurse is teaching a client about automatic epinephrine injectors. Which statement made by the client indicates a need for additional education?
Correct1
"I will keep the device in the refrigerator."
2
"I will keep the device away from light."
3
"If the cap is loose, I will obtain a replacement device."
4
"I will have at least two drug-filled devices on hand at all times."
The device should be protected from extreme temperatures. Therefore the device should not be refrigerated. The device should be protected from light. If the cap is loose or comes off accidentally, the client should obtain a replacement device. The client should have at least two drug-filled devices on hand in case more than one dose is required.
76%of students nationwide answered this question correctly.
View Topics
3156442181
Confidence: Nailed it
Stats
Issue with this question?
4.
What should the nurse expect to assess in a client with a type IV delayed hypersensitivity reaction? Select all that apply.
Correct 1
Edema
2
Bruising
Correct 3
Ischemia
Correct 4
Induration
Correct 5
Tissue damage
A type IV delayed hypersensitivity reaction consists of edema, ischemia, induration, and tissue damage at the site. Bruising is not a typical occurrence in a type IV delayed hypersensitivity reaction.
14%of students nationwide answered this question correctly.
View Topics
3202830409
Confidence: Pretty sure
Stats
Issue with this question?
6.
A client reports nasal discharge, sneezing, lacrimation, and itching all over the body. During an assessment, the client reports that he or she was exposed to dust and molds. The nurse observes swelling and a pink color at the itching site. Which class of drugs is best used to treat this condition in the client?
Correct1
Antihistamines
2
Antipruritic drugs
3
Mast cell-stabilizing drugs
4
Leukotriene receptor antagonists
Allergic rhinitis is a hypersensitivity reaction that occurs due to airborne substances such as dust, pollen, and molds. The symptoms of allergic rhinitis are nasal discharge, sneezing, lacrimation, and pruritus around the eyes, nose, and throat. Urticaria is a cutaneous reaction characterized by pink, raised, edematous, pruritic areas that vary in size and shape and may occur all over the body. Antihistamines are the best drugs used to treat allergic rhinitis and urticaria. Antipruritic drugs are effective only in relieving itching. Mast cell-stabilizing drugs are used to manage allergic rhinitis. Leukotriene receptor antagonists are most effective in treating allergic rhinitis and asthma.
73%of students nationwide answered this question correctly.
View Topics
3156444017
Confidence: Pretty sure
Stats
Issue with this question?
7.
Which reason should the nurse request that the healthcare provider increase the intravenous fluid infusion for an older client with an infection?
1
Pruritus
2
Erythema
Correct3
Acute confusion
4
General malaise
The nurse should consider the development of dehydration if acute confusion occurs in an older client with an infection. Additional fluids would not be helpful if pruritus, erythema, or general malaise develop in a client with an infection.
90%of students nationwide answered this question correctly.
View Topics
3202675407
Confidence: Just a guess
Stats
Issue with this question?
9.
Chart/Exhibit 1

The nurse is reviewing the medication chart of four clients. Which client’s medication chart indicates a need for correction?
Correct1
Client A
2
Client B
3
Client C
4
Client D
Enterococcus faecalis can cause a urinary tract infection, which is treated with penicillin G or ampicillin. E. faecalis in client A is resistant to medications such as streptomycin, vancomycin, and gentamicin and should be corrected by the nurse. All the rest are correct treatments. Staphylococcus epidermidis associated osteomyelitis in client B can be effectively treated with vancomycin. Streptococcus pneumoniae associated pneumococcal pneumonia in client C can be treated safely with cefotaxime and ceftriaxone. Klebsiella pneumoniae associated pneumonia in client D is treated with meropenem.

Test-Taking Tip: Apply your analytical thinking to recollect the medications that the pathogens are resistance to and select the correct answer.
55%of students nationwide answered this question correctly.
View Topics
3028949841
Confidence: Just a guess
Stats
Issue with this question?
11.
The nurse is caring for a client with sepsis who is hemodynamically stable. The client is complaining of abdominal pain. Which of these primary health care provider prescriptions should the nurse do first?
Correct1
Draw peripheral blood cultures.
2
Administer levofloxacin 500 mg intravenously over 30 minutes.
3
Administer 1 L intravenous bolus of Ringer’s lactate over 30 minutes.
4
Take the client to x-ray for an abdominal computed tomography (CT) scan.
This question requires the learner to recall the priority treatments for clients with sepsis. Mortality in septic clients increases by 7.6% for every hour an antibiotic is delayed. Because this client is hemodynamically stable, the priority is to draw the blood cultures so the antibiotic can be initiated as soon as possible. Administering the antibiotic before obtaining blood cultures could mask the infection, delaying appropriate treatment. Taking the client to x-ray before obtaining the blood cultures would delay antibiotic initiation.

Test-Taking Tip: Notice how the subjects of the questions are related and, through that relationship, the answers to some of the questions may be provided within other questions of the test.
50%of students nationwide answered this question correctly.
View Topics
3203037629
Confidence: Just a guess
Stats
Issue with this question?
12.
A client reports painful and bleeding gingivae, increased saliva with metallic taste, fetid mouth odor, decreased hunger, fever, and general malaise. Upon assessment, the nurse finds eroding necrotic lesions of the interdental papillae and bleeding ulcerations in the mouth. Which microorganism is responsible for this condition?
1
Candida species
Correct2
Spirochetes
3
Herpes simplex viruses
4
Staphylococcus species
Painful, bleeding gingivae, eroding necrotic lesions of interdental papillae, ulcerations that bleed, increased saliva with metallic taste, fetid mouth odor, anorexia, fever, and general malaise are symptoms of Vincent’s infection, which is caused by spirochetes. The Candida species cause oral candidiasis manifested as pearly, bluish white membranous lesions on the mucosa of the mouth and larynx. Herpes simplex viruses cause cold sores manifested as lesions on lips and vesicle formation. The Staphylococcus species causes parotitis, which is manifested as pain in the area of the gland and ear, absence of salivation, and purulent exudate from the gland.
67%of students nationwide answered this question correctly.
View Topics
EAQ Home
Class Tools
Progress
Assignments
Grades
Custom Quiz
Elsevier Evolve Credits

A primary health care provider prescribes airborne precautions for a client with tuberculosis. After being taught about the details of airborne precautions, the client is seen walking down the hall to get a glass of juice from the kitchen. What is the most effective nursing intervention?
1
Ensure regular visits by staff members.
Correct2
Explore what the precautions mean to the client.
3
Report the situation to the infection control nurse.
Incorrect4
Reteach the concepts of airborne precautions to the client.
Communication facilitates joint solution of the problem; the nurse must first determine the client’s understanding and perceptions before solutions to the problem can be attempted. Ensuring regular visits by staff members will not collect data about why the client is leaving the room. Reporting the situation to the infection control nurse abdicates the responsibility of the primary nurse. Reteaching the concepts of airborne precautions to the client may be done, but not until further assessment is performed to determine the reason why the client is leaving the room.

Test-Taking Tip: A psychological technique used to boost your test-taking confidence is to look into a mirror whenever you pass one and say out loud, "I know the material, and I’ll do well on the test." Try it; many students have found that it works because it reduces "test anxiety."
40%of students nationwide answered this question correctly.
View Topics
3203026303
Confidence: Just a guess
Stats
Issue with this question?
3.
The nurse has administered an immunosuppressant that acts on T helper cells to prevent production and release of IL-2 and γ-interferon to a client. Which side effects should the nurse monitor for in this client? Select all that apply.
Correct 1
Hirsutism
Correct 2
Gingival hyperplasia
3
Peptic ulcer
Correct 4
Nephrotoxicity
Incorrect5
Bone marrow suppression
Immunosuppressants that belong to calcineurin inhibitors may inhibit T helper cells to prevent production and release of IL-2 and γ-interferon. Hirsutism, gingival hyperplasia, and nephrotoxicity may occur as side effects of calcineurin inhibitors. Peptic ulcer may occur when corticosteroids are injected for immunosuppressant therapy. These drugs do not cause bone marrow suppression or alterations of the normal inflammatoryresponse. Bone marrow suppression may occur when cytotoxic drugs are administered.
0%of students nationwide answered this question correctly.
View Topics
3156442187
Confidence: Pretty sure
Stats
Issue with this question?
5.
What assessment findings indicate that an older client is at risk for developing an infection? Select all that apply.
Correct 1
Thin skin
Correct 2
Weak cough
Correct 3
Sluggish bowel sounds
4
Male-pattern baldness
Correct 5
Indwelling urinary catheter
Thin skin indicates a loss of protection by the integumentary system. A weak cough indicates a loss of protection by the respiratory system. Sluggish bowel sounds indicate a loss of protection by the gastrointestinal system. An indwelling urinary catheter is an invasive device which can introduce microorganisms into the client’s system. Male-pattern baldness does not indicate a loss of protection by a body system.
10%of students nationwide answered this question correctly.
View Topics
3203097024
Confidence: Nailed it
Stats
Issue with this question?
8.
Which drugs are used for the treatment of clients with rheumatoid arthritis that inhibit tumor necrosis factor-A? Select all that apply.
Incorrect1
Anakinra
Correct 2
Infliximab
Incorrect3
Abatacept
Correct 4
Etanercept
Correct 5
Golimumab
Biological response modifiers (BRMs) are the substances that modify immune responses by either enhancing an immune response or suppressing it. Infliximab, etanercept, and golimumab are BRMs used in the treatment of rheumatoid arthritis that inhibit tumor necrosis factor (TNF)-A. Anakinra is aninterleukin-1 receptor antagonist used in the treatment of rheumatoid arthritis. Abatacept is a selective T-lymphocyteco-stimulator modulator (T-cell inhibitor) used in the treatment of rheumatoid arthritis.
0%of students nationwide answered this question correctly.
View Topics
3202830473
Confidence: Pretty sure
Stats
Issue with this question?
10.
A registered nurse is evaluating the actions of a nursing student who is injecting an allergen in a client having a severe anaphylactic reaction to insect venom. Which action of the nursing student requires correction?
1
Rotating the sites for each injection
2
Aspirating for blood before giving the injection
Correct3
Injecting in an extremity close to a joint
Incorrect4
Observing the client for 20 minutes after an injection
The allergen extract should always be administered in an extremity away from a joint so that a tourniquet can be applied for a severe reaction. The injection sites should be rotated for each injection to prevent skin damage. Aspirating for blood before giving an injection should always be done to ensure that the allergen extract is not injected into a blood vessel. Systemic reactions are likely to occur immediately. Therefore the client should be observed for 20 minutes after the injection.

Test-Taking Tip: Understand that the nurse is injecting allergen. The nursing intervention should result in the positive outcome towards the health of the client.
50%of students nationwide answered this question correctly.
View Topics
EAQ Home
Class Tools
Progress
Assignments
Grades
Custom Quiz
Elsevier Evolve Credits

Share This
Flashcard

More flashcards like this

NCLEX 10000 Integumentary Disorders

When assessing a client with partial-thickness burns over 60% of the body, which finding should the nurse report immediately? a) ...

Read more

NCLEX 300-NEURO

A client with amyotrophic lateral sclerosis (ALS) tells the nurse, "Sometimes I feel so frustrated. I can’t do anything without ...

Read more

NASM Flashcards

Which of the following is the process of getting oxygen from the environment to the tissues of the body? Diffusion ...

Read more

Unfinished tasks keep piling up?

Let us complete them for you. Quickly and professionally.

Check Price

Successful message
sending